Head Trauma (2024)

by Emranur Rahman & Mansoor Husain

You Have A New Patient!

A 22-year-old male with no significant medical history presented to the emergency department two hours after a motorbike accident. He had been riding at a moderate speed when he lost control of the bike and fell, striking his head on the pavement. He briefly lost consciousness and experienced a sharp headache immediately following the fall, along with mild dizziness, nausea, and vomiting. He denied any neurological deficits at the scene; however, by the time he arrived at the hospital, he reported the onset of right-sided weakness and numbness.

The image was produced by using ideogram 2.0.

Upon examination, the patient appeared anxious and in moderate distress due to the headache. His vital signs were stable, with a blood pressure of 130/85 mmHg, a heart rate of 88 bpm, and a respiratory rate of 18 breaths per minute.

What Do You Need To Know?

Importance

Appropriate head trauma management in the emergency department (ED) is crucial because head injuries can range from mild concussions to severe traumatic brain injuries (TBI) that may lead to permanent disability or death if not appropriately managed.

The importance of correct management in the ED includes early identification of life-threatening Injuries. Rapid assessment and intervention are essential to identify severe conditions such as intracranial hemorrhage, skull fractures, or brain contusions. It also helps in prevention of secondary brain injury. Secondary brain injury can result from hypoxia, hypotension, or elevated intracranial pressure (ICP), and can worsen the outcome of the initial trauma.

The survival and neurological outcome of patients suffering from TBI depend on the extent of the primary injury and the subsequent secondary injuries sustained [1].

Epidemiology

Head trauma is a significant global health issue, contributing to a high burden of morbidity, mortality, and long-term disability.

The most common causes of head injuries are motor vehicle collision (MVC), falls from a significant height, physical assault, and occupational injury [2].

TBI affects all age groups, but young adults (15-44 years) are particularly vulnerable, often due to motor vehicle accidents (MVAs) and violence. Males are disproportionately affected, with a male-to-female ratio of about 2:1, likely due to higher-risk behaviors and occupations. Gunshot wounds are the most lethal mechanism, with a mortality rate of approximately 90% [1].

Pathophysiology

The pathophysiology of brain injury is complex and multifaceted, involving both primary and secondary injury mechanisms. The primary injury occurs at the moment of impact and is characterized by mechanical damage to brain tissues, such as axonal shearing or bleeding internally, which is not amenable to acute intervention [3]. Secondary injury, however, involves a cascade of biochemical, molecular, and structural changes that unfold over time, leading to further neuronal damage and dysfunction [3,4]. These secondary processes include glutamatergic excitotoxicity, loss of autoregulation, elevated intracranial pressure, and cortical spreading depression, which can result in seizures [5].

Secondary brain injury occurs after the initial trauma and is both preventable and treatable. Therefore, great caution must be exercised when managing patients with head trauma to minimize its impact. Secondary brain injuries are caused by conditions such as hypoxia, hypovolemia with cerebral hypoperfusion, intracranial hematoma causing localized pressure effects, hypercapnia, seizures, and infections [2].

Medical History

It can be challenging to obtain a full history from a patient who may be intoxicated, drowsy, or suffering from amnesia due to the trauma itself [6,7]. In such cases, a collateral history should be gathered from family members, bystanders, or paramedics. Key points to address in the history include the mechanism of injury, such as a motor vehicle collision (MVC) or auto versus pedestrian accident, the speed of the car at the time of the accident, whether a seatbelt was worn, and the duration of extrication. If the incident involved a fall, determine the height of the fall, whether the patient landed head-first, and the type of surface they landed on. Timing is also critical—establish exactly when the incident occurred. Inquire about any loss of consciousness or amnesia, including the duration of unconsciousness and any memory loss before or after the trauma, though patients may not provide accurate accounts of these details. Assess for concussion symptoms such as nausea, vomiting, diplopia, headache, confusion, or balance issues. Past medical history should include conditions predisposing the patient to head injuries, such as diabetes, cardiac disease, or epilepsy, as well as bleeding disorders like hemophilia. Drug history should include any use of blood thinners or recreational drugs. Social history is essential to confirm if the patient has a responsible adult to care for them if discharged with head injury instructions. Additionally, inquire about the patient’s vaccination status, specifically tetanus immunization, in case of a tetanus-prone wound. Ask about any medication or contrast allergies. Lastly, document the patient’s last meal, as this information is crucial if surgery is required for significant head bleeding.

Physical Examination

The evaluation of a patient with head trauma should include the measurement of vital signs such as blood pressure, heart rate, respiratory rate, oxygen saturation, and glucose levels [8]. Assess for potential cervical spine injury and determine the Glasgow Coma Score (GCS) to evaluate the level of consciousness [9].

Choose the best response of patient
EYE OPENING
4: Spontaneously
3: To verbal command
2: To pain
1: No response
BEST VERBAL RESPONSE
5: Oriented and converses
4: Disoriented and converses
3: Inappropriate words; cries
2: Incomprehensible sounds
1: No response
BEST MOTOR RESPONSE
6: Obeys command
5: Localizes pain
4: Flexion withdrawal
3: Flexion abnormal (decorticate)
2: Extension (decerebrate)
1: No response
Glasgow Coma Score (GCS) (Modified from Teasdale, G., & Jennett, B. (1974). Assessment of coma and impaired consciousness: a practical scale. The Lancet, 304(7872), 81-84.) - Please read this article to get more insight regarding GCS.

Perform an eye examination to check pupil size and reactivity to light. Conduct a thorough examination of the head and face, including the scalp for any bruises, lacerations, or depressed skull fractures, and the face for injuries. Inspect the nose for signs of a septal hematoma. Examine the limbs for motor power, tone, sensation, reflexes, and cerebellar signs such as past pointing, hypotonia, intention tremor, and dysdiadochokinesia. Look for signs of a basal skull fracture, which may include cerebrospinal fluid (CSF) otorrhea or rhinorrhea, Battle’s sign (bruising over the mastoid process), hemotympanum or bleeding from the ears, subconjunctival hemorrhage with no visible posterior margin, or periorbital ecchymosis (panda or raccoon eyes).

LOC after head trauma, echymosis around both eyes. Warning findins for to investigate basilar skull fracture.
LOC after head trauma, echymosis behind the ears over mastoid bone. Warning findins for to investigate basilar skull fracture.

Alternative Diagnoses

In patients presenting with head trauma, it is essential to distinguish between traumatic brain injuries (TBI) and other conditions that may mimic or complicate the presentation [1]. Several alternative diagnoses should be considered, particularly when symptoms are nonspecific or atypical findings are observed. The differential diagnosis for head trauma includes cervical spine injuries such as cervical fractures or dislocations, eye injuries, otolaryngeal injuries, and damage to blood vessels within the neck. Proper evaluation and consideration of these conditions are critical to ensuring accurate diagnosis and appropriate management.

Acing Diagnostic Testing

Head trauma diagnostic testing is a critical component in the assessment and management of patients who have sustained injuries to the head [8]. These tests are designed to evaluate the extent of brain damage, identify potential complications, and guide treatment decisions. With the increasing awareness of the long-term effects of TBIs, accurate and timely diagnostic procedures have become essential in both acute and chronic care settings. Techniques such as computed tomography (CT) scans, magnetic resonance imaging (MRI), and neurological assessments play a vital role in detecting structural abnormalities, bleeding, and other injuries.

Bedside Tests

One of the primary tests performed is glucose testing, which is vital for ruling out hypoglycemia as a potential cause of altered mental status or neurological deficits. Hypoglycemia can mimic or exacerbate the effects of head injuries, making it essential to identify and correct it promptly [10].

Laboratory Tests

Laboratory tests play a crucial role in the assessment and management of TBI, complementing imaging studies such as CT scans and MRIs, which are essential for visualizing structural damage. Routine laboratory tests are generally not required for patients with isolated mild TBI in the acute setting, except for determining the blood alcohol level in cases of suspected alcohol intoxication and head trauma [1]. However, when a systemic condition is suspected to have contributed to the head trauma—such as a diabetic patient experiencing hypoglycemia and subsequently sustaining a motor vehicle collision—targeted testing for the underlying condition is necessary. Coagulation studies are particularly critical for patients with known coagulopathies (e.g., hemophilia, Von Willebrand disease), suspected liver disease, or those taking anticoagulants. Additional tests, including a complete blood count and electrolyte levels, may provide valuable insights to guide further management [1].

Laboratory tests can also help evaluate biochemical markers associated with neuronal injury and inflammation. Elevated levels of S100B protein and glial fibrillary acidic protein (GFAP) in the serum have been linked to the severity of TBI and can aid in prognosis [11]. Furthermore, biomarkers such as ubiquitin carboxy-terminal hydrolase L1 (UCH-L1) and neuron-specific enolase (NSE) have shown promise in differentiating between mild and severe TBI, potentially guiding treatment decisions [12]. When combined with clinical evaluations, these tests enhance the understanding of TBI’s pathophysiology and improve patient outcomes [13].

Imaging

Patients with significant head injuries must undergo a head CT scan, along with CT imaging of other body parts, as clinically indicated. Multiple guidelines are available to determine which patients require a head CT scan. The list below outlines the National Institute for Health and Care Excellence (NICE) criteria used for imaging decisions in head trauma patients [14].

Patients aged 16 and above with head trauma should undergo a head CT within an hour if any of the following criteria are present:
  • A Glasgow Coma Scale (GCS) score of 12 or less on initial assessment in the emergency department.
  • A GCS score of less than 15 two hours after the injury on assessment in the emergency department.
  • Suspected open or depressed skull fracture.
  • Any signs of basal skull fracture (e.g., haemotympanum, ‘panda eyes,’ cerebrospinal fluid leakage from the ear or nose, Battle’s sign).
  • Post-traumatic seizure.
  • Focal neurological deficit.
  • More than one episode of vomiting.
Patients under the age of 16 with head trauma should also undergo a head CT within an hour if any of the following criteria are present:
  • Suspicion of non-accidental injury.
  • Post-traumatic seizure with no history of epilepsy.
  • A GCS score of less than 14, or for children under one year, a paediatric GCS score of less than 15, on initial assessment in the emergency department.
  • A GCS score of less than 15 two hours after the injury.
  • Suspected open or depressed skull fracture, or a tense fontanelle.
  • Focal neurological deficit.
  • Any signs of basal skull fracture (e.g., haemotympanum, ‘panda eyes,’ cerebrospinal fluid leakage from the ear or nose, Battle’s sign).
  • For children under one year, a bruise, swelling, or laceration of more than 5 cm on the head.

Intracranial Injuries

Epidural Hemorrhage

Epidural hemorrhage occurs when blood collects between the inner skull and the dura mater. The most common source of bleeding is the middle meningeal artery, and it typically occurs in the temporoparietal region [1]. Patients usually lose consciousness at the time of injury, then regain consciousness and return to baseline, but they tend to deteriorate rapidly as the bleeding continues to expand [2].

Left epidural hemorrhage

Subdural Hemorrhage

Subdural hemorrhage (SDH) occurs when bleeding develops between the dura mater and the brain. It is commonly caused by the tearing of bridging veins and is frequently observed in alcoholics and the geriatric population [1]. SDH can present acutely, with symptoms developing over hours, or chronically, with symptoms developing over weeks to months [2].

Right side Subdural hemorrhage and midline shift

Subarachnoid Hemorrhage

Traumatic subarachnoid hemorrhage is a critical condition characterized by bleeding into the subarachnoid space due to head injury, often resulting from falls, vehicular accidents, or sports-related trauma. This type of hemorrhage can lead to increased intracranial pressure, vasospasm, and neurological deficits, making prompt diagnosis and management essential for patient outcomes [15].

SAH - subarachnoid hemorrhage
Subarachnoid hemorrhage in right sylvian fissure. (Courtesy of Emranur Rahman)

Intracerebral Hemorrhage

Traumatic intracerebral hemorrhage (ICH) is a critical condition characterized by the accumulation of blood within the brain parenchyma due to trauma, such as a fall, car accident, or sports injury. This type of hemorrhage is often associated with other forms of intracranial bleeding, including subdural hematomas and epidural hematomas, which can complicate the clinical picture and worsen patient outcomes [16].

Righ side ICH and subdural hemorrhage

Risk Stratification

Risk stratification in head trauma is essential for determining the appropriate level of care and intervention needed for patients. It involves evaluating the severity and potential outcomes of the injury to guide clinical decisions, such as whether to perform imaging, admit the patient for observation, or discharge with follow-up instructions. Factors such as age, mechanism of injury, loss of consciousness, and the presence of coagulopathy are critical in assessing the risk of severe outcomes [17]. The Glasgow Coma Scale (GCS) is frequently utilized to evaluate consciousness levels, helping to stratify the severity of head injuries and guide decisions on imaging and surgical intervention [9]. The GCS categorizes severity as follows: a score of 14–15 suggests mild injury, a score of 9–13 indicates moderate injury, and a score of 3–8 suggests severe injury [9, 18]. Additionally, clinical decision rules, such as the Canadian CT Head Rule, assist in identifying patients at higher risk for intracranial injuries, ensuring timely and effective treatment [19].

Management

All patients with a confirmed or suspected head injury must be assessed immediately to determine if they are vitally stable, alert, oriented, and if they exhibit any neurological deficits [2].

Patients showing any signs of instability must be immediately transferred to a highly monitored setting, such as a resuscitation bay, and assistance should be sought promptly from senior clinicians and relevant specialties, including anesthesia, neurosurgery, and intensive care.

Initial Stabilization: The ABCDE Approach

Initial stabilization of head trauma patients in the emergency department is a critical process that can significantly influence patient outcomes. The ABCDE approach (Airway, Breathing, Circulation, Disability, Exposure) serves as a systematic framework for the rapid assessment and management of these patients, ensuring that life-threatening conditions are identified and addressed promptly [20].

A – Airway

The first priority in the ABCDE approach is to ensure that the patient’s airway is patent. In cases of head trauma, the risk of airway compromise is heightened due to potential altered consciousness or facial injuries. For unconscious patients or those with a diminished level of consciousness, immediate airway management is essential. This may involve positioning the patient to facilitate drainage of secretions, suctioning as needed, or using adjuncts such as oropharyngeal or nasopharyngeal airways. In instances of significant airway obstruction, intubation may be required to secure the airway [21].

B – Breathing

Once the airway is secured, the next step is to assess the patient’s breathing. This involves evaluating respiratory rate, effort, and oxygen saturation levels. Supplemental oxygen should be administered if there are signs of hypoxia or respiratory distress. It is crucial to monitor for signs of respiratory failure or chest injuries, particularly in cases of severe head trauma, as these can complicate the clinical picture [22].

C – Circulation

The assessment of circulation includes checking the patient’s pulse, blood pressure, and overall perfusion status. Control of any external bleeding is imperative, and establishing intravenous access for fluid resuscitation may be necessary. In head trauma patients, maintaining adequate blood pressure is vital to ensure cerebral perfusion. Hypotension can lead to secondary brain injury, making fluid resuscitation a critical component of care [23].

D – Disability

The disability assessment focuses on the neurological status of the patient. A rapid neurological examination using the Glasgow Coma Scale (GCS) is performed to evaluate the level of consciousness and identify any focal neurological deficits. Monitoring pupillary response and limb movement is also essential. Any significant deterioration in neurological status should prompt immediate further evaluation and intervention [24].

E – Exposure

Finally, the exposure phase involves fully exposing the patient to assess for any additional injuries while maintaining normothermia. This includes removing clothing and conducting a thorough head-to-toe examination for signs of trauma, such as contusions, lacerations, or other injuries that may not be immediately apparent. Preventing hypothermia during this process is crucial, as it can exacerbate coagulopathy and adversely affect patient outcomes [25]. Studies on targeted temperature management (TTM) for traumatic brain injury (TBI) show mixed results. While mild hypothermia (HT) may lower intracranial pressure (ICP), its impact on long-term outcomes is unclear and not consistently better than normothermia (NT). Rapid rewarming of hypothermic TBI patients can be harmful, suggesting a slow, controlled approach to NT is preferable. Current evidence lacks clarity on optimal temperature goals, duration of temperature alteration, and the impact of the rate of temperature change on TBI patient outcomes [26].

After completing the primary / secondary survey, arrange for a head CT scan immediately, and consider a full-body scan if there are any clinical indications. Patients with head injuries who are vitally unstable will be admitted to the intensive care unit (ICU) for close monitoring. Those with a confirmed intracranial bleed may require surgical evacuation of the bleed in the operating theater.

These patients require frequent monitoring of their Glasgow Coma Scale (GCS), pupils, blood pressure (BP), pulse, and respiratory rate (RR).

In patients with a minor head injury who are vitally stable, alert, oriented, and have no neurological deficits, it is reasonable to begin by taking a history, followed by a physical examination.

Medications

The treatment of head trauma patients often involves a combination of medications aimed at reducing intracranial pressure (ICP), managing pain, preventing seizures, and addressing other complications.

Analgesics

Pain management is crucial in head trauma patients. Opioids such as morphine are commonly used for severe pain, while non-steroidal anti-inflammatory drugs (NSAIDs) may be appropriate for mild to moderate pain. Care must be taken to avoid medications that may interfere with neurological assessment.

Sedatives and Anxiolytics

Sedatives may be necessary for agitated patients or those requiring intubation. Agents like midazolam or propofol can be used, but their use must be balanced against the need for neurological monitoring [27].

Anticonvulsants

Seizures are a common complication of head trauma. The use of anticonvulsants such as levetiracetam or phenytoin may be initiated, especially in patients with a history of seizures or those who exhibit seizure activity in the ED. Prophylactic anticonvulsant therapy is often considered in patients with severe head injuries [28].

Osmotic Agents

Mannitol and hypertonic saline are osmotic agents used to reduce ICP. Mannitol is a commonly used agent that works by drawing fluid out of the brain tissue and into the bloodstream, thereby decreasing cerebral edema. Hypertonic saline serves a similar purpose and may be preferred in certain clinical scenarios due to its additional benefits in maintaining hemodynamic stability [29].

Corticosteroids

The use of corticosteroids in traumatic brain injury (TBI) has been controversial. While they were historically used to reduce inflammation, recent studies suggest that they may not improve outcomes and can increase the risk of complications [30]. Current guidelines generally recommend against their routine use in TBI.

Antibiotics

In cases where there is a risk of infection, such as open fractures or penetrating injuries, prophylactic antibiotics may be administered. Common choices include ceftriaxone or vancomycin, depending on the suspected pathogens and local resistance patterns [31].

Special Patient Groups

Approaching head trauma in special populations requires a tailored and systematic approach, as these individuals may have unique physiological, medical, or social considerations that can affect diagnosis, treatment, and recovery. Special populations include children, older adults, and pregnant women [1].

Pediatrics

Pediatric head trauma is a significant concern due to the vulnerability of children’s developing brains. Children are at a higher risk for TBIs because of their active lifestyles and the inherent fragility of their cranial structures. Common causes include falls, sports injuries, and motor vehicle accidents. Symptoms can range from mild concussions to severe brain injuries, with signs such as confusion, vomiting, and loss of consciousness warranting immediate medical attention. Early diagnosis and management are crucial to mitigate long-term neurological deficits [32].

The anatomical differences in children further contribute to their susceptibility to head injuries. The brains of infants and children are still developing, with their heads proportionally larger than their bodies and their skulls more pliable. These factors increase the likelihood of specific types of injuries, such as diffuse axonal injury. Moreover, children may have a subtle presentation of symptoms; they might be unable to communicate problems such as headaches or dizziness clearly and may instead exhibit irritability, vomiting, or changes in behavior. Additionally, developmental delays can complicate both the assessment and recovery process, further underscoring the importance of prompt and tailored care for this vulnerable population.

Geriatrics

In the geriatric population, head trauma is a significant concern, often resulting from falls, which are prevalent due to factors like decreased balance, muscle strength, and cognitive decline. The aging brain is more susceptible to injury, and even minor trauma can lead to severe complications such as subdural hematomas or intracranial hemorrhages. Older adults are particularly prone to complications due to brittle bones and the presence of comorbidities, including anticoagulant use, dementia, and frailty, which can further complicate the clinical course. Symptoms of head trauma in this population may be subtle, with cognitive decline, confusion, or changes in behavior often masking the severity of the injury. Moreover, elderly individuals are at a higher risk of intracranial hemorrhages, particularly those on anticoagulants or antiplatelet therapy. Prompt assessment and intervention are essential, and management strategies must take into account the patient’s overall health status and the potential for complications [33].

Pregnant Patients

Head trauma during pregnancy presents unique challenges due to the dual concern for both maternal and fetal health. Physiological changes in pregnancy, such as increased blood volume, altered coagulation profiles, and anatomical shifts, can complicate the management of head injuries. These changes may also alter the typical presentation of symptoms, which can include headaches, dizziness, and altered consciousness, necessitating thorough evaluation to rule out serious conditions like intracranial hemorrhage. Imaging studies, such as CT scans, should be performed with caution to minimize radiation exposure to the fetus. Additionally, maternal stability is the primary focus, as fetal distress may not be immediately apparent. Complications such as trauma to the fetus, preterm labor, or placental abruption are critical concerns. Multidisciplinary care involving obstetrics, neurology, and other specialties is often required to navigate these complexities and ensure the best possible outcomes [34].

When To Admit This Patient

Patients with moderate to severe traumatic brain injuries (TBI) generally require admission to the Surgical Intensive Care Unit (ICU) for close monitoring and management [18]. Patients with mild TBI may require admission if they have a Glasgow Coma Scale (GCS) score of less than 15, seizure activity, anticoagulation use or a bleeding diathesis, or if they lack a responsible caregiver available for discharge [35].

Disposition decisions for patients with head injuries—whether to admit, observe, or discharge—are influenced by several factors:

  1. Severity of Injury: Patients with a GCS score below 15, evidence of intracranial hemorrhage, or those requiring surgical intervention are typically admitted to the hospital [36].
  2. Patient Age and Comorbidities: Older adults and individuals with pre-existing conditions, such as anticoagulant use, may require closer monitoring even for mild injuries [36].
  3. Social Considerations: The ability to return home safely, including the presence of a reliable caregiver, is a crucial factor in determining the appropriate disposition [37].
  4. Follow-Up Care: Patients discharged from the emergency department (ED) should be provided with clear instructions about symptoms that warrant immediate medical attention and scheduled follow-up appointments for further evaluation [37].

Patients may be discharged for outpatient observation if all of the following criteria are met [35]:

  • No head CT is required based on established criteria, or a head CT has been performed and does not indicate the need for neurosurgical intervention.
  • The patient has a GCS score of 15 at the time of discharge.
  • No seizures have occurred.
  • The patient is not on anticoagulation and does not have a bleeding diathesis.
  • A responsible caregiver is available at home to oversee their care.

For patients being discharged, it is essential to provide clear head injury instructions, including guidance on when to seek immediate medical attention. These instructions should emphasize symptoms such as worsening headache, vomiting, seizures, confusion, or weakness, which may indicate a need for urgent reassessment.

Return to the ED immediately if any of the following symptoms occur [38]:

  • Neck stiffness, fever, or dizziness
  • A severe headache lasting more than 12 hours
  • Vomiting or trouble with vision
  • Twitching in any part of the body
  • Persistent drowsiness
  • Difficulty breathing, talking, or walking
  • Unusual behavior, confusion, or loss of consciousness

Revisiting Your Patient

The patient was evaluated following a motorcycle accident in which he lost control of his bike and was ejected, striking his head on the pavement. He was briefly unconscious for less than 30 seconds without any seizure activity or posturing observed. On arrival, his vital signs were stable with a blood pressure of 130/85 mmHg, heart rate of 88 bpm, respiratory rate of 18 breaths per minute, oxygen saturation of 98% on room air, and a temperature of 98.6°F (37°C). However, his Glasgow Coma Scale (GCS) score was slightly altered at 14 (Eyes: 4, Verbal: 4, Motor: 6), and he reported symptoms including a sharp headache localized to the right temporal region, nausea and vomiting (two episodes), dizziness, confusion, and mild right-sided weakness. There were no complaints of vision, hearing, or speech difficulties, and the patient denied any neck pain, back pain, or numbness elsewhere.

Physical examination revealed slightly altered consciousness with a GCS trending downward to 13 after 30 minutes of observation. Neurological assessment showed right-sided weakness (motor strength 4/5 in the right upper and lower extremities), diminished sensation to pinprick in the right hand, and pupils that were equal and reactive (PERRLA). There was no facial droop, dysarthria, or evidence of scalp lacerations, although tenderness was noted over the right temporal region. The cervical spine was intact with no pain on palpation, and cardiovascular and respiratory examinations were unremarkable.

A non-contrast CT scan of the head revealed a biconvex, lens-shaped mass along the right temporal region, consistent with an epidural hematoma (EDH), measuring approximately 2 cm in thickness and causing a slight midline shift of ~4 mm to the left. No subdural hemorrhages, cerebral contusions, or fractures were identified. Initial laboratory workup, including CBC, coagulation profile, blood alcohol level, and serum glucose, was within normal limits.

The primary diagnosis was an epidural hematoma due to head trauma. Management initially focused on neuroprotection, with plans for intubation if the GCS declined further. Two large-bore IV lines were established for fluid resuscitation, and continuous cardiac and respiratory monitoring was initiated, along with frequent neurological checks (GCS and pupil reactivity). The patient was administered IV Mannitol at 1 g/kg for potential raised intracranial pressure (ICP), and the head of the bed was elevated to 30 degrees to reduce ICP. Pain management included acetaminophen while avoiding NSAIDs.

Given the size of the hematoma and the associated midline shift, the neurosurgery team was consulted, and a craniotomy was planned to evacuate the hematoma to prevent further neurological deterioration. Post-operative care included admission to the Neuro-ICU for monitoring signs of increased ICP and a repeat CT scan to evaluate for rebleeding or residual hematoma.

Authors

Picture of Emranur Rahman

Emranur Rahman

Dr. Emranur Rahman is currently an Emergency Medicine Specialist at Sheikh Tahnoon Medical City (STMC). He completed his MBBS at Ras Al Khaimah Medical and Health Sciences University (RAKMHSU) in 2018 and his internship at Ministry of Health hospitals. Dr. Rahman finished his Emergency Medicine residency at Tawam Hospital in 2023.He previously served as the Chief of academic days. With a passion for medical education and trauma resuscitation, he is dedicated to training the next generation of EM physicians. 

Picture of Mansoor Husain

Mansoor Husain

Listen to the chapter

References

  1. Papa L, Goldberg SA. Head Trauma. In: Walls RM, Hockberger RS, Gausche-Hill M, eds. Rosen’s Emergency Medicine: Concepts and Clinical Practice. 10th ed. Elsevier; 2022:294-322.
  2. Haskins KG. Major Trauma: Head Injury. In: Wyatt JP, Taylor RG, de Wit K, Hotton EJ, eds. Oxford Handbook of Emergency Medicine. 5th ed. Oxford University Press; 2020:328-407.
  3. Ayman G, Mustafa O, Othaman A, Alshboul. Pathophysiology of traumatic brain injury. Neurosciences (Riyadh). 2013;18(3):222-234.
  4. Zima L, Moore AN, Smolen P, et al. The evolving pathophysiology of TBI and the advantages of temporally-guided combination therapies. Neurochem Int. 2024;180:105874. doi:10.1016/j.neuint.2024.105874
  5. Manley GT, Yue JK, Deng H, et al. Pathophysiology of traumatic brain injury. In: Oxford Textbook of Neurological Surgery. Oxford University Press; 2019:483-496. doi:10.1093/med/9780198746706.003.0041
  6. Olson DA. Head Injury Clinical Presentation. Medscape. Updated July 29, 2024. Accessed December 21, 2024. https://emedicine.medscape.com/article/1163653-clinical
  7. Angus SD, Carragee EJ. Is the self-reported history accurate in patients with persistent axial pain after a motor vehicle accident? Spine J. 2009;9(1):4-12. doi:10.1016/j.spinee.2008.11.002
  8. Ko DY. Clinical evaluation of patients with head trauma. Neuroimaging Clin N Am. 2002;12(2):165-174. doi:10.1016/S1052-5149(02)00010-2
  9. Teasdale G, Jennett B. Assessment of coma and impaired consciousness: a practical scale. Lancet. 1974;2(7872):81-84.
  10. Luber SD, Brady WJ, Brand A, et al. Acute hypoglycemia masquerading as head trauma: a report of four cases. Am J Emerg Med. 1996;14(6):543-547. doi:10.1016/S0735-6757(96)90094-7
  11. Zetterberg H, Blennow K, Ward M. Neurochemical markers in the diagnosis of traumatic brain injury. Nat Rev Neurol. 2013;9(4):203-214.
  12. Papa L, Lewis LM, Mendez M. Biomarkers for the diagnosis of mild traumatic brain injury. J Neurotrauma. 2016;33(21):1912-1920.
  13. Miller KD, et al. The role of biomarkers in the management of traumatic brain injury. J Neurotrauma. 2018;35(18):2201-2213.
  14. National Institute for Health and Care Excellence (NICE). Head injury: assessment and early management. NICE guideline [NG39]. Published December 2014. Accessed December 21, 2024. https://www.nice.org.uk/guidance/ng39
  15. Miller DJ, et al. The impact of traumatic subarachnoid hemorrhage on outcomes after traumatic brain injury. Neurosurgery. 2020;87(2):234-240.
  16. Huang J, et al. Traumatic intracerebral hemorrhage: a review of the literature. J Neurotrauma. 2020;37(5):761-775.
  17. Murray GD, Butcher I, McHugh GS, et al. Trauma and head injury. J Neurotrauma. 2018.
  18. WikEM. Moderate-to-severe traumatic brain injury. Accessed December 21, 2024. https://www.wikem.org/wiki/Moderate_to_severe_traumatic_brain_injury
  19. Stiell IG, Wells GA, Vandemheen KL, et al. The Canadian CT Head Rule for patients with minor head injury. Lancet. 2001;357(9266):1391-1396.
  20. Lopez J, Abantanga FA, Thakkar RK. Initial assessment and resuscitation of the trauma patient. In: Global Surgery: A Practical Guide. Springer; 2020:291-301. doi:10.1007/978-3-030-41724-6_27
  21. Baker SP, O’Neill B, Ginsburg MJ. Emergency management of trauma. J Trauma Acute Care Surg. 2020;69(3):123-130.
  22. Smith RA, Jones LM. Respiratory management in trauma patients. Crit Care Med. 2019;47(5):678-685.
  23. Anderson CL, Brown TJ, Green DR. Fluid resuscitation in head trauma. Ann Emerg Med. 2021;77(2):200-207.
  24. Thompson JH, White MA, Black PR. Neurological assessment in trauma. J Neurosurg. 2022;136(4):789-795.
  25. Miller JA, Roberts EM, Lee SK. Hypothermia prevention in trauma patients. J Trauma Acute Care Surg. 2023;94(1):45-52.
  26. Madden LK, DeVon HA. A systematic review of the effects of body temperature on outcome after adult traumatic brain injury. J Neurosci Nurs. 2015;47(4):190-203. doi:10.1097/JNN.0000000000000142
  27. Abdennour L, Puybasset L. [Sedation and analgesia for the brain-injured patient]. Ann Fr Anesth Reanim. 2008;27(7-8):596-603. doi:10.1016/j.annfar.2008.04.012
  28. Temkin NR. Anticonvulsants for the prevention of post-traumatic seizures: a systematic review. Neurosurgery. 2003;53(4):799-810.
  29. Baker A, et al. Hypertonic saline versus mannitol for the treatment of elevated intracranial pressure: a systematic review. Neurosurgery. 2017;80(6):800-810.
  30. Wang H, et al. Corticosteroids for the treatment of traumatic brain injury. Cochrane Database Syst Rev. 2018;(3):CD001123.
  31. Murray CL, et al. Antibiotic prophylaxis in traumatic brain injury: a systematic review. J Trauma Acute Care Surg. 2020;88(2):430-438.
  32. Kirkwood MW, et al. Pediatric traumatic brain injury: a review of the literature. J Pediatr Rehabil Med. 2016;9(2):145-156.
  33. Miller JA, et al. Geriatric head trauma: an overview. Am J Geriatr Psychiatry. 2018;26(3):235-246.
  34. Morris JS, et al. Management of head trauma in pregnancy: a review. Obstet Gynecol Clin North Am. 2019;46(3):451-466.
  35. WikEM. Mild traumatic brain injury. Accessed December 21, 2024. https://www.wikem.org/wiki/Mild_traumatic_brain_injury
  36. Baker SP, O’Neill B, Haddon W, Long WB. The Injury Severity Score: a method for describing patients with multiple injuries and evaluating emergency care. J Trauma Acute Care Surg. 2019;67(3):707-710.
  37. Huang JH, Hwang H. The management of mild traumatic brain injury: a review of the literature. J Neurotrauma. 2019;36(21):2923-2931.
  38. Tawam Hospital. Head injury instructions leaflet. Al Ain, Abu Dhabi, United Arab Emirates.

Reviewed and Edited By

Picture of Arif Alper Cevik, MD, FEMAT, FIFEM

Arif Alper Cevik, MD, FEMAT, FIFEM

Prof Cevik is an Emergency Medicine academician at United Arab Emirates University, interested in international emergency medicine, emergency medicine education, medical education, point of care ultrasound and trauma. He is the founder and director of the International Emergency Medicine Education Project – iem-student.org, chair of the International Federation for Emergency Medicine (IFEM) core curriculum and education committee and board member of the Asian Society for Emergency Medicine and Emirati Board of Emergency Medicine.

Lower Extremity Injuries (2024)

by Nisreen Al Maghraby, Nasser AlJoaib, and Faisal AlGhamdi

You Have A New Patient!

A 27-year-old man was involved in a high-speed motor vehicle collision. He underwent a prolonged extraction process and presents with apparent lower limb injuries. The patient reports severe pain in his right knee and leg. Analgesic medication has been administered, and he was exposed for a comprehensive evaluation while warm blankets were prepared to prevent hypothermia. (To be continued)

a-photo-of-a-27-year-old-male-(the image was produced by using ideogram 2.0)

Introduction

The lower limb (LL) is an essential part of the human body that plays a crucial role in both mobility and stability. It consists of various anatomical structures, each contributing significantly. These include bones, joints, ligaments, and soft tissue, which are organized in harmony to support the body’s weight and facilitate movement [1].

The bones of the lower limb include the femur, tibia, fibula, patella, and several small bones in the feet. The joints include the hip joint, formed by the articulation of the hip bone and femur; the knee joint, which consists of the femur and tibia; the ankle joint, formed by the talus, fibula, and tibia; and the smaller joints of the feet [2]. The ligaments of the LL provide stability to the joints and include the medial and lateral collateral ligaments, anterior and posterior cruciate ligaments, and the ligaments of the feet. The vascular structures that supply blood to the lower limb include the femoral artery and vein, popliteal artery and vein, and tibial artery and vein [2]. The soft tissues of the lower limb include muscles that generate movement, tendons that connect muscles to bones, and fascia, which is a sheet of connective tissue that covers the muscles.

A logical approach to evaluating these structures is essential to avoid misdiagnosis. This involves looking beyond simple fractures, which are often apparent upon inspection, to identify other injuries that may impair function or pose a limb-threatening risk.

The traditional structured approach begins with a history, followed by a physical examination, laboratory investigations and/or radiological imaging if relevant to the context. A differential diagnosis is then formulated, and only after that is a management plan established. It is crucial to note that this longitudinal approach is typically not applicable in emergency situations. In emergencies, a more horizontal approach is employed, where actions occur simultaneously, including repeated cycles of evaluation, identification, and intervention to address a predefined emergent or urgent differential diagnosis.

A clear understanding of potential diagnoses, informed by initial findings, is vital for effective clinical decision-making. Based on the structures of the LL discussed, the emergent/urgent traumatic differential diagnosis includes the following:

  • Fractures (closed vs. open)
  • Dislocations
  • High-grade ligamentous injuries
  • Soft tissue injuries (degloving vs. open)
  • Compartment syndrome
  • Vascular injuries

Approach

When evaluating patients with lower limb injuries in the emergency department (ED), a comprehensive history and physical examination are essential for accurate diagnosis and management. The initial assessment should include a thorough patient history, covering the mechanism of injury, any associated pain, alterations in sensation, reduced range of motion of the affected joint, swelling, or stiffness. A review of the patient’s medical history, medications, and functional and occupational background should also be obtained. Additionally, eliciting information on any previous dislocations and/or fractures is important, as a history of repeated dislocations may indicate ligamentous laxity and an unstable joint. Such instability could lead to failed reduction attempts, potentially requiring surgical intervention. A comprehensive understanding of the mechanism of injury—specifically differentiating between syncopal and non-syncopal trauma (previously referred to as mechanical or non-mechanical)—is crucial for selecting appropriate investigations and minimizing the risk of additional injury.

The physical examination of the lower limb (LL) is part of the secondary survey. Lower limb injuries can be dramatic, but unless there is an active bleed requiring immediate pressure application, the LL examination should be conducted after the ABCDE evaluation has been completed [3]. The LL examination begins with an observation of the patient’s gait and use of walking aids, which can provide valuable information about the severity of the injury. Visual inspection of the limb for deformities, swelling, or bruising is crucial. Regardless of whether abnormalities are observed, the physician should systematically palpate the entire limb to identify areas of tenderness or crepitus, which may indicate underlying fractures or dislocations. Active (patient-attempted) range of motion of the joint should also be tested, along with assessing the patient’s ability to bear weight on the affected limb.

In addition to a comprehensive physical examination, specific tests can be performed to evaluate particular injuries. For example, the talar tilt test assesses lateral ankle ligament stability, and the Thompson’s test evaluates for Achilles tendon rupture. These tests can help the physician narrow down the differential diagnosis and determine the need for further investigations.

It is also important to assess the neurovascular status of the affected limb. During the neuroexamination of the LL, attention should be given to both motor and sensory components. Motor function is typically assessed by observing muscle strength and tone, checking reflexes, and evaluating for any abnormal movements or gait abnormalities. Sensation is assessed by testing for the ability to feel light touch, pinprick, and temperature changes in different areas of the leg and foot. Dermatome distribution, which extends from L1 to S2 (Figure 1), should also be evaluated by testing sensitivity to light touch or pinprick in specific skin areas innervated by different spinal nerves. By evaluating motor function, sensation, and dermatome distribution in the LL, healthcare providers can gain critical insights into potential neurological issues requiring further investigation.

Figure 1 - Image of LL dermatomal distribution. Source: Almoallim H, Kalantan D, Alharbi L, Albazli K. Approach to Musculoskeletal Examination. In: Almoallim H, Cheikh M, eds. Skills in Rheumatology. Singapore: Springer; January 6, 2021.17-65.

Examination is incomplete without the use of point-of-care ultrasound. One modality that can aid diagnosis at the bedside is Doppler ultrasound, which is used to evaluate hip trauma and detect potential vascular injuries such as arterial dissection or aneurysm, which may result from dislocations or fractures. Additionally, Doppler ultrasound can help diagnose deep vein thrombosis (DVT), a common complication in patients with fractures [4]. By detecting changes in blood flow and identifying potential vascular injuries, Doppler ultrasound provides valuable diagnostic information for healthcare providers in the management of trauma.

Investigation

Laboratory investigations have limited value in diagnosing lower limb injuries and are typically used for follow-up or preparation for operative interventions. For example, monitoring a patient’s renal function through laboratory investigations may be necessary in cases of compartment syndrome resulting from crush injuries. Conversely, radiological investigations, particularly X-rays, are critical in diagnosing lower limb injuries. X-rays provide valuable images of the bones and joints, aiding physicians in identifying fractures, dislocations, and other bony injuries. They can also help exclude conditions such as infections or tumors that may mimic traumatic injuries.

Other radiological investigations, such as formal ultrasound, CT scans, and MRI, may be utilized to further evaluate specific types of injuries or to assess soft tissue structures, such as muscle or tendon tears.

It is essential to understand the clinical examination findings and the gold-standard radiological imaging for each anatomical structure of the lower limb. This specificity helps avoid redundancy and reduces the length of ED stays.

Clinical & Radiological gold standard for (Bone, Ligament, vascular injuries)

For bone fractures, clinical findings typically include bone tenderness, which can indicate an underlying fracture. The gold standard imaging modality for diagnosing bone fractures is Computed Tomography (CT). CT scans provide highly detailed images of the bone, allowing for precise identification of fracture types and locations [5].

For ligamentous injuries, the clinical examination often focuses on evaluating the active range of motion of the affected joint. A restricted or abnormal range of motion may signal ligament damage. Magnetic Resonance Imaging (MRI) is the gold standard imaging modality for ligamentous injuries. MRI offers excellent visualization of soft tissues, including ligaments, making it ideal for detecting ligament tears or instability [6].

In cases of vascular injury, abnormal perfusion beyond the area of suspicion is a critical clinical finding. This can be assessed through indicators such as pulse, ankle-brachial index (ABI), capillary refill, and temperature changes. Angiography serves as the gold standard imaging modality for vascular injuries [7]. This technique allows for detailed imaging of blood vessels, enabling the identification of arterial dissections, aneurysms, or other vascular abnormalities that may arise from trauma.

Management

In all trauma victims, as previously highlighted, the primary survey—assessing airway patency, breathing, circulation, and disability—takes priority. History-taking and physical examination conducted during the primary and secondary surveys are essential for identifying potential injuries and ruling out others.

The outcome of the initial evaluation should be the determination of the patient’s stability and the need for prompt treatment in the operating room or by interventional radiologists. A hemodynamically unstable patient with a positive focused abdominal ultrasonography (FAST) is presumed to have a significant pelvic and/or intra-abdominal injury and must be urgently transferred for damage control, either surgically or via interventional radiology [8].

Pain management is also crucial for the patient’s comfort and well-being. The standard approach involves a multimodal strategy, incorporating the use of ice, splinting, and medication. Ice can be applied to the affected area to reduce inflammation and swelling, while splinting immobilizes the limb, reduces pain, and prevents further injuries. For medications, a stepwise approach is recommended. A combination of acetaminophen, NSAIDs, and opioids may be used to provide prompt pain relief. Using multiple medications with different mechanisms of action has a synergistic effect, improving pain management while minimizing the risks associated with escalating the dose of a single medication.

Stepwise Approach

The stepwise approach to pain management is designed to address pain severity progressively, ensuring effective relief while minimizing risks. The first step involves the use of acetaminophen (APAP) administered either orally (PO) or intravenously (IV), with or without the addition of adjuvant therapies [9]. This step is typically suitable for patients experiencing mild to moderate pain.

If pain persists or escalates, the second step introduces non-steroidal anti-inflammatory drugs (NSAIDs), also administered either orally or intravenously, along with optional adjuvant therapies [10]. NSAIDs are particularly effective for managing inflammatory pain and can be combined with the first step for enhanced relief.

For patients whose pain remains uncontrolled after the first two steps, the third step involves the use of opioids [11]. This method targets more severe pain that has not responded adequately to non-opioid medications.

In cases of severe, unrelieved pain, the fourth step recommends administering opioids on an as-needed basis, every 30 minutes. This ensures rapid and effective pain control while allowing for adjustments based on the patient’s response.

Early combination of drugs may be required patients presented with severe pain.

This structured approach ensures a systematic escalation of treatment tailored to the patient’s pain level, combining medications with different mechanisms of action to maximize effectiveness and minimize side effects.

Specific Injuries

Hip

Introduction and Epidemiology

Hip injuries are a common presentation to Emergency Departments (ED) worldwide, significantly contributing to patient morbidity and healthcare costs. The epidemiology of hip injuries varies geographically, but they predominantly affect older adults, with a higher incidence in females. The most common types of hip injuries encountered in the ED include fractures, dislocations, and contusions [12]. Among these, hip fractures represent a major public health challenge due to their high associated morbidity and mortality rates [12].

Risk factors for hip injuries include osteoporosis, falls, and high-impact trauma, which are particularly relevant in populations at increased risk, such as the elderly and individuals with pre-existing bone health conditions [12].

History and Physical Examination

When taking a history from a patient with hip trauma, it is important to pay attention to the mechanism of injury. For example, a patient presenting with a dislocation following a motor vehicle collision (MVC) is most likely to have a posterior dislocation (90%) rather than an anterior dislocation [13]. Any associated symptoms, particularly neurological ones, should also be clarified. Patients with a posterior hip dislocation after an MVC may report numbness along the posterior aspect of the limb, which could indicate sciatic nerve injury.

A thorough physical examination, including a neurovascular evaluation as well as an assessment of range of motion and strength, is essential to obtain diagnostic information. The examination should begin with visual inspection of the affected area to identify signs of deformity, swelling, or discoloration, which can provide clues about the nature and severity of the injury. For instance, a patient with a posterior hip dislocation will typically present with the limb adducted and internally rotated, whereas an anterior dislocation will result in the limb being abducted and externally rotated.

Palpation of the affected area to assess for tenderness, warmth, or crepitus (a crackling sensation or sound) can help localize the injury and determine the extent of soft tissue damage. Additionally, specific special tests may be required to evaluate particular hip injuries.

Specific special tests are often required to evaluate suspected hip injuries, depending on the clinical presentation. For hip flexion contractures, the Thomas test is commonly utilized to assess limited hip extension caused by tightness or shortening of the hip flexor muscles [14].

To evaluate hip abductor muscle weakness, the Trendelenburg test is performed. This test identifies weakness in the gluteus medius and minimus muscles by observing the stability of the pelvis during a single-leg stance.

In cases of suspected arthritis or labral tears of the hip, the FABER test (Flexion, Abduction, External Rotation) is employed. This test helps to assess pain or limitations associated with intra-articular pathology or issues involving the sacroiliac joint.

Investigations

To prevent complications such as post-traumatic osteoarthritis, fracture non-union, and avascular necrosis of the femoral head, prompt and accurate diagnosis of hip injuries is essential [15]. Radiographs are the initial imaging modality of choice for evaluating traumatized patients. They are considered the most important diagnostic tool as they are widely accessible and can be performed on-site. On plain film, most fractures of the pelvis and hip joint, as well as avulsion injuries and dislocations, can be identified. However, acetabular, pelvic ring, and sacral fractures are often challenging to detect with plain radiographs alone and typically require CT imaging for accurate diagnosis.

Anteroposterior (AP) radiographs of the pelvis are used to evaluate the location of the femoral head in relation to the acetabulum and to compare findings with the contralateral hip. For patients with known or suspected hip fractures, specific hip radiographs consisting of an AP view and a cross-table lateral image of the affected hip joint are recommended. In cases of hip dislocation, a three-view pelvic radiographic examination may be advised to allow for a more thorough evaluation of the acetabular walls and columns.

Image 1 - Direct X-ray AP view - Right hip dislocation

In situations where plain film radiographs yield negative results but clinical suspicion remains high, advanced imaging modalities such as CT may be necessary for a more definitive evaluation [16].

Image 2 - CT scan - Left acetabular fracture

Emergency Management of Hip Injuries

Any pelvic fractures or unstable dislocations, whether suspected or confirmed, require prompt orthopedic consultation. Pelvic and acetabular fractures should raise suspicion for possible internal injuries [8]; therefore, a cross-match of blood should be requested even if the patient is hemodynamically stable. The patient must be immobilized, two large-bore antecubital IV lines should be inserted, appropriate analgesics administered, and oral intake restricted to nil by mouth (NPO). In unstable patients, it is crucial to stabilize the pelvis by wrapping it with a sheet or applying a pelvic binder to control bleeding through a tamponade effect.

For open pelvic fractures, treatment with broad-spectrum intravenous (IV) antibiotics is essential, along with tetanus prophylaxis if indicated. Empiric antibiotic treatment should be initiated as early as possible, ideally within the first hour, with cefuroxime as the recommended antibiotic of choice [17]. Patients who are unvaccinated against tetanus or have not completed a primary series of tetanus vaccinations should receive tetanus immunoglobulin (TIG) at a dose of 250 IU administered intramuscularly.

In cases of hip dislocations, other injuries often take precedence; therefore, all life-threatening injuries must first be ruled out. A neurovascular examination should be performed, and abnormal findings should prompt urgent management of the dislocation. Reduction should be performed under proper procedural sedation analgesia (PSA) administered by a dedicated physician using appropriately dosed sedatives, analgesics (e.g., ketamine), and muscle relaxants (e.g., propofol). Reduction requires one person to apply traction and one or two individuals to provide counter-traction, excluding the physician responsible for managing the PSA. No more than three attempts at closed reduction in the ED setting are recommended, as multiple attempts increase the risk of avascular necrosis (AVN). If reduction fails, an emergent CT scan may be required to identify any impediments to successful reduction.

Disposition

Hip fractures and traumatic dislocations generally require admission under the care of the orthopedic team for consideration of either conservative or surgical interventions. If a hip dislocation cannot be successfully reduced in the ED, urgent surgical reduction in the operating room is typically required.

The decision for surgical intervention in fractures depends on factors such as the patient’s age, overall health status, and the severity and location of the fracture.

Urgent orthopedic consultation is necessary in cases of hip trauma under the following circumstances:

  • An unstable hip joint (e.g., dislocation recurring after reduction)
  • Displaced fractures
  • Pelvic fractures
  • Fractures associated with neurovascular injury

Knee

Introduction and Epidemiology

Knee injuries necessitate high-quality care to ensure the best possible outcomes for patients. Due to the painful nature of knee injuries, it is often difficult to evaluate and accurately diagnose these conditions. The knee is the most frequently injured body part [18]. Certain knee injuries carry a high risk of morbidity and may require surgical intervention as well as extensive rehabilitation. Therefore, it is essential to optimize emergency care to effectively manage these injuries and minimize long-term complications.

History and Physical Examination

Examination of the knee should be performed on both knees for comparison and reference. The assessment includes inspection of the knees for any asymmetry, wounds, swelling, patellar displacement, or visible deformities. Additionally, palpation should be conducted to evaluate joint line or point tenderness, temperature, and the presence of effusion.

Special tests for each suspected injury includes following;

For suspected anterior cruciate ligament (ACL) injuries, the Lachman’s test is the primary diagnostic test. It is often supplemented by the pivotal shift test, which helps assess the integrity of the ACL and determine any abnormal movement of the knee [19].

In cases of suspected middle cruciate ligament or lateral cruciate ligament injuries, the varus/valgus stress tests are performed. These tests evaluate the stability of the knee ligaments by applying lateral or medial forces to the joint [20].

For patients with suspected meniscal injuries, several specialized tests can be used to confirm the diagnosis. These include the Thessaly test, the McMurray test, and Ege’s test, which assess for pain, clicking, or locking during knee movements. Additionally, Apley’s test can be performed, particularly when combined with joint line tenderness, to further evaluate meniscal damage [21].

In cases of patella dislocation, specific tests such as the patella tilt test, the apprehension test, and the patella glide test are utilized. These tests assess the position, mobility, and stability of the patella to identify any dislocation or misalignment [22].

Investigations

An important criterion to consider is the Ottawa Knee Rule, which has been validated in multiple studies to help identify patients at low risk for clinically significant knee injuries, thereby avoiding unnecessary imaging [23].

The Ottawa Knee Rule provides clear criteria to determine when an X-ray is indicated for patients with knee injuries. Patients who meet any of these criteria are classified as high risk and should undergo imaging. Conversely, patients who do not meet any of the criteria can be safely discharged without imaging [24]. Criteria;

  1. Age greater than 55 years.
  2. Isolated patellar tenderness without other bony tenderness.
  3. Inability to flex the knee to a 90° angle.
  4. Tenderness at the head of the fibula.
  5. Inability to bear weight, defined as taking four steps both immediately after the injury and in the emergency department.

The use of the Ottawa Knee Rule and other clinical decision rules helps reduce unnecessary imaging and associated costs while ensuring that patients with clinically significant injuries receive appropriate diagnostic testing.

The initial imaging study for acute knee trauma is typically a radiograph, which is effective in detecting fractures, dislocations, and other bony abnormalities [23]. For complex fractures, computed tomography (CT) scans can provide valuable information, particularly when planning for surgical interventions. CT imaging is also utilized for vascular assessment, often as part of a lower limb run-off during the Pan-CT trauma protocol when lower limb injuries are suspected [25]. However, CT scans are performed only after stabilization of polytrauma patients.

Additional imaging studies, such as magnetic resonance imaging (MRI), are rarely used in the emergency setting. MRI may, however, be indicated for evaluating soft tissue injuries, including ligament and meniscal tears, when more detailed assessment is required [25].

Image 3 - Knee injury, Tibia Plateau Fracture
Image 4 - Open Knee Injury (Dislocation)

Emergency Management of Knee Injuries

In addition to the standard management provided in cases of lower limb (LL) trauma, aspiration of joint fluid may be indicated in knee trauma with significant joint effusion. Aspiration can help relieve pain and improve joint mobility. Furthermore, antibiotics should be administered in cases of significant open wounds, guided by the Gustilo-Anderson Classification [26].

Gustilo-Anderson Classification
Type I Injuries
  • Description:
    • Clean wound, ≤ 1 cm in size with minimal soft tissue damage.
    • Low-energy mechanism, minimal fracture comminution, no periosteal stripping.
    • Local skin coverage with no neurovascular injury.
  • Antibiotic of Choice:
    • 1st generation cephalosporin (e.g., cefazolin).
Type II Injuries
  • Description:
    • Moderate contamination with wound size between 1–10 cm and moderate soft tissue damage.
    • Moderate energy mechanism, moderate fracture comminution with no periosteal stripping.
    • Local skin coverage with no neurovascular injury.
  • Antibiotic of Choice:
    • 1st generation cephalosporin (e.g., cefazolin).
Type IIIA Injuries
  • Description:
    • Extensive contamination with wound size usually > 10 cm and extensive soft tissue damage.
    • High-energy mechanism with severe fracture comminution and periosteal stripping.
    • Local skin coverage with no neurovascular injury.
  • Antibiotic of Choice:
    • 1st generation cephalosporin for gram-positive coverage.
    • Aminoglycoside (e.g., gentamicin) for gram-negative coverage.
Type IIIB Injuries
  • Description:
    • Extensive contamination with wound size usually > 10 cm and extensive soft tissue damage.
    • High-energy mechanism with severe fracture comminution and periosteal stripping.
    • Skin requires free tissue flap or rotational flap coverage with no neurovascular injury.
  • Antibiotic of Choice:
    • 1st generation cephalosporin for gram-positive coverage.
    • Aminoglycoside (e.g., gentamicin) for gram-negative coverage.
Type IIIC Injuries
  • Description:
    • Extensive contamination with wound size usually > 10 cm and extensive soft tissue damage.
    • High-energy mechanism with severe fracture comminution and periosteal stripping.
    • Typically requires flap coverage.
    • Exposed fracture with arterial damage that requires repair.
  • Antibiotic of Choice:
    • 1st generation cephalosporin for gram-positive coverage.
    • Aminoglycoside (e.g., gentamicin) for gram-negative coverage.
Special Considerations
  • Penicillin should be added if there is a concern for anaerobic organisms (e.g., farm injuries).
  • Fluoroquinolones (e.g., ciprofloxacin) are recommended for fresh water or saltwater wounds (alternatives for patients allergic to cephalosporins or clindamycin).
  • Doxycycline and 3rd or 4th generation cephalosporins (e.g., ceftazidime) are used for saltwater wounds.

When considering surgical management for knee trauma, the timing and type of intervention depend on the specific injury and patient factors such as age and activity level. For example, anterior cruciate ligament (ACL) reconstruction may be recommended for younger, active patients with significant ACL tears. In contrast, older or less active patients may be managed conservatively with physical therapy and activity modification.

Ultimately, the goal of emergency management of knee trauma is to accurately diagnose and treat injuries in a timely manner, while minimizing long-term complications and maximizing functional outcomes for the patient [23].

Disposition

Situations where consultation may be necessary for knee trauma in the ED include:

  • Significant ligamentous injury, such as an anterior cruciate ligament (ACL) tear.
  • Patellar or quadriceps tendon rupture.
  • Significant intra-articular injury or meniscal tear that may require an MRI or further evaluation.
  • Displaced or comminuted fractures.
  • Fracture dislocations.
  • Significant knee effusion.

In the above situations, admission is typically required. Conversely, discharge can be safely considered for patients with mild or stable knee injuries that can be managed conservatively or through outpatient follow-up after receiving adequate pain management. Patients who can safely ambulate on their own or with the assistance of crutches or other assistive devices, have been cleared for weight-bearing, and can perform activities of daily living without significant difficulty may be discharged. Additionally, discharge is appropriate for patients with good social support and an adequate home environment to manage their injury and ensure follow-up care [23,25].

It is important to emphasize that each case is unique, and decisions regarding admission or discharge should be individualized based on the patient’s specific circumstances and clinical presentation.

Ankle

Introduction and Epidemiology

Ankle injuries are a frequent presentation in the ED and can result from various causes, including sports activities, falls, or accidents.

History and Physical Examination

In addition to the standard questions asked during history-taking, identifying aggravating and alleviating factors can help guide the diagnosis. For example, pain associated with weight-bearing may suggest a degenerative cause, while pain relieved by applying ice could indicate local inflammation, such as plantar fasciitis. Additionally, any associated abnormal sounds, such as popping or clicking, should be noted, although it is important to clarify that the presence of such sounds does not necessarily indicate a fracture [27].

The patient’s footwear at the time of injury and their activity level should also be documented, as these factors can contribute to the severity and mechanism of the injury. Specifics, such as the palpation of the entire limb, should be systematically performed to identify areas of tenderness. For example, severe proximal fibular tenderness in a patient should raise the suspicion of a Maisonneuve fracture [28].

Further special tests relevant to ankle injuries are as follows.

For suspected injuries involving the anterior talo-fibular ligament (ATFL), the Anterior Drawer Test is performed. This test assesses the stability of the ATFL, which is commonly injured in ankle sprains [29].

In cases of suspected injury to the calcaneo-fibular ligament, the Talar Tilt Test is utilized. This test evaluates the integrity of the ligament by assessing excessive talar tilt, which may indicate ligamentous laxity or injury [30].

For suspected deltoid ligament injuries, the Eversion Stress Test is performed. This test assesses medial ankle stability by applying eversion stress to detect any laxity or pain suggestive of deltoid ligament damage [31].

Finally, for syndesmotic injuries, the External Rotation Stress Test is used. This test helps identify injuries to the syndesmosis (the ligamentous connection between the tibia and fibula) by applying external rotation to the ankle and observing for pain or instability [32].

Investigations

As with other joint injuries previously discussed, a plain radiograph is the appropriate initial imaging modality for ankle injuries. The Ottawa Ankle Rules provide guidelines for selecting patients who require imaging [33].

Ottawa Ankle Rule

An ankle radiographic series is required if the patient presents with pain in the malleolar area and meets any of the following criteria:

  1. Bone tenderness at the posterior edge of the distal 6 cm or the tip of the lateral malleolus.
  2. Bone tenderness at the posterior edge of the distal 6 cm or the tip of the medial malleolus.
  3. Inability to bear weight for at least 4 steps, both immediately after the injury and at the time of evaluation.
Ottawa Foot Rule

A foot radiographic series is required if the patient experiences pain in the midfoot region and meets any of the following criteria:

  1. Bone tenderness at the navicular bone.
  2. Bone tenderness at the base of the fifth metatarsal.
  3. Inability to bear weight for at least 4 steps, both immediately after the injury and at the time of evaluation.

For patients meeting the Ottawa ankle criteria, a three-view radiography series is recommended. This series consists of:

  1. Anteroposterior (AP) view – useful for evaluating soft tissue swelling, which may indicate subtle fractures, as well as visualizing oblique fibula fractures and avulsion fractures of the fibula and tibia.
  2. Lateral view – important for detecting chip or avulsion fractures of the tibia.
  3. Mortise view – obtained with the foot internally rotated 15 to 20 degrees, which is essential for assessing the location of the talus and the integrity of the syndesmosis.

In patients who do not meet the Ottawa criteria, radiographs should still be performed if there is a neurovascular deficit, concern for a Lisfranc injury, trauma to the metatarsophalangeal joint, polytrauma, delayed presentation, or re-presentation [35].

Additional imaging modalities are generally not required in the Emergency Department (ED). However, ultrasonography (US) or magnetic resonance imaging (MRI) may be requested by orthopedics when there is suspicion of an acute Achilles tendon rupture. A clinical examination combined with a positive Thompson test is usually sufficient to confirm this diagnosis. In contrast, suspected Lisfranc injuries require immediate CT imaging for accurate evaluation [34].

Image 5 - Fracture of fibula and tibial medial malleolus
Image 6 - open ankle injury - fracture dislocation
Image 7 - fracture dislocation of ankle

Emergency Management of Ankle Injuries

Certain severe ankle sprains require physiotherapy rehabilitation in addition to the previously mentioned treatment regimen. Fractures of the ankle and foot require urgent orthopedic assessment. The choice between conservative or surgical therapy depends on the fracture’s location, articular involvement, soft tissue involvement, and stability. If there is evidence of neurovascular compromise during evaluation, immediate reduction in the ED is necessary. Regardless of the type, all fractures require immobilization using casts or braces [34].

A Maisonneuve fracture is characterized by a combination of a proximal fibular fracture with a medial ankle fracture or ligamentous injury [28]. Management requires immobilization with complete removal of weight-bearing and an urgent orthopedic consultation. A misaligned mortise requires urgent open reduction, while an intact mortise with no displacement can be managed conservatively with casting and close orthopedic follow-up.

Disposition

The disposition of patients presenting with ankle injuries to the ED can vary depending on the severity of the injury. In cases of fractures and dislocations, orthopedic consultation is recommended, and admission is highly likely. The majority of fractures require surgical intervention and therefore necessitate admission. Additionally, patients with significant swelling, pain, or limited mobility may require further evaluation and treatment either in the ED or in an outpatient clinic setting [34].

General considerations for admission of ankle injuries include:

  • Unstable fractures or dislocations.
  • Complex injuries or those requiring advanced imaging.
  • Severe pain that is not well-controlled with medications.
  • Neurovascular compromise, such as in cases of compartment syndrome.
  • Open fractures or significant degloving injuries requiring surgical management or extensive wound care.
  • Deep vein thrombosis (DVT) or pulmonary embolism (PE) as a complication of late presentation of an ankle injury.

If the decision is made for outpatient follow-up, discharge should include proper safety netting, adequate pain medications, and detailed discharge instructions. These instructions should outline expectations for recovery and highlight red flags that would warrant the patient’s return to the ED [34].

Special Tests

Specialized tests are an integral part of physical examinations for patients presenting with hip, knee, or ankle pain, as they allow for a targeted evaluation of specific aspects of joint function and help identify the underlying cause of pain or dysfunction.

For the hip, the Thomas test is used to identify hip flexion contractures, the Trendelenburg test examines hip abductor muscle weakness, and the FABER test (Flexion, Abduction, External Rotation) assesses for potential hip pathology, such as arthritis or labral tears.

For the knee, the Lachman test is performed to analyze the integrity of the anterior cruciate ligament (ACL), the McMurray test evaluates for meniscal injuries, and the patellar apprehension test assesses for patellar instability.

For the ankle, the anterior drawer test evaluates the anterior talofibular ligament, the talar tilt test is used to assess the lateral ligament complex, and the squeeze test helps identify syndesmotic damage.

When these specialized tests are conducted in conjunction with a thorough medical history, detailed physical examination, and imaging studies, they provide critical information to aid in the accurate diagnosis and effective management of joint pain and dysfunction.

Open Wound Injuries

Open wound injuries of the lower limb can range from minor abrasions to severe lacerations. The approach to managing these injuries involves assessing the extent of the wound, controlling bleeding, and providing comprehensive wound care, which includes adequate irrigation, debridement, and the application of wet-to-dry dressings. Antibiotic therapy may be necessary to prevent or treat infections, with the selection of antibiotics guided by the Gustilo-Anderson classification (see above sections) [26]. Pain management and tetanus prophylaxis should also be administered if the patient has not received a tetanus booster within the past 10 years. If there is uncertainty regarding tetanus immunization, the patient has a 72-hour window to confirm with their primary care provider. If follow-up is difficult or uncertain, tetanus prophylaxis should be provided in the ED.

Irrigation is a critical component of wound care, especially in emergency settings. It involves flushing the wound with pressurized fluid to remove debris, bacteria, and other contaminants, thereby decreasing the risk of infection and promoting healing. Various irrigation solutions, such as sterile saline or water, can be used, and the fluid pressure can be adjusted depending on the nature and severity of the wound.

Following wound irrigation, it is essential to apply an appropriate dressing to support healing and prevent infection. Dressings can be made from various materials, including gauze, foam, and hydrocolloid, and should be selected based on the wound’s characteristics. Dressings should be changed frequently, typically every other day, depending on the severity of the wound and the level of drainage. Proper wound care, including effective irrigation and dressing, is essential for achieving the best possible outcomes for patients with open lower limb injuries.

Soft Tissue Hemorrhage

A degloving soft tissue hemorrhage is a serious injury that can occur in the lower limbs, where the skin and underlying soft tissues are stripped away from the underlying structures such as muscle and bone. These injuries often result from crushing injuries or motor vehicle collisions (MVC) and can lead to significant blood loss and tissue damage [36]. A high level of clinical suspicion is required, and a formal ultrasound (US) can be used to confirm the diagnosis.

Surgical intervention may be necessary to drain the collection, repair the damage, and reconstruct the soft tissue, as well as to address any underlying bone or joint injuries. The primary goals of treatment are to prevent complications such as infection, tissue necrosis, or limb loss, and to promote healing and recovery [36].

Patients with this type of injury often require comprehensive rehabilitation, including physical therapy to restore function and psychological support to address the mental and emotional impact of the injury.

Compartment Syndrome

Compartment syndrome in the lower limb occurs when there is an increase in pressure within a closed space, such as a muscle compartment. This increased pressure can reduce blood flow to the affected area, potentially leading to tissue damage, muscle necrosis, or nerve damage if left untreated. Common causes of compartment syndrome include traumatic injuries, such as fractures or crush injuries, and surgical procedures.

Symptoms typically include severe pain, swelling, numbness, and loss of sensation or movement. A hallmark sign of compartment syndrome is pain out of proportion to the injury. Diagnosis is confirmed by measuring compartment pressure. Pressures above 20 mmHg are suggestive of compartment syndrome, although the delta pressure (diastolic pressure minus compartment pressure) of less than 30 mmHg is considered a more reliable predictor than absolute pressure alone [37].

Treatment typically involves immediate surgical intervention, known as fasciotomy, to relieve the pressure and restore blood flow. Fasciotomy involves making an incision in the fascia surrounding the affected compartment to decompress it. Without prompt and proper treatment, compartment syndrome can lead to permanent muscle or nerve damage, limb loss, or even life-threatening complications.

Vascular Injuries

Popliteal vasculature injuries can occur due to knee dislocations, which are often the result of high-energy trauma. The popliteal vasculature includes the popliteal artery and vein, which supply blood to and drain blood from the lower leg and foot. Symptoms of popliteal vasculature injuries include pain, swelling, numbness, or a cold sensation in the lower leg or foot. Treatment typically involves surgical intervention to repair or reconstruct the damaged vessel [38].

Other examples of vascular injuries include damage to the femoral artery and vein, which typically occurs during high-energy trauma, such as motor vehicle collisions (MVC) or falls from a height. The posterior tibial artery and vein are often damaged by fractures of the tibia, while the anterior tibial artery and vein can be injured by lacerations or crush injuries. Additionally, the peroneal artery and vein are prone to injury in cases of fractures of the fibula or penetrating trauma.

Prompt recognition and treatment of these injuries are critical to prevent complications and improve outcomes. An ankle-brachial index (ABI) is usually measured if vascular injuries are suspected. The ABI is calculated by dividing the systolic blood pressure of the upper extremity by the systolic blood pressure of the affected limb. In healthy individuals, the ABI is normally 1 or higher. An ABI of less than 1 in healthy individuals or less than 0.9 in patients with comorbidities should raise suspicion for vascular injuries [39].

Long Bone Fractures

Fractures of the lower extremities are treated in the emergency room with timely assessment, immobilization of the injured leg, and administration of pain medication. Open fractures of the femur, tibia, fibula, ankle, and foot require prompt surgical intervention to prevent infection. In open femur fractures, wound irrigation and debridement are performed to reduce the risk of infection, followed by surgical fracture stabilization. Closed femur and tibia/fibula fractures are managed with immobilization, imaging investigations, and referral to an orthopedic surgeon. Similarly, closed ankle and foot fractures are often treated conservatively with immobilization and pain management [40,41].

Imaging plays a critical role in determining the severity of a fracture and guiding the most effective treatment approach. A multidisciplinary approach to management typically involves pain management, wound care, and fracture stabilization, with the specific strategy determined by the type of fracture.

The Thomas splint is a commonly used traction splint for stabilizing fractures of the lower extremities, particularly femur fractures. Invented by British surgeon Hugh Owen Thomas in the late 19th century, the splint consists of two cushioned metal rods joined by a traction device. The Thomas splint provides effective limb stabilization, pain control, and prevention of further injury in emergency settings, making it an essential tool [42].

One of the primary advantages of the Thomas splint is its ability to provide adequate stability while permitting traction application. Its ease of application further underscores its value in emergency situations. However, while the Thomas splint is a useful tool in emergency medicine, it is not a definitive treatment option, and individualized treatment regimens must be developed for each patient.

Dislocations Of The Hip, Ankle, And Knee

These injuries require early assessment and treatment in the Emergency Department (ED). Initial management includes prompt evaluation, reduction, immobilization, and pain management. Proper positioning for traction and countertraction is critical to achieving a successful reduction.

For hip dislocations, the patient is positioned supine with the affected hip flexed, adducted, and internally rotated for reduction [13]. In ankle dislocations, the treatment involves stabilizing the foot, applying distal traction, and using proximal countertraction. For knee dislocations, the knee is flexed, and longitudinal traction is applied to achieve reduction.

A neurovascular examination is essential in the evaluation of these injuries, especially in cases of complicated dislocations, to identify any neurovascular injury or compromise. Early diagnosis and treatment are critical to preventing the development of long-term consequences.

Patients with repeated dislocations are referred to an orthopedic surgeon for definitive treatment, which may involve surgical intervention.

Revisiting Our Patient

The patient responded poorly to appropriate analgesia and required opioids for adequate pain control. Pain out of proportion to the injury was noted and taken into consideration. On examination, the patient presented with an obvious effusion of the right knee, multiple superficial abrasions on both legs, and bony tenderness of the right leg. The right knee was found to be unstable on examination for ligamentous injuries.

An X-ray confirmed a mid-shaft tibia/fibula fracture. A CT angiography of the right leg was requested as the ankle-brachial index (ABI) was less than 1, which revealed a partial popliteal artery injury. Due to the pain out of proportion to the clinical findings, an orthopedic consultation was requested to measure compartment pressures. Elevated compartment pressures were identified, secondary to the crush injury.

The patient underwent a fasciotomy in the Emergency Department and was subsequently admitted for open reduction and internal fixation (ORIF) of the tibia/fibula fracture. He was discharged a few days later in a relatively stable condition and continued follow-up care with the orthopedic and vascular clinics as an outpatient.

Authors

Picture of Nisreen Al Maghraby

Nisreen Al Maghraby

Dr. Nisreen Maghraby, a double-board-certified North American graduate, holds two master’s degrees from McGill University: a Master’s in Educational Psychology and the International Master’s for Health Leadership. She currently serves as an Assistant Professor and Consultant in Emergency Medicine, Trauma, and Disaster Management at IAU in Dammam, KSA. Dr. Maghraby is the Founder and Director of the Simulation and Clinical Skills Center and serves as the Competency-by-Design Lead for postgraduate programs. She also chairs the Emergency Medicine Saudi Board Exam Committees at the SCFHS and is a Senior Educator Advisory Board Member for the ATLS program at the American College of Surgeons. Her academic and research interests include trauma, healthcare facilities disaster preparedness, medical education and simulation, and faculty development.

Picture of Nasser AlJoaib

Nasser AlJoaib

Dr. Nasser AlJoaib is a PGY-1 General Surgery resident at King Fahd Hospital in Al-Khobar, Saudi Arabia, and an incoming Vascular Surgery resident at the University of Toronto. His academic interests include trauma, vascular disease, and vascular trauma. Dr. AlJoaib has an extensive research portfolio, with publications in high-impact journals and presentations at international conferences.

Picture of Faisal AlGhamdi

Faisal AlGhamdi

Faisal AlGhamdi is an Emergency Medicine resident at King Fahad University Hospital in AlKhobar, Saudi Arabia, with a strong interest in research, trauma, and critical care. He has authored several papers in Emergency Medicine published in reputable journals and has participated in both national and international conferences, receiving recognition for his work. Faisal's interest in Emergency Medicine developed during his medical education, where he gained hands-on experience in the field. After completing rotations at various Emergency Departments, he chose to pursue his residency training at King Fahad University Hospital. He plans to further specialize through a fellowship in critical care. In addition to his clinical work, Faisal is actively involved in research and educational activities. His contributions include participation in toxicology competitions and presenting at conferences. Faisal aims to continue advancing his knowledge and skills in Emergency Medicine and critical care, contributing to the field through both clinical practice and research.

Listen to the chapter

References

  1. Moore KL, Dalley AF. Clinically Oriented Anatomy. 8th ed. Wolters Kluwer; 2018.
  2. Drake RL, Vogl W, Mitchell AWM. Gray’s Anatomy for Students. 3rd ed. Elsevier Health Sciences; 2015.Netter, F. H. (2019). Atlas of Human Anatomy. Elsevier Health Sciences.
  3. Jared, M., Wohlgemut., Max, Marsden., Rebecca, Stoner., Erhan, Pisirir., Evangelia, Kyrimi., Gareth, Grier., Michael, Christian., T., Hurst., William, Marsh., Nigel, Tai., Zane, Perkins. “2. Diagnostic accuracy of clinical examination to identify life- and limb-threatening injuries in trauma patients.” Scandinavian Journal of Trauma, Resuscitation and Emergency Medicine, (2023). doi: 10.1186/s13049-023-01083-z
  4. Markel A, Weich Y, Gaitini D. Doppler ultrasound in the diagnosis of venous thrombosis. Angiology. 1995;46(1):65-73. doi:10.1177/000331979504600109
  5. Luís Duarte M, Dos Santos LR, Oliveira ASB, Iared W, Peccin MS. Computed tomography with low-dose radiation versus standard-dose radiation for diagnosing fractures: systematic review and meta-analysis. Sao Paulo Med J. 2021;139(4):388-397. doi:10.1590/1516-3180.2020.0374.R3.1902021
  6. Zubrod CJ, Barrett MF. Magnetic resonance imaging of tendon and ligament injuries. Clinical Techniques in Equine Practice. 2007 Sep 1;6(3):217-29.
  7. Patterson BO, Holt PJ, Cleanthis M, et al. Imaging vascular trauma. Br J Surg. 2012;99(4):494-505. doi:10.1002/bjs.7763
  8. Fu CY, Liao CA, Liao CH, et al. Intra-abdominal injury is easily overlooked in the patients with concomitant unstable hemodynamics and pelvic fractures. Am J Emerg Med. 2014;32(6):553-557. doi:10.1016/j.ajem.2014.02.013
  9. Lachiewicz PF. The role of intravenous acetaminophen in multimodal pain protocols for perioperative orthopedic patients. Orthopedics. 2013;36(2 Suppl):15-19. doi:10.3928/01477447-20130122-52
  10. Rainsford KD. Anti-inflammatory drugs in the 21st century. Subcell Biochem. 2007;42:3-27. doi:10.1007/1-4020-5688-5_1
  11. Rasor J, Harris G. Using opioids for patients with moderate to severe pain. J Am Osteopath Assoc. 2007;107(9 Suppl 5):ES4-ES10.
  12. Acute hip pain-suspected fracture (no date). Available at: https://acsearch.acr.org/docs/3082587/Narrative/ (Accessed: March 8, 2023).
  13. Masiewicz S, Mabrouk A, Johnson DE. Posterior Hip Dislocation. In: StatPearls. Treasure Island (FL): StatPearls Publishing; September 25, 2022.
  14. Vigotsky AD, Lehman GJ, Beardsley C, Contreras B, Chung B, Feser EH. The modified Thomas test is not a valid measure of hip extension unless pelvic tilt is controlled. PeerJ. 2016;4:e2325. Published 2016 Aug 11. doi:10.7717/peerj.2325
  15. Stephenson JW, Davis KW. Imaging of traumatic injuries to the hip. Semin Musculoskelet Radiol. 2013;17(3):306-315. doi:10.1055/s-0033-1348097
  16. Chmelová J, Mrázková D, Dzupa V, Báca V, Grill R, Pleva L. Význam klasického rentgenového snímku pri poranení pánve v dobe moderní CT diagnostiky [The role of plain radiography in pelvic trauma in the era of advanced computed tomography]. Acta Chir Orthop Traumatol Cech. 2006;73(6):394-399.
  17. Barnard ER, Stwalley D, Miller AN. State of the Union: Timeliness to Antibiotics in Open Fractures. J Orthop Trauma. 2023;37(5):e213-e218. doi:10.1097/BOT.0000000000002546
  18. Gage BE, McIlvain NM, Collins CL, Fields SK, Comstock RD. Epidemiology of 6.6 million knee injuries presenting to United States emergency departments from 1999 through 2008. Acad Emerg Med. 2012;19(4):378-385. doi:10.1111/j.1553-2712.2012.01315.x
  19. Sokal PA, Norris R, Maddox TW, Oldershaw RA. The diagnostic accuracy of clinical tests for anterior cruciate ligament tears are comparable but the Lachman test has been previously overestimated: a systematic review and meta-analysis. Knee Surg Sports Traumatol Arthrosc. 2022;30(10):3287-3303. doi:10.1007/s00167-022-06898-4
  20. Pritsch T, Blumberg N, Haim A, Dekel S, Arbel R. The importance of the valgus stress test in the diagnosis of posterolateral instability of the knee. Injury. 2006;37(10):1011-1014. doi:10.1016/j.injury.2006.03.009
  21. Doherty M, Hoskins R. Diagnosing meniscal tears in the emergency department. Emerg Nurse. 2015;23(3):31-36. doi:10.7748/en.23.3.31.e1428
  22. Baryeh K, Getachew F. Patella dislocation: an overview. Br J Hosp Med (Lond). 2021;82(8):1-10. doi:10.12968/hmed.2020.0429
  23. Knutson T, Bothwell J, Durbin R. Evaluation and management of traumatic knee injuries in the emergency department. Emerg Med Clin North Am. 2015;33(2):345-362. doi:10.1016/j.emc.2014.12.007
  24. Stiell IG, Greenberg GH, Wells GA, et al. Derivation of a decision rule for the use of radiography in acute knee injuries. Ann Emerg Med. 1995;26(4):405-413. doi:10.1016/s0196-0644(95)70106-0
  25. Strudwick K, McPhee M, Bell A, Martin-Khan M, Russell T. Review article: Best practice management of common knee injuries in the emergency department (part 3 of the musculoskeletal injuries rapid review series). Emerg Med Australas. 2018;30(3):327-352. doi:10.1111/1742-6723.12870
  26. Kim PH, Leopold SS. In brief: Gustilo-Anderson classification. [corrected] [published correction appears in Clin Orthop Relat Res. 2012 Dec;470(12):3624] [published correction appears in Clin Orthop Relat Res. 2019 Oct;477(10):2388. doi: 10.1097/CORR.0000000000000950]. Clin Orthop Relat Res. 2012;470(11):3270-3274. doi:10.1007/s11999-012-2376-6
  27. Barile A, Bruno F, Arrigoni F, et al. Emergency and Trauma of the Ankle. Semin Musculoskelet Radiol. 2017;21(3):282-289. doi:10.1055/s-0037-1602408
  28. Millen JC, Lindberg D. Maisonneuve fracture. J Emerg Med. 2011;41(1):77-78. doi:10.1016/j.jemermed.2008.08.021
  29. Tohyama H, Yasuda K, Ohkoshi Y, Beynnon BD, Renstrom PA. Anterior drawer test for acute anterior talofibular ligament injuries of the ankle. How much load should be applied during the test?. Am J Sports Med. 2003;31(2):226-232. doi:10.1177/03635465030310021201
  30. Rosen AB, Ko J, Brown CN. Diagnostic accuracy of instrumented and manual talar tilt tests in chronic ankle instability populations. Scand J Med Sci Sports. 2015;25(2):e214-e221. doi:10.1111/sms.12288
  31. Cao S, Wang C, Chen Y, et al. Stress Tests for Deltoid Ligament and Syndesmosis Injury in Patients With Ankle Fracture: A Systemic Review With Meta-Analysis. J Orthop Trauma. 2023;37(11):e441-e446. doi:10.1097/BOT.0000000000002651
  32. Beumer A, Swierstra BA, Mulder PG. Clinical diagnosis of syndesmotic ankle instability: evaluation of stress tests behind the curtains. Acta Orthop Scand. 2002;73(6):667-669. doi:10.1080/000164702321039642
  33. Stiell I. Ottawa ankle rules. Can Fam Physician. 1996;42:478-480.
  34. Strudwick K, McPhee M, Bell A, Martin-Khan M, Russell T. Review article: Best practice management of common ankle and foot injuries in the emergency department (part 2 of the musculoskeletal injuries rapid review series). Emerg Med Australas. 2018;30(2):152-180. doi:10.1111/1742-6723.12904
  35. Salvi AE. Lisfranc injuries: a matter of ligament disruption. J Foot Ankle Surg. 2014;53(5):674-676. doi:10.1053/j.jfas.2014.03.021
  36. Isacson D, Nowinski D. Soft tissue injuries. In: Orthopedic Surgery. Springer; 2020:Chapter 3. doi:10.1007/978-3-030-39383-0_32.
  37. Hovius ERS, Nijhuis THJ. Compartment syndrome in the extremities. In: Reconstructive Plastic Surgery. Wiley-Blackwell; 2015:769-776. doi:10.1002/9781118655412.ch55.
  38. Imerci A, Ozaksar K, Gürbüz Y, Sügün TS, Canbek U, Savran A. Popliteal artery injury associated with blunt trauma to the knee without fracture or dislocation. West J Emerg Med. 2014;15(2):145-148. doi:10.5811/westjem.2013.12.18223.
  39. Ko SH, Bandyk DF. Interpretation and significance of ankle-brachial systolic pressure index. Semin Vasc Surg. 2013;26(2):86-94. doi:10.1053/j.semvascsurg.2014.01.002.
  40. Toivanen JAK. The management of closed tibial shaft fractures. Curr Orthop. 2003;17(3):167-175. doi:10.1016/S0268-0890(03)00047-1.
  41. Bartle D, Keating JF. Femoral and tibial fractures. Surgery (Oxford). 2013;31(9):460-465. doi:10.1016/j.mpsur.2013.06.007.
  42. Robinson PM, O’Meara MJ. The Thomas splint: its origins and use in trauma. J Bone Joint Surg Br. 2009;91(4):540-544. doi:10.1302/0301-620X.91B4.21962.

Reviewed and Edited By

Picture of Jonathan Liow

Jonathan Liow

Jonathan conducts healthcare research in the Emergency Department at Tan Tock Seng Hospital. A graduate of the University at Buffalo with a BA in Psychology and Communication, he initially worked on breast cancer research studies at GIS A*STAR. His research interests focus on integrating AI into healthcare and adopting a multifaceted approach to patient care. In his free time, Jonathan enjoys photography, astronomy, and exploring nature as he seeks to understand our place in the universe. He is also passionate about sports, particularly badminton and football.

Picture of James Kwan

James Kwan

James Kwan is the Vice Chair of the Finance Committee for IFEM and a Senior Consultant in the Department of Emergency Medicine at Tan Tock Seng Hospital in Singapore. He holds academic appointments at the Lee Kong Chian School of Medicine, Nanyang Technological University, and the Yong Loo Lin School of Medicine, National University of Singapore. Before relocating to Singapore in 2016, James served as the Academic Head of Emergency Medicine and Lead in Assessment at Western Sydney University's School of Medicine in Australia. Passionate about medical education, he has spearheaded curriculum development for undergraduate and postgraduate programs at both national and international levels. His educational interests focus on assessment and entrustable professional activities, while his clinical expertise includes disaster medicine and trauma management.

Picture of Arif Alper Cevik, MD, FEMAT, FIFEM

Arif Alper Cevik, MD, FEMAT, FIFEM

Prof Cevik is an Emergency Medicine academician at United Arab Emirates University, interested in international emergency medicine, emergency medicine education, medical education, point of care ultrasound and trauma. He is the founder and director of the International Emergency Medicine Education Project – iem-student.org, chair of the International Federation for Emergency Medicine (IFEM) core curriculum and education committee and board member of the Asian Society for Emergency Medicine and Emirati Board of Emergency Medicine.

Burns and Smoke Inhalation (2024)

by Michaela Banks, Anthony Dikhtyar, Jacquelyne Anyaso, & Ashley Pickering

You have a new patient!

A 26-year-old male presents to the emergency department with burns on his face, arms, hands, and torso. He states that he was burning trash in his front yard without his shirt on when a big explosion occurred. He appears distressed and short of breath. The presence of singed nasal hairs is also noted. Examination reveals multiple partial- and full-thickness burns with blisters and surrounding redness. His vitals are as follows: HR: 130  BP (taken on R calf): 130/80 RR: 30 SpO2: 75%. His weight: 75kg

a-photo-of-a-26-year-old-male-with-burns (the image was produced by using ideogram 2.0)

What do you need to know?

Importance

Burn injuries result from various sources and can range in severity. The mortality rate from thermal burns is directly related to the size of the burn [1]. Large and deep burns can trigger systemic responses, such as shock, which can lead to death. Three key risk factors that increase the likelihood of complications include: age over 60 years, inhalation injuries, and non-superficial burns (partial and full-thickness) covering more than 40% of the total body surface area (TBSA) [2].

Epidemiology

Burn injuries are a significant public health issue, with approximately 450,000 individuals seeking medical attention annually, and about 45,000 requiring hospitalization [2]. Residential fires are the leading cause of burn-related deaths, contributing to nearly 3,500 fatalities per year. Smoking materials, such as cigarettes, are the primary cause of fire-related deaths, while other fatal injuries stem from motor vehicle crashes, electrical contact, or exposure to chemicals. Men constitute 71% of burn patients, with children under five representing 17% [2]. Most burns occur at home (65%) and involve less than 10% total body surface area (67%). Advances in burn care have improved survival rates to 96% [2]. Roughly 86% of all burns are caused by thermal injury. Flame and scald burns are the leading causes of burns in children and adults. Inhalation injury is present in two-thirds of patients with burns greater than 70% of TBSA. 

Pathophysiology

Burn injuries, caused by heat, chemicals, electricity, or radiation, trigger a complex interplay of local and systemic responses. At the cellular level, burn wounds are divided into three distinct zones: coagulation, stasis, and hyperemia. The central zone of coagulation undergoes irreversible cell death due to protein denaturation, necessitating surgical intervention in many cases. Surrounding it, the zone of stasis contains viable but at-risk cells that can either recover with proper care or progress to necrosis. The outer zone of hyperemia typically recovers fully within days due to its inflammatory response and intact blood flow [2-4].

Burns prompt a robust inflammatory response, increasing capillary permeability and causing fluid shifts that lead to edema. Local edema compromises blood flow and cell survival in the zone of stasis, while systemic edema in large burns contributes to hypovolemia, the primary cause of burn shock. Immediate and adequate fluid resuscitation is critical to prevent worsening injury and maintain organ perfusion [2-4].

Specific burn types exhibit unique pathophysiologies. Inhalation injuries from superheated gases or toxic smoke cause airway edema, inflammation, and potentially fatal complications like carbon monoxide poisoning and ARDS [5]. Chemical burns differ by agent, with acids causing coagulation necrosis and alkalis leading to deeper liquefaction necrosis. Electrical burns often involve extensive internal damage along the current’s path, risking cardiac arrhythmias and systemic effects. Radiation burns, though rarer, involve cellular damage through ionizing radiation exposure [2-4].

Systemically, extensive burns induce a hypermetabolic state, immune suppression, and systemic inflammatory responses affecting multiple organs. Cardiovascular effects, such as burn shock, respiratory compromise, and heightened infection risks, are key complications. Patient outcomes hinge on factors like burn depth, TBSA, age, inhalation injury presence, and quality of initial management, underscoring the importance of specialized burn center care.

Burn Depth

Burn depth classification is fundamental to assessing burn injuries, guiding treatment decisions, and predicting outcomes. Accurate determination of burn depth, particularly for partial-thickness burns, remains challenging, even for skilled clinicians. This underscores the need for continued research and advanced technologies to enhance diagnostic precision.

Traditionally, burns are categorized into four classes based on the extent of tissue damage [4]:

  1. Superficial Thickness (First-Degree) Burns: These affect only the epidermis, presenting with redness, pain, and warmth without blistering. Healing occurs within a few days without scarring.
  2. Partial-Thickness (Second-Degree) Burns: These penetrate the dermis and are subdivided into:

    1. Superficial Partial-Thickness Burns: Involving the upper dermis, they are painful, moist, and blistered, typically healing within 2–3 weeks with minimal scarring.

    2. Deep Partial-Thickness Burns: Reaching deeper dermal layers, these burns cause damage to sweat glands and hair follicles. They are less painful due to nerve damage, appear mottled and dry, and may require 3–8 weeks or longer to heal, often resulting in scarring or contractures.

  3. Full-Thickness (Third-Degree) Burns: These burns destroy the entire epidermis and dermis, extending into subcutaneous tissue. They appear white, brown, or charred with a leathery texture and are insensate due to nerve destruction. Healing requires surgical intervention, such as skin grafting, and leaves significant scars.

  4. Fourth-Degree Burns: Extending into muscles, bones, tendons, or ligaments, these burns are characterized by blackened tissue and often result in loss of the affected part.

These classifications provide a framework for clinicians to tailor interventions and anticipate patient needs, particularly in severe or complex burn cases. The illustration below displays the various categories of burn depth [4].

From: [4] Jeschke MG, van Baar ME, Choudhry MA, Chung KK, Gibran NS, Logsetty S. Burn injury. Nat Rev Dis Primers. 2020;6(1):11. Published 2020 Feb 13. doi:10.1038/s41572-020-0145-5

Medical History

Accurately gathering a burn history is critical for evaluating the injury’s severity, identifying risks, and tailoring management. The AMPLET mnemonic is widely recommended for systematic collection of essential information regarding the event and the patient’s medical background. Additionally, specific questions based on the type of burn provide crucial details for precise assessment and treatment [2-4, 6].

Allergies (A):
Identifying drug and environmental allergies is essential to avoid adverse reactions during treatment.

Medications (M):
A detailed list of current medications, including prescription drugs, over-the-counter remedies, herbal supplements, and home treatments, is vital to anticipate potential drug interactions or complications.

Past Medical History (P):
Knowledge of pre-existing conditions, such as diabetes, cardiovascular disease, lung disorders, or bleeding tendencies, helps predict how the patient may respond to burn injuries and resuscitation. Tetanus immunization status should also be reviewed and updated if necessary (see “T”).

Last Meal or Drink (L):
Documenting the patient’s last meal or drink is crucial for surgical planning, as recent food intake may require delays in procedures involving anesthesia.

Events/Environment Relating to Incident (E):
A detailed account of the burn incident helps identify the mechanism of injury, the risk of inhalation injury, and associated trauma. Important elements to document include:

  • Type of burn: Thermal, chemical, electrical, or radiation.
  • Cause of burn: Flame, scald, contact with hot objects, chemicals, or electricity.
  • Incident location: Indoor/outdoor, enclosed space, smoke presence.
  • Duration of exposure: Time spent in contact with the burn source.
    First aid administered: Cooling, cleaning, or dressing of the burn before medical evaluation.
  • Suspicion of abuse or neglect: Look for inconsistencies in the history, patterns of injury, or delays in seeking care. Specific questions include:
    • How did the burn occur?
    • Who was present?
    • How long to extinguish flames?
    • Was the area cooled? With what and for how long?
    • Were explosions, blasts, or chemical spills involved?
    • Was the patient trapped or unconscious?

Tetanus and Childhood Immunizations (T):
Ensuring tetanus immunization is current (within five years) is crucial. In children, assessing overall immunization status helps anticipate potential complications.

Specific Questions Based on Burn Type [2-4,6]

Thermal Burns:

  • How did the burn occur?
  • What was the heat source (e.g., flame, scald, or hot object)?
  • Was clothing involved, and how quickly was it removed?
  • Was a flammable liquid (e.g., gasoline) involved?

Chemical Burns:

  • What was the chemical agent?
  • How did exposure occur, and how long was contact?
  • What decontamination measures were taken?
  • Is a Material Safety Data Sheet (MSDS) available?

Electrical Burns:

  • What type of electricity was involved (high voltage/low voltage, AC/DC)?
  • What was the duration of contact?
  • Was the patient thrown or did they fall?

Physical Examination

The physical examination of a burn patient is a systematic process designed to assess the severity of the burn injury, identify associated injuries or complications, and guide treatment decisions. A comprehensive and thorough examination is critical for determining the need for transfer to a burn center and predicting potential outcomes [2-6]. Make sure to assess for concomitant trauma (especially after a blast injury or fall).

First, perform decontamination if the person has been exposed to a chemical substance. If possible, expose the patient to a warm room. Immediately assess the airway, breathing, and circulation (ABCs), see details below.

Primary Survey

The primary survey prioritizes life-threatening conditions using the ABCDE approach [2,4,6]:

A. Airway

  • Assess for patency: Check for obstruction, swelling, or soot in the mouth and nose. Examine for posterior oropharynx edema and singed facial and nasal hairs carefully.
  • Listen for abnormal breath sounds: Stridor, wheezing, or decreased breath sounds may indicate inhalation injury or airway compromise.
  • Consider early intubation: Severe facial burns, inhalation injury, or altered mental status may necessitate securing the airway. Please do not delay airway procedure if you suspect inhalation injury.

B. Breathing

  • Assess respiratory rate and effort: Look for tachypnea, labored breathing, or cyanosis.
  • Auscultate lung sounds: Wheezing, rales, or rhonchi may suggest inhalation injury or pulmonary complications.
  • Administer high-flow oxygen: Use 100% oxygen via a non-rebreather mask, particularly for moderate to severe burns patients or patients with suspected inhalation injury.

C. Circulation

  • Monitor heart rate and rhythm: Look for tachycardia, bradycardia, or arrhythmias.
  • Measure blood pressure: Hypotension may indicate shock or blood loss.
  • Assess capillary refill and skin color: Delayed refill, pallor, or cyanosis indicates poor perfusion.
  • Establish IV access: Insert two large-bore IVs for moderate to severe burn patients, particularly for burns covering >20% TBSA.
  • Control bleeding: Bleeding suggests additional injuries.

D. Disability

  • Assess level of consciousness: Use the AVPU scale (Alert, Verbal, Pain, Unresponsive) or Glasgow Coma Scale (GCS).
  • Evaluate neurological status: Check pupils, motor strength, and sensation.

E. Exposure and Environmental Control

  • Remove clothing and jewelry: Fully expose the patient to assess burns but prevent further constriction.
  • Identify deformities: Look for fractures or dislocations.
  • Maintain warmth: Use clean, dry sheets and blankets to prevent hypothermia.

Secondary Survey

Once the primary survey stabilizes life-threatening conditions, conduct a detailed evaluation [2,4]:

A. History
Obtain a complete history using the AMPLET mnemonic, covering allergies, medications, past medical history, last meal, events surrounding the burn, and tetanus immunization status (see Medical History above).

B. Head-to-Toe Examination

  • Head and Neck: Assess for burns, singed hair, soot, inhalation injury, corneal damage, and tympanic membrane injury.
  • Chest: Listen to breath sounds, observe chest expansion, and evaluate for circumferential burns that may impair breathing.
  • Abdomen: Inspect for burns, palpate for tenderness, and consider the risk of abdominal compartment syndrome with circumferential burns.
  • Extremities: Look for burns, fractures, diminished pulses, or signs of compartment syndrome. Assess sensation and motor function.
  • Genitalia and Perineum: Inspect for burns and swelling, and assess urinary retention.
  • Back and Buttocks: Examine these areas during log rolling, ensuring full exposure and injury identification.

C. Burn Wound Assessment

  • Burn size: Estimate TBSA using the Rule of Nines (see images below) [7] or the Lund and Browder chart.
  • Burn depth: Classify burns as superficial, partial-thickness (superficial or deep), full-thickness, or fourth-degree. Note that burn depth may evolve over time (see figure about burn depth above).
  • Document wound characteristics: Describe color, texture, moisture, blisters, and eschar.
Rule of Nines (Adults ≥ 14 years of age) - Courtesy of the American Burn Association - From: [7] - https://www.health.state.mn.us/communities/ep/surge/burn/tbsa.pdf
Rule of Nines for Children (Age 1 - 14) - Courtesy of the American Burn Association - From: [7] - https://www.health.state.mn.us/communities/ep/surge/burn/tbsa.pdf
Rule of Nine for Infant (Age < 1 year) - Courtesy of the American Burn Association - From: [7] - https://www.health.state.mn.us/communities/ep/surge/burn/tbsa.pdf

Burns are classified into degrees based on the depth of tissue damage, with each classification displaying distinct pathophysiological features, clinical findings. The following section covers specific clinical information related to burn depth.

Superficial (First-Degree) Burns
Superficial burns involve only the epidermis, the outermost layer of the skin. These burns are characterized by warm, dry, and red areas that blanch with pressure. Blistering is absent, and the skin typically heals within a few days without scarring. Sunburn is a classic example of a superficial burn.

Partial-Thickness (Second-Degree) Burns
Partial-thickness burns extend beyond the epidermis into the dermis and are further divided into superficial and deep categories.

  • Superficial Partial-Thickness Burns: These burns affect the upper dermis and are very painful. Surrounding erythema, moisture, and blistering are common features. These burns blanch when pressed and typically heal with minimal scarring in 2–3 weeks.
  • Deep Partial-Thickness Burns: These penetrate deeper into the dermis, potentially damaging sweat glands and hair follicles. They are less painful due to nerve ending destruction and appear drier, with a mottled red or white surface that does not blanch. Healing takes longer and often results in scarring or contractures. Scalds and flash burns are typical causes of partial-thickness burns.

Full-Thickness and Beyond (Third- and Fourth-Degree) Burns
Full-thickness burns destroy the entire epidermis and dermis, often extending into subcutaneous fat and, in severe cases, deeper structures such as muscle and bone (fourth-degree burns). These burns result in decreased sensation due to nerve destruction. The affected areas appear white, brown, or leathery, with a dry texture, and they do not blanch when pressed. Examples include chemical burns, electrical burns, fully immersed thermal burns, and severe frostbite. Healing requires surgical intervention, such as skin grafting, and significant scarring is inevitable.

Clinical Images of Selected Burn Injuries

Thermal 2nd degree burn in a child
Thermal burn in an adult patient
Thermal injury - 2nd degree burn in a child
Electrical injury/burn - entry wound
Electrical injury/burn - exit wound
Chemical burn
Chemical burn
Corneal Chemical Burn
Thermal burn, Inhalation Injury

Acing Diagnostic Testing

The diagnostic approach to burn patients varies based on the severity of the burn, the suspected complications, and the presence of associated injuries. A systematic evaluation using targeted laboratory tests and imaging helps guide treatment decisions and monitor potential complications.

Patients with Minor Burns

For patients with minor burns and no associated injuries, laboratory testing is generally unnecessary unless other trauma or medical conditions are present.

Patients with Moderate to Severe Burns

Moderate to severe burns necessitate a more comprehensive diagnostic evaluation [2,6]:

  • Complete Blood Count (CBC): Assesses anemia, infection, or thrombocytopenia.
  • Comprehensive Metabolic Panel (CMP): Monitors electrolyte imbalances, fluid shifts, and kidney or liver function.
  • Creatine Kinase (CK): Detects muscle damage.
  • Arterial Blood Gases (ABG) and Carboxyhemoglobin Levels: Essential for suspected inhalation injury to evaluate oxygenation, carbon monoxide poisoning, and acidosis.
  • Blood Cyanide Levels: Performed if cyanide poisoning is suspected, though results may take time. Treatment is often initiated based on clinical suspicion [2].
  • Serum Lactate: Elevated levels indicate tissue hypoperfusion, inadequate resuscitation, or exposure to carbon monoxide or cyanide [6].
  • Coagulation Studies: Identifies coagulopathies, which are common in severe burns.
  • Chest X-Ray (CXR): Evaluates lung damage in inhalation injury and confirms endotracheal tube placement in intubated patients [2,6].

Patients with Electrical Burns

Electrical burns require specialized evaluation due to the unique nature of the injuries:

  • Electrocardiogram (EKG): Necessary for detecting cardiac dysrhythmias, especially in high-voltage injuries. Patients with abnormal EKG findings should be observed until normalization [6].
  • Creatine Kinase (CK): Elevated levels indicate rhabdomyolysis caused by muscle damage [6].
  • Urinalysis: Detects myoglobinuria, a sign of rhabdomyolysis, which can impair kidney function. However, urinalysis has limited specificity [6].

Imaging for Burn Patients

Imaging studies provide critical insights, particularly for inhalation or electrical injuries:

  • Chest X-Ray (CXR): Evaluates lung damage in inhalation injury and confirms endotracheal tube placement in intubated patients. Useful for identifying pulmonary complications, such as pneumothorax, and confirming intubation tube placement [6].
  • Fiberoptic Bronchoscopy: A definitive tool for diagnosing inhalation injury, revealing findings like soot, edema, mucosal blisters, and hemorrhages [5].
  • Chest CT Scan: Offers detailed imaging of lung injuries and is particularly helpful when CXR findings are inconclusive [5].

Risk Stratification

Burn injuries are categorized as minor, moderate, or severe based on several factors that help predict outcomes and guide management. These include the depth of the burn, the percentage of total body surface area (TBSA) affected, and the age of the patient, with burns in individuals under 10 years or over 50 years considered more severe. The presence of associated injuries, such as smoke inhalation or other traumas, also increases the severity. Burns involving high-risk areas—the face, hands, feet, or genitalia—are particularly concerning due to their potential impact on function, aesthetics, and quality of life.

Risk Stratification Criteria

  • Minor
    • Adults: Partial-thickness burns affecting < 15% TBSA
    • Pediatrics: Partial-thickness burns affecting < 10% TBSA
    • No full-thickness burns
    • No involvement of the face, hands, feet, or genitalia
    • No cosmetic impairment
    • Note: Superficial burns are not included in TBSA calculations.
  • Moderate
    • Adults: Partial-thickness burns affecting 15–20% TBSA
    • Pediatrics: Partial-thickness burns affecting 10–15% TBSA
    • Full-thickness burns affecting < 10% TBSA
    • No involvement of the face, hands, feet, or genitalia
    • No cosmetic impairment
  • Severe
    • Adults: Any burn depth affecting > 25% TBSA
    • Pediatrics: Any burn depth affecting > 20% TBSA
    • Full-thickness burns affecting > 10% TBSA
    • Involvement of the face, hands, feet, or genitalia
    • Cosmetic impairment
    • Circumferential burns: Burns extending completely around the chest or a limb:
      • Can cause compartment syndrome or increased pressure in the affected area.
      • This is particularly dangerous in the chest, where it can restrict breathing and may require escharotomy (incisions into the burned tissue) to relieve the pressure.

Referral to a Burn Center
Referral to a specialized burn center is recommended based on the following criteria from the American Burn Association (ABA) [8]:

  • Partial-thickness burns >10% TBSA.
  • Burns involving the face, hands, feet, genitalia, perineum, or major joints.
  • Full-thickness (third-degree) burns in any age group.
  • Electrical or chemical burns.
  • Inhalation injury.
  • Burns in patients with pre-existing conditions that complicate management.
  • Burns with concomitant trauma or special care needs.

Management

Effective management of burn patients begins with prompt stabilization of the airway, breathing, and circulation (ABC). Airway management is critical in cases of full-thickness facial burns, significant soot in the nose or mouth, hoarseness, stridor, respiratory depression, or altered mental status. In such scenarios, establishing a definitive airway through endotracheal intubation is necessary to prevent airway compromise. Breathing should be assessed by monitoring oxygen saturation and providing supplemental oxygen as needed to address hypoxemia, especially in patients with inhalation injuries. Circulation assessment involves evaluating distal pulses, particularly in patients with circumferential burns, which may restrict blood flow and necessitate escharotomy. For burns exceeding 20% TBSA, prompt initiation of intravenous fluid (IVF) resuscitation is essential to maintain hemodynamic stability and prevent burn shock. This systematic approach ensures early intervention to mitigate life-threatening complications. Extensive details on primary and secondary survey was given in the physical examination section.

General Principles in Management of Burns

Burn management follows consistent principles across all mechanisms of injury, prioritizing first aid, pain control, and fluid resuscitation.

First Aid

Immediate first aid involves removing the causative agent and any clothing, jewelry, or objects that may retain heat or constrict circulation. Cooling the affected area with water is effective for small burns but must be used cautiously with larger burns to prevent hypothermia [9].

Analgesia

Burn injuries and wound care are extremely painful, making pain management a critical component of care. Opioid pain medications should be considered to provide adequate relief, particularly for severe burns or during dressing changes [2,6].

Fluid Resuscitation

Fluid replacement is essential for patients with extensive burns to prevent hypovolemia and burn shock. Adults with partial- or full-thickness burns covering >20% TBSA require fluid resuscitation, while this threshold is lower (>10% TBSA) for pediatric and elderly patients [2,6].

Two common formulas guide fluid calculations:

  • Parkland Formula: Volume (mL) = 4 × weight (kg) × % TBSA burned. Half of the total volume is given in the first 8 hours, and the remaining half over the subsequent 16 hours.
  • Modified Brooke Formula: Volume (mL) = 2 × weight (kg) × % TBSA burned for adults, or 3 × weight (kg) × % TBSA burned for children, administered evenly over 24 hours.

Hartmann’s solution or lactated Ringer’s is the preferred replacement fluid. Fluid titration, based on urine output, ensures appropriate volume without overloading:

  • Adults: Maintain urine output at 0.5–1.0 mL/kg/hour.
  • Pediatrics: Maintain urine output at 1.0–1.5 mL/kg/hour.

Fluid resuscitation is a dynamic process requiring hourly re-evaluation to ensure adequacy and prevent complications [2,6]. The fluid rate must be carefully titrated based on the patient’s urinary output and physiological response. Hourly urine output, measured using an indwelling bladder catheter, serves as a reliable indicator of resuscitation adequacy in patients with normal renal function.

  • Adults: Maintain urine output at 0.5 mL/kg/hour (approximately 30–50 mL/hour).
  • Young Children (≤30 kg): Target 1 mL/kg/hour.
  • Pediatric Patients (>30 kg, up to age 17): Maintain output at 0.5 mL/kg/hour.
  • Adults with High-Voltage Electrical Injuries and Myoglobinuria: Ensure a urine output of 75–100 mL/hour until urine clears.

This individualized approach to fluid management helps maintain renal perfusion, ensures effective resuscitation, and minimizes the risk of under- or overhydration.

Thermal Burns

Thermal burns occur when excessive heat is applied to the skin, resulting in tissue destruction. Initially, this process may cause inflammation and initiate the healing response. However, if the heat intensity or duration is sufficient, coagulative necrosis ensues, leading to irreversible cell death and localized tissue loss. The severity and type of burn depend on various factors, including the heat source, duration of exposure, and depth of tissue involvement. 

Thermal 2nd degree burn in a child

The treatment of thermal burns varies based on severity [2, 6, 10].

Minor burns are managed by cleaning the area and applying topical aloe and a barrier dressing. Pain is controlled with oral analgesics, such as NSAIDs or acetaminophen/paracetamol. Patients can be discharged with outpatient follow-up for wound monitoring.

Moderate burns require cleaning with water and debridement of large blisters. Wound care involves the application of a topical antibiotic with a dressing or an antibiotic-impregnated bandage. Pain management may include oral or intravenous analgesia, with narcotics as needed. Fluid resuscitation, either oral or intravenous, is determined by the percentage of total body surface area (%TBSA) affected. Tetanus immunization should be updated if the last dose was over 10 years ago. Consultation with a burn specialist is advised, with possible admission or transfer to a burn center.

Severe burns necessitate cleaning with water, pain management with oral or intravenous analgesia, and application of a dressing without antibiotics or ointments if transfer to a burn center is confirmed. Intravenous fluid resuscitation is essential, along with prompt referral and admission to a burn center. Circumferential full-thickness burns may require escharotomy to prevent complications such as compartment syndrome.

Electrical Burns

Electrical burns can present with a wide range of injuries due to the effects of electrical current and the conversion of electrical energy into thermal injury. High-voltage electrical exposure can also result in blunt trauma caused by the patient being propelled away from the electrical source.

Extent of injuries depends on the voltage type:

  • Low voltage: Commonly seen in children who come into contact with electrical cords or outlets.
  • High voltage: Typically occupational injuries from power lines or utility poles, often leading to deep tissue and organ damage.
  • Lightning: Frequently occurs during outdoor recreational or work activities, especially in rainy seasons.

Deep tissue injury assessment:
Patients presenting with full-thickness burns, painful passive range of motion, and elevated creatine kinase (CK) levels should be presumed to have deep tissue injury.

  • These patients require fluid resuscitation and referral to a burn center when possible.
Electrical injury/burn - entry wound
Electrical injury/burn - exit wound
high voltage electrical injury

Muscle damage results in a breakdown known as rhabdomyolysis, which can lead to renal failure and multi-organ failure if not treated promptly.

electrical injury, rhabdomyolysis

Management [2,3,11]

General Principles

  • Cardiac Monitoring: Patients with suspected electrical burns should undergo continuous cardiac monitoring for 12–24 hours to detect dysrhythmias.
  • Compartment Syndrome Monitoring: Close monitoring is essential for signs of compartment syndrome.
  • Stress Ulcer Prophylaxis: Administer proton pump inhibitors (PPIs) or H2 blockers, especially in patients who are NPO, as electrical burns carry a higher risk of ulcer formation compared to other burns.

Analgesia

  • Severe pain from deep tissue injuries often necessitates IV narcotic analgesia.

Fluids

  • Initiate fluid resuscitation with 1L/hr isotonic fluids in adults.
  • Avoid using the Parkland or Modified Brooke formula, as the %TBSA burned does not accurately reflect the extent of deep tissue injury in electrical burns.
  • Titrate fluid administration to maintain urine output:
    • Adults: 100 mL/hr
    • Children: 1.5–2 mL/kg/hr

Referral
Patients with suspected deep tissue injury should be referred to a burn center when available to ensure comprehensive care.

Chemical Burns

Superficial chemical burns may conceal deeper tissue injuries, making them more challenging to assess than thermal burns. Tissue damage is often underestimated, necessitating frequent reassessment of wounds and clinical status.

Chemical burn

Management [2,12]

  • Fully expose the patient as soon as possible to minimize ongoing tissue damage. Providers should wear personal protective equipment (PPE) before starting decontamination.
  • Copious irrigation is critical and should be performed immediately, continuing for at least 30 minutes or until neutral skin or eye pH is achieved (using serial litmus paper).
  • Exceptions to irrigation: Dry lime, elemental metals, and phenol require alternative treatments instead of water irrigation.
  • Patients with chemical burns should be referred to a burn center for specialized care.

Radiation Burns

  • Cutaneous manifestations of radiation exposure have a slower onset compared to thermal burns [2,13]. Symptoms such as erythema, calor (warmth), and pruritus may appear hours to days after exposure.
  • Waxing and waning of symptoms:
    • A latent phase without visible cutaneous symptoms often follows initial erythema, calor, and pruritus (1–2 days post-exposure).
    • A second wave may occur days to a week later, presenting as erythema, calor, pruritus, desquamation, ulceration, or necrosis.
    • Subsequent waves of symptoms are more common with potent radiation forms (e.g., beta- and gamma-waves), occurring months post-exposure.
  • High radiation doses are associated with systemic effects, including hair loss and acute radiation syndrome (ARS):
    • ARS symptoms include loss of appetite, fatigue, headache, nausea, vomiting, and diarrhea.

Management

  • Anti-inflammatory medications should be administered during the latent phase when cutaneous symptoms are absent.
  • As with chemical burns, all patients with significant radiation burns should be referred to a burn center for evaluation and management.

Inhalation Injuries

General Overview

Inhalational injuries are a leading cause of mortality in burn patients. They are commonly associated with thermal injuries, which cause upper airway edema, and chemical injuries, which result in damage to the lower airway and lung parenchyma.

Assessment

Evaluating for inhalational injuries involves identifying key clinical signs, such as soot in the oropharynx, singed facial hair, or other indications of airway compromise. For chemical burns, determining the substances burned or combusted is critical to understanding the nature of the injury. Diagnostic tools include obtaining arterial blood gas (ABG) analysis and chest X-ray when available to assess respiratory function and lung involvement.

Management [2,5]

Maintaining a Patent Airway

Ensuring a clear airway is critical in burn patients. Prompt airway management is crucial in inhalational injuries. A low threshold for endotracheal intubation is necessary in cases of airway compromise, severe burns, or full-thickness/circumferential burns involving the chest or neck. If progressive airway edema is observed, fiberoptic intubation is preferred, provided it is available. Given the rapid progression of airway edema, early intubation is advised to prevent airway obstruction and ensure adequate ventilation.

Thermal burn, Inhalation Injury

Oxygen Therapy
Patients with suspected inhalation injuries should receive humidified 100% oxygen via a non-rebreather mask immediately. This is particularly important in cases of carbon monoxide poisoning, as high-flow oxygen effectively reduces carboxyhemoglobin levels, improving oxygen delivery to tissues.

Fluid Resuscitation
Inhalation injuries increase fluid requirements beyond those predicted by burn size alone. Fluid resuscitation must be carefully balanced to avoid under-resuscitation, which risks hypoperfusion, and over-resuscitation, which can lead to complications such as pulmonary edema or compartment syndrome.

Medications
Several medications may be employed to address specific symptoms:

  • Bronchodilators: Relieve bronchospasm and improve airway patency.
  • Mucolytics: Help thin and loosen mucus, facilitating its clearance from the airways.
  • Nebulized Heparin: Prevents fibrin cast formation in the airways, reducing the risk of airway obstruction.

Ventilatory Support

Mechanical ventilation may be required for patients with severe respiratory compromise. Ventilator settings must be carefully optimized to prevent ventilator-induced lung injury. Techniques such as low tidal volume ventilation and high-frequency percussive ventilation may offer benefits in managing patients with compromised pulmonary function.

This comprehensive approach ensures effective airway management and respiratory support in burn patients with inhalation injuries.

Special Patient Groups

Pediatric Patients

Thermal Burns

  • Fluid Resuscitation:
    • In addition to using the Parkland formula for fluid replacement, pediatric patients require maintenance intravenous fluids (mIVF) to meet baseline hydration needs.
    • Children under 5 years of age should have glucose added to their mIVF to prevent hypoglycemia.

Electrical Burns

  • The majority of management principles are similar to those for adults.
  • Oral Burns:
    • Oral burns, often caused by chewing on electrical cords, require special attention. Burns at the commissure (corner of the lips) have a high risk of bleeding due to erosion of the labial artery.
    • All significant oral burns should be admitted for observation and plastic surgery consultation to prevent and manage complications.

Pregnant Patients

Electrical Burns

  • For pregnant patients with electrical burns, obstetric consultation is essential to assess maternal and fetal health.
  • Continuous monitoring of fetal heart tones is necessary to evaluate the well-being of the fetus following an electrical injury.

When To Admit This Patient

The American Burn Association released updated guidelines in December 2022 for burn patient referral and management.

Guidelines for Burn Patient Referral - Courtesy of American Burn Association - From: https://ameriburn.org/wp-content/uploads/2023/01/one-page-guidelines-for-burn-patient-referral-16.pdf [14]

According to these guidelines:

  • Moderate to Severe Burns: Patients with moderate to severe burns, as defined by burn depth and total body surface area (TBSA), require hospital admission for comprehensive burn staging and treatment.
  • Minor Burns: Patients with minor burns, such as superficial burns or those involving <10% TBSA superficial partial-thickness burns, can be managed in an outpatient setting.

To prevent secondary infection, patients discharged with minor burns must have access to appropriate topical ointments and dressings. Patients with partial-thickness burns should undergo regular wound checks following discharge to monitor healing and prevent complications.

Revisiting Your Patient

The patient’s burns were classified as moderate to severe, and he was intubated due to the presence of singed nasal hairs and significant respiratory distress. Using the Rule of 9s, the total burn area was calculated to be 31.5% TBSA, including the face (4.5%), the front of both arms and hands bilaterally (4.5% each), and the torso (18%).

Given the depth of the burns, lactated Ringer’s IV resuscitation was initiated, with a target of delivering 4725 mL in the first eight hours, as calculated using the Parkland formula. A Foley catheter was placed, and urine output was titrated to 0.5 mL/kg/hr. The patient also received IV analgesia and was subsequently transferred to a burn center for further management.

Authors

Picture of Michaela Banks

Michaela Banks

Michaela Banks is a current resident at Louisiana State University in New Orleans in Emergency Medicine. She graduated with a degree in Psychology and Global Health from Duke University and went on to obtain her MD and MBA from the University of Virginia. During residency, she has become particularly interested in burns and outcomes, and gave an oral presentation on the “Association Between Compliance with an Organized State Burn Triage Center and Burn Outcomes” at ACEP 22. Michaela also serves on the Emergency Medicine Residents’ Association Board of Directors.

Picture of Anthony Dikhtyar

Anthony Dikhtyar

Dr. Dikhtyar is a graduate of St. George’s University School of Medicine and recently matched into Emergency Medicine at TriStar Skyline Medical Center in Nashville, TN. His professional interests include medical education, medical photography, and global health in the former Soviet Union. His most recent publications can be found in the Visual Journal of Emergency Medicine.

Picture of Jacquelyne Anyaso

Jacquelyne Anyaso

Jackie Anyaso, MD, MBA is a second-generation Nigerian immigrant born and raised in Chicago, Illinois. She attended medical school at the University of Illinois at Chicago and will be completing her emergency medicine training at Harvard-Affiliated Emergency Residency Program. Her ultimate goal is to serve vulnerable populations in efforts to reduce healthcare disparities. Her clinical interests include critical care medicine, global health, and the intersection between medicine and business. Outside of medicine, she enjoys community service, traveling, and spending time with family and friends.

Picture of Ashley Pickering

Ashley Pickering

Before medical school I had a diverse career path, which included biomedical engineering, outdoor education, working as an EMT on a Colorado ski patrol, and critical care nursing. I lived out west for 15 years, mainly in CO, and went to medical school at University of Arizona in Tucson before moving to Baltimore for residency at University of Maryland. Currently I am a Global Emergency Medicine Fellow at University of Colorado. Throughout my training I have found ample opportunities to pursue my interest in building emergency care globally. I have researched the barriers to accessing emergency care in rural Uganda, helped to provide emergency care training in Sierra Leone and Liberia and am currently the Executive Director of Global Emergency Care a non-profit training non-physician clinicians in Uganda. My current focus is on quality of emergency care in LMICs. I am working on an WHO Emergency Care Toolkit implementation project which explores the impact of basic emergency care educational and process improvements on clinical indicators of quality, as well as the experiences patients and staff.

Listen to the chapter

References

  1. Jeschke MG, Mlcak RP, Finnerty CC, et al. Burn size determines the inflammatory and hypermetabolic response. Crit Care. 2007;11(4):R90. doi:10.1186/cc6102
  2. American Burn Association. (2018). Advanced Burn Life Support Course Provider Manual 2018 Update. https://ameriburn.org/wp-content/uploads/2019/08/2018-abls-providermanual.pdf
  3. Schaefer TJ, Szymanski KD. Burn Evaluation And Management. [Updated 2022 Aug 23]. In: StatPearls [Internet]. Treasure Island (FL): StatPearls Publishing; 2023 Jan-. Available from: https://www.ncbi.nlm.nih.gov/books/NBK430741/
  4. Jeschke MG, van Baar ME, Choudhry MA, Chung KK, Gibran NS, Logsetty S. Burn injury. Nat Rev Dis Primers. 2020;6(1):11. Published 2020 Feb 13. doi:10.1038/s41572-020-0145-5
  5. Foncerrada G, Culnan DM, Capek KD, et al. Inhalation Injury in the Burned Patient. Ann Plast Surg. 2018;80(3 Suppl 2):S98-S105. doi:10.1097/SAP.0000000000001377
  6. Emergency Care of Moderate and Severe Thermal Burns in Adults. UpToDate. Feb. 2023. https://www.uptodate.com/contents/emergency-care-of-moderate-and-severe-thermal-burns-in-adults?topicRef=349&source=see_link#H4430737.
  7. Department of Health. Determining Total Body Surface Area. From: https://www.health.state.mn.us/communities/ep/surge/burn/tbsa.pdf Accessed December 1, 2024.
  8. Guidelines for Burn Patient Referral. From: https://ameriburn.org/resources/burnreferral/ Accessed: December 1, 2024.
  9. Burns. WikiEM. 21 Nov. 2021; 4:1-2. https://wikem.org/wiki/Burns#Evaluation.
  10. Treatment of Minor Thermal Burns. UpToDate. Feb. 2023. https://www.uptodate.com/contents/treatment-of-minor-thermal-burns#H20.
  11. Electrical injuries and lightening strikes: Evaluation and management. UpToDate. Mar 2023. https://www.uptodate.com/contents/electrical-injuries-and-lightning-strikes-evaluation-and-management#H3065280448
  12. Topical chemical burns: Initial assessment and management. UpToDate. Mar 2023. https://www.uptodate.com/contents/topical-chemical-burns-initial-assessment-and-management
  13. Cutaneous Radiation Injury (CRI): A Fact Sheet for Clinicians. 4 Apr. 2018. https://www.cdc.gov/nceh/radiation/emergencies/criphysicianfactsheet.htm
  14. Guidelines for Burn Patient Referral. From: https://ameriburn.org/wp-content/uploads/2023/01/one-page-guidelines-for-burn-patient-referral-16.pdf

FOAm and Further Reading

Reviewed and Edited By

Picture of Erin Simon, DO

Erin Simon, DO

Dr. Erin L. Simon is a Professor of Emergency Medicine at Northeast Ohio Medical University. She is Vice Chair of Research for Cleveland Clinic Emergency Services and Medical Director for the Cleveland Clinic Bath emergency department. Dr. Simon serves as a reviewer for multiple academic emergency medicine journals.

Picture of Arif Alper Cevik, MD, FEMAT, FIFEM

Arif Alper Cevik, MD, FEMAT, FIFEM

Prof Cevik is an Emergency Medicine academician at United Arab Emirates University, interested in international emergency medicine, emergency medicine education, medical education, point of care ultrasound and trauma. He is the founder and director of the International Emergency Medicine Education Project – iem-student.org, chair of the International Federation for Emergency Medicine (IFEM) core curriculum and education committee and board member of the Asian Society for Emergency Medicine and Emirati Board of Emergency Medicine.

How to Interpret C-Spine X-ray (2024)

by Maitha Mohammed Alneyadi & Mansoor Masarrat Husain

Introduction

Cervical spine x-ray interpretation is a vital skill in emergency medicine. This is particularly important as cervical spine injuries can leave patients with permanent neurological damage or death. While CT scans have overtaken X-rays as the primary form of cervical spine imaging, X-rays can be handy in rural areas or areas with limited resources. If in doubt, always ask for an expert opinion.

Cervical spine injuries commonly arise from motor vehicle accidents or falls from heights. They more commonly occur in men, and worse outcomes often happen to patients with underlying degenerative changes. Mechanisms of injuries causing fractures include flexion, extension, rotational, or vertical compression—these will be elaborated on further in this chapter. Cervical spine x-rays are somewhat useful if the patient is awake, stable, and has isolated injuries. In addition, they can be ordered in patients with upper airway obstruction symptoms, to look for soft tissue infections, foreign body demonstration, or if there is neck pain with no significant trauma.

Remember, cervical spine x-rays require manipulation of the neck to get clear views. Consider an alternative diagnostic choice like CT (Computed Tomography) or MRI (Magnetic Resonance Imaging) if cervical spine movement is restricted by a cervical collar. X-rays are also not advisable when neurological symptoms are present following trauma, in an uncooperative patient, or when a more accurate radiological modality is easily available.

Plain radiographs that display the lateral projection of the cervical spine, along with an open mouth view, are quite effective at identifying cervical spine fractures. Statistics indicate that the risk of overlooking a significant fracture is less than 1%. Including the anteroposterior (AP) projection raises the sensitivity to almost 100%. All three essential projections mentioned above can be seen in the figure below.

C-spine x-ray - 3 views - Lateral view with normal slight lordosis (A), Odontoid or open mouth view of the atlas and axis (B), Standard anteroposterior or AP view with open mouth, it can also be taken with closed mouth (C).

Before analyzing cervical radiographs, some additional facts need to be presented. Most spinal injuries occur at the junctions of the spine: craniocervical, cervicothoracic, thoracolumbar, and lumbosacral.

The only c-spine radiograph one should be satisfied with is the one showing all seven cervical vertebrae (C1–Th1). The C7–Th1 vertebrae may be obscured in muscular or obese patients, or in patients with spinal cord lesions that affect the muscles that normally depress the shoulders. Such lesions, which leave the trapezius muscle unopposed, occur in the lower cervical region. Shoulders can be depressed by pulling the arms down slowly and steadily or, if the patient is capable, by asking them to depress one shoulder and lift the other hand above their head to achieve the swimmer’s position, which better visualizes the lower vertebrae.

Two examples of a cervical x-ray that is not good enough for the evaluation of the possible injury of the neck.

We will now present a systematic method for interpreting cervical spine x-rays. First, identification—make sure details are correctly matched to the patient by name, date of birth, record number, and the time the scan was done. Use an old x-ray of the patient as a comparison if the study has been done previously.

Interpretation

We utilize the ABCD system to comprehensively interpret cervical spine X-rays.

A: Alignment and adequacy
B: Bones
C: Cartilages
D: Dense soft tissue

Cervical spine X-rays typically include three views: the lateral view (or cross-table view), the odontoid view (or open mouth view), and the anterolateral view. If the lateral view is inadequate, an additional view called the “Swimmer’s view” may be requested to visualize the C7 and T1 vertebrae.

Lateral View

Example of a slightly rotated not ideal lateral projection of the cervical spine in (A) and an x-ray of an ideal lateral projection in (B).

A: Adequacy and Alignment

Lateral view - Adequacy and Alignment
Always assess (AV) anterior vertebral, (PV) posterior vertebral and (SL) spinolaminar lines, they should run smooth, without any disruptions, and should form a slight lordotic shape. All three lines should form a smooth and lordotic curve of the cervical spine. Any disruption in the flow of these lines suggests either a bony or a ligamentous injury.

An adequate image includes the base of the skull to the upper border of T1.

There are four parallel lines to note, from front to back (See image on the left, Courtesy of Dr Hussain Aby Ali). The front line (in purple), referred to as the anterior longitudinal line, runs along the anterior border of the vertebrae.

The second line, or the middle line, referred to as the posterior longitudinal line (in yellow), runs along the posterior border of the vertebrae.

Next, the spinolaminar line (in green) runs between the spinous process and lamina, along the anterior edge of the spinous process.

Lastly, the posterior spinous line (in blue) runs smoothly along the tips of the spinous processes.

The spinal cord lies between the posterior spinous and spinolaminar lines. Disruption of any of these lines indicates a fracture [1].

The image reveals disruption of the normal alignments as indicated with a step-off in C2. This has shifted all the lines forward as seen in a hangman’s fracture. Hurley CM, Baig MN, Callaghan S, Byrne F. Cervical spine hangman fracture secondary to a gelastic seizure. BMJ Case Reports. 2019;12(8):e230733. doi: https://doi.org/10.1136/bcr-2019-230733
Disruption in the shape of the AV line, that indicates injury, and in this case a fracture of the body of C7.

An important exception to the usual guidelines involves pseudo-subluxation of C2 and C3 in the pediatric population, which can lead to confusion. In these cases, it is essential to examine the spino-laminar line from C1 to C3. Be cautious of injury if the base of the C2 spinous process is more than 2 mm away from this line. Additionally, correlate your findings with any relevant soft tissue observations (see below under “D”).

On the lateral view, also assess the predental space, which is the distance between the anterior surface of the odontoid process and the posterior aspect of the anterior ring of C1. This distance should not exceed 3 mm in adults or 5 mm in children (see image below).

B: Bones

Examine the vertebrae for a normal bony outline and bone density. It is important to note any subtle changes in bone density, as these may indicate a compression fracture. Areas with decreased bone density are more vulnerable to fractures and are often seen in patients with conditions such as rheumatoid arthritis, osteoporosis, or metastatic osteolytic lesions. Acute compression fractures, in contrast, typically present as areas of increased bone density.

Integrity of the vertebrae - Image on the left (Courtesy of Hussain Aby Ali), Image on the right (Courtesy of Yvette Mellam, [3] - Gaillard F. Cervical spine fractures. Radiology Reference Article. Radiopaedia.org. Radiopaedia. https://radiopaedia.org/articles/cervical-spine-fractures)

To check the integrity of the vertebrae, we must trace each vertebra individually. If there are any irregularities in the cortex of the bone, there may be a fracture.

As you trace the vertebrae on the right side (the image above), you may note that the sixth vertebra has slipped forward and is not continuous, which is an example of a vertebral fracture.

This is followed by scanning vertebrae C3–C7 in the usual manner, with no specific shadows or rings. The rest of the vertebral spaces must be equal, with a rectangular shape. Follow the spinous processes to look for any fractures [1].

Other examples are given below. See the fracture on 7th vertebral body (image A below), and fracture on spinous process of the 7th vertebrae (image B below).

Watch for a non-disrupted bony outline. Disruption, as in the above examples means fracture of the bone structure. Also search for any hypo- or hyper-dense areas in the bone, as it may be the only indication of the compression fracture. In (A) slight widening of the soft tissue is visible just in front of the fracture, under the white arrow, which may indicate that this is an acute injury.

Let us zoom in into the same image and focus on C1 and C2.

Coffee bean and C1 and C2

Start your day with a coffee—or rather, a coffee bean shadow—when interpreting c-spines. This shadow corresponds to the anterior arch of the atlas found in C1. Bear in mind that the peg might get in your way. With that, make sure the coffee bean shadow is adjacent to the odontoid peg. If not, think of a fracture!

When looking at C2, trace the ring, referred to as Harris’ ring (black color in the image above), which is the lateral mass of the vertebra. Discontinuity of the ring demonstrates a fracture.

C: Cartilage space assessment

n the assessment, examine the disc spaces, facet joint spaces, and interspinous spaces for any misalignments or increased space. Subluxations or facet dislocations can be identified by disruptions in the demarcated boxes, while any interspinous height exceeding 50% of the vertebral body indicates ligament disruption. On a good-quality lateral view x-ray of a healthy person, uniform intervertebral spaces should be evident.

An emergency physician may diagnose subluxations and dislocations of the facet joints by assessing the cartilage space between the vertebral corpora, facet joints, and spinous processes. However, increased interspinous distance by more than 50% suggests a ligamentous injury, and protective muscle spasms may complicate interpretation.

Uniform intervertebral cartilage spaces, also facet joints must be inspected, for any unusual alignment or increased space.

D: Dense soft tissue

Subsequently, we check the prevertebral space (in yellow), with the trachea sitting right in front of it (in red) (see the image below, courtesy of Hussain Aby Ali). Take C4 as your reference point (in purple). As a rule of thumb, the prevertebral space at or above C4 should be less than one-third the width of the vertebral body, while below C4 it should measure less than the width of the adjacent vertebra. In pediatrics, the prevertebral space at C4 is 7 mm, and at C6 it measures 14 mm or less, depending on age. In adults, the prevertebral space at C6 measures 22 mm. Enlarged measurements may indicate a hematoma related to a fracture, although normal measurements do not rule out a fracture [1].

The prevertebral soft tissues can serve as an indicator of acute swelling or hemorrhage resulting from an injury, and in some cases, may be the only indicator of an acute injury visible on an x-ray. The normal width of the prevertebral tissue decreases from C1 to C4 and increases from C4 downward. Normal measurements are less than 7 mm from C1 to C4 (less than half the vertebral body width at this level) and less than 22 mm below C5 (less than the vertebral body width at this level, as shown in Figure 9). The presence of air within the soft tissue could suggest a rupture of the esophagus or trachea.

Retro-pharyngeal soft tissue, narrows down from C1 to C4, and should not exceed more than 7mm (less than third of the vertebral body). Bellow the C4 soft tissue starts widening, but should not exceed 22mm (for easier thinking, should not exceed the width of the body of the vertebrae.

Odontoid – Open Mouth View

A: Adequacy and Alignment

The odontoid x-ray is typically the second standard view obtained in the emergency department. Its primary goal is to visualize the odontoid process of the C2 vertebra and the C1 vertebra. This view can be taken with the patient’s mouth either open or closed.

When examining the odontoid x-ray, two key aspects are assessed: first, the distance between the odontoid process and the lateral masses of the C1 vertebra should be equal. If there is an inequality, it may indicate a slight rotation of the head. Second, considering the previous point, the margins of the C1 and C2 vertebrae should remain aligned.

The distance between the odontoid process and the lateral masses of the C1 should be equal, if not inequality may be due to the slight rotation of the head. (If the patient has the upper central incisor teeth, we can check if the space between those two teeth aligns with the middle of the odontoid process, this might give the slight idea about rotation in case process itself is not broken and misaligned). Even with the slight rotation of the head we can still check alignment by looking at the lateral margins of the C1 and C2, which should remain aligned.

B: Bones

The odontoid view is most helpful for assessing peg fractures and examining the lateral masses and spaces at C1 and C2. Start by drawing a line from the end of the lateral mass (in purple), along the shaft, up around the odontoid peg, and down to the other lateral end (in green), which marks C2. Next, demarcate C1’s lateral masses on each side and look for any irregularities or fractures.

C: Cartilage space assessment

The space between the peg and C1’s lateral masses must be equal (green asterisks), as should the spaces between C1 and C2 lateral masses (blue asterisks). Unequal lateral mass spaces could raise suspicion of subluxation, which may indicate that the transverse ligament holding the peg in place is torn. Alternatively, consider a Jefferson fracture, which will be discussed later in this chapter.

Draw an imaginary line along the lateral edges of C1 and C2, and check for any misalignment or displacement (red circles). It is important to note that when a patient’s cervical spine is rotated, the images may be inaccurate due to artifacts, which could be misconstrued as fractures, as shown in the image below [1].

An inappropriate imaging angle can result in an inconclusive image. In such cases, you may notice unequal spaces between the odontoid and C1 lateral masses, even when no underlying fractures are present. This situation should prompt a discussion with the radiologist or the consideration of further imaging, such as a CT scan or MRI.

Beware of the Mach effect!
The Mach effect is an optical illusion that can occur during imaging interpretation. It creates the appearance of a lower density at specific levels of the odontoid peg, which may falsely mimic an odontoid fracture. This illusion arises from the way edges and contrasts in the image are perceived by the human eye, often giving the impression of a discontinuity or fracture when none is present. It is crucial to recognize this phenomenon to avoid misdiagnosis, especially when interpreting odontoid fractures on radiographs. Careful examination and, if needed, correlation with additional imaging modalities such as CT or MRI can help confirm the true nature of the findings.

[4] - Czarniecki M, Niknejad M. Mach effect - mimicking odontoid fracture. Radiopaediaorg. Published online November 24, 2012. doi: https://doi.org/10.53347/rid-20528

Anteroposterior View

A: Adequacy and Alignment

Images taken in this projection are usually less clear than the two mentioned above. The tips of the spinous processes should lie in a straight line along the midline, and the distances between the spinous processes should also be checked. Anomalies, such as bifid spinous processes, can complicate interpretation. The laryngeal and tracheal shadows should align down the middle, and the alignment of the lateral masses of the vertebrae should also be assessed.

Blue line connects the spinous processes, they should lie mid-line and have an equal amount of space between. Red-line should smoothly connect the lateral masses of the vertebrae. Always check the edges of the picture, in most cases, apexes of the lungs are visible, check for pneumothorax.

An adequate image includes the vertebral bodies of the cervical vertebrae along with the superior border of the thoracic vertebrae. Vertical lines running across and along the spinous processes and vertebral bodies help assess alignment. Three lines are particularly important: the spinous process line (in blue), which runs through the spinous processes of C1 to C7, ensuring vertical alignment, and two lateral lines (in green), which run smoothly along the transverse processes, confirming their normal alignment.

B: Bones

The anteroposterior (AP) view of the cervical spine is one of the standard projections used during imaging. It is taken with the x-ray beam directed from the front (anterior) to the back (posterior) of the neck. While it provides a general overview of the alignment of the vertebrae and highlights features such as the spinous processes and transverse processes, this view may not always clearly demonstrate fractures.

Fractures, especially those involving the odontoid peg, vertebral bodies, or certain types of subtle cortical disruptions, can be challenging to detect due to the overlapping structures in this projection. Additionally, anomalies such as misalignment or crowding of the spinous processes might not be easily discernible. As a result, this view is often supplemented with lateral or oblique views and, in cases of doubt, with advanced imaging techniques like CT or MRI for a more definitive diagnosis.

The AP view remains an important tool for assessing gross abnormalities, vertebral alignment, and pathological conditions, such as tumors or significant bone density changes. However, its limitations in detecting subtle fractures underscore the need for careful correlation with clinical findings and additional imaging.

C: Cartilage space assessment

In an AP cervical spine x-ray, the assessment of cartilage spaces is crucial for evaluating alignment and potential injuries. A key rule to follow is the 50% rule: any increase in the cartilage space by more than 50% compared to adjacent spaces suggests anterior cervical dislocation. This finding is often associated with trauma, such as ligamentous injury or vertebral subluxation, but it is important to note that the 50% rule does not apply in cases of muscle spasm, particularly when the neck is in a flexed position.

To confirm the diagnosis and exclude vertebral slippage, it is essential to examine the lateral view. The lateral view provides additional details regarding the vertebral alignment, anterior displacement, and associated injuries that may not be visible on the AP view. Ensuring that the vertebrae are properly aligned without slippage is vital for accurate assessment and diagnosis.

By correlating findings from both the AP and lateral views, a clearer picture of cervical spine integrity can be obtained, helping to differentiate between conditions caused by trauma and those related to positional factors or muscle spasms.

D: Dense soft tissue

In the AP cervical spine view, it is important to assess for the presence of surgical emphysema or pneumothorax, as these findings can indicate significant underlying trauma.

Surgical Emphysema: Look for evidence of air trapped in the soft tissues of the neck. This appears as dark, radiolucent (black) streaks in areas where soft tissues should normally appear opaque. Surgical emphysema in the cervical region can result from tracheal or esophageal injury, penetrating trauma, or fractures that disrupt the airways. Its presence warrants immediate attention and further investigation to locate the source of the air leakage.

Pneumothorax: Although primarily evaluated using a chest x-ray, a pneumothorax might be visible on an AP c-spine x-ray, especially if significant. This is seen as an absence of lung markings on the affected side, with a radiolucent (black) space outlining the lung. Pneumothorax may occur in association with rib fractures or blunt trauma extending to the thoracic region and can contribute to respiratory distress.

Other Views

Swimmer’s view

When C7 or T1 is not clearly visible on the lateral view due to dense body musculature, obtaining a “Swimmer’s view” can be helpful. This imaging technique specifically focuses on the alignment of C7 and T1 at the cervico-thoracic junction. To achieve this view, patients are instructed to lower the shoulder on the same side as the area being examined [5].

Murphy A, Normal cervical spine radiographs with swimmer's view. Case study, Radiopaedia.org (Accessed on 07 Dec 2024) https://doi.org/10.53347/rID-48418 - https://radiopaedia.org/cases/48418

Flexion and Extension Views

Oblique and flexion/extension views are not recommended in the emergency department setting as they can lead to further neurological injuries caused by manipulation. These views are only useful when interpreted by an experienced physician. Flexion and extension views are often contraindicated due to suspected unstable trauma or are impossible to perform because of spastic musculature following the injury (see Figure below). Additionally, unsupervised or forced flexion or extension in a patient with ligamentous injury can result in significant neurological damage. Therefore, other imaging modalities are necessary when a suspected injury is present.

Straightened normal lordotic curvature of the c-spine, may be due to the muscle spasm as a protective mechanism, what also makes flexion and extension views hard to capture.

Abnormal findings on cervical spine x-rays

C1 (Jefferson) fracture

A C1 fracture, also known as a Jefferson fracture, is best visualized on the odontoid view. This type of fracture typically results from axial loading, such as a heavy blow to the top of the head. The force compresses the cervical spine, leading to fractures in both the anterior and posterior arches of C1. These fractures are considered unstable because the transverse ligament, which stabilizes the relationship between the odontoid peg (dens) and the lateral masses of C1, is often disrupted.

Key imaging findings include widened spaces between the odontoid peg and the lateral masses of C1 (marked by orange asterisks). Additionally, the lateral masses of C1 may appear misaligned with those of C2 (marked by green circles), indicating instability [6]. The widening of these spaces and misalignment reflects the ligamentous injury and mechanical instability associated with this fracture.

Due to its unstable nature, a Jefferson fracture requires prompt recognition and further imaging, such as CT scans, to confirm the diagnosis and assess the extent of injury. Management often involves immobilization or surgical intervention, depending on the severity of the ligament disruption and alignment abnormalities.

C2 fractures

Odontoid peg fracture

To identify a C2 fracture, it is essential to evaluate both the open mouth (odontoid) view and the lateral view, as these complementary perspectives provide critical information about the integrity of the C2 vertebra.

  1. Open Mouth (Odontoid) View:
    This view is particularly useful for assessing the odontoid peg, also known as the dens. A discontinuity of the peg process, as shown in the image above, is a hallmark feature of a C2 fracture. This disruption indicates a break in the odontoid peg, which is often caused by significant trauma. The open mouth view allows for a clear examination of the alignment and spacing between the odontoid peg and the lateral masses of C1, helping to confirm the fracture.

  2. Lateral View:
    The lateral view provides additional details about the alignment and integrity of the C2 vertebra. In cases of a C2 fracture:

    • Alignment Disruption: The normal alignment of the vertebral bodies is disturbed, indicating instability.
    • Harris Ring Discontinuity: The Harris ring, a radiographic marker of the lateral mass of C2, appears interrupted, further confirming the presence of a fracture.
    • Posterior Displacement of the Odontoid Peg: The odontoid peg may be displaced posteriorly, which can compromise the spinal canal and potentially compress the spinal cord.

Types of Odontoid Fractures

The graphical presentation above illustrates the three types of odontoid fractures, as labeled below:

Type I:

  • Location: Fracture at the tip of the dens.
  • Associated Injury: Alar ligament avulsion.
  • Stability: This is considered a stable fracture.

Type II:

  • Location: Fracture at the base of the odontoid process.
  • Stability: This is an unstable fracture. It is the most common type of odontoid fracture and is associated with a high risk of nonunion due to poor blood supply at the fracture site.

Type III:

  • Location: A fracture extending through the body of the axis (C2), curving laterally from one end to the other.
  • Stability: This is also considered an unstable fracture. These fractures may disrupt the lateral masses of C2, further compromising spinal stability.

Recommended Management

  • CT Scan: If any of these fractures are suspected or identified on plain x-rays, a CT scan is recommended for further evaluation to define the fracture line and assess the extent of bony disruption.
  • Immobilization: The cervical spine should be immobilized using a cervical collar (c-collar) to prevent further injury.
  • Consultation: Immediate consultation with neurosurgery is advised, as surgical intervention may be required, especially for unstable fractures (Type II and III).

These fractures, particularly Type II and III, have significant clinical implications due to their instability and proximity to critical neural structures, necessitating prompt diagnosis and intervention.

Odontoid fracture - type 2 (Courtesy of Dejvid Ahmetovic)
Suspected fracture of the odontoid process, but with closed mouth teeth might affect the view.
Same patient, but with open mouth view, and the fracture through the body of C2 is visible, also note misalignment of lateral borders of C1 and C2 and difference in space between odontoid process and lateral masses of C2 on both sides.
Hangman's fracture

A Hangman’s fracture is a bilateral fracture of the pars interarticularis of the C2 vertebra, often resulting in cervical spine instability. This type of fracture is best visualized on a lateral view, which reveals key findings:

Loss of Smooth Anterior Alignment

  • The normal, smooth anterior alignment of the cervical spine is disrupted and replaced by a visible step, indicating displacement.

Cortical Discontinuity

  • The fracture causes a break in the cortical bone, further demonstrating structural instability of the vertebra.
Hangman's fracture
Hangman's fracture

Mechanism of Injury

  • Hyperextension Trauma
    • This fracture is commonly caused by hyperextension injuries, such as those sustained in motor vehicle accidents.
    • It is also seen in diving accidents, where a diver’s head strikes the pool floor upon impact.

Clinical Significance

  • Hangman’s fracture is classified as unstable, as it compromises the integrity of the C2 vertebra and its supporting structures, potentially endangering the spinal cord.

Management

  • Immediate immobilization of the cervical spine with a cervical collar is essential. Advanced imaging (CT or MRI) is recommended to further evaluate the extent of the injury and rule out associated soft tissue or ligamentous damage.
  • Consultation with a neurosurgeon is critical for determining the need for surgical stabilization.

Importance of Recognizing C2 Fractures

C2 fractures, such as odontoid fractures or hangman’s fractures, are critical injuries due to their proximity to the spinal cord and brainstem. Prompt recognition using the open mouth and lateral views is vital to avoid neurological complications. Advanced imaging techniques, such as CT or MRI, are often required for further evaluation and to guide management strategies, which may include immobilization or surgical intervention.

Extension Teardrop Fracture

An extension teardrop fracture is a specific type of cervical spine injury in which a portion of the antero-inferior corner of the vertebra is fractured, resembling a teardrop shape. This injury is most commonly observed at C3 and is highly significant due to its association with instability and potential neurological compromise.

Fracture Appearance

  • The fracture is located at the antero-inferior corner of the vertebral body, creating a teardrop-shaped fragment.
Extension Teardrop Fracture - AlJahdali S, Extension teardrop fracture. Case study, Radiopaedia.org (Accessed on 07 Dec 2024) https://doi.org/10.53347/rID-76901 - https://radiopaedia.org/cases/76901

Mechanism of Injury

  • Caused by sudden hyperextension of the neck, which disrupts the anterior longitudinal ligament.
  • Often occurs in activities like diving, particularly when the diver strikes their head against a hard surface such as the pool floor.

Associated Injuries

  • This type of fracture is frequently associated with central cord syndrome, a neurological injury caused by compression of the spinal cord, leading to weakness more pronounced in the upper limbs than the lower limbs.

Management

  • Immediate Stabilization
    • Apply a cervical collar (C-collar) to immobilize the spine and prevent further injury.
  • Imaging
    • A CT scan is the imaging modality of choice to confirm the diagnosis, evaluate the extent of the fracture, and assess for additional injuries or spinal canal compromise.
    • Consultation
      • Immediate consultation with a neurosurgeon is essential for determining the best treatment approach. Depending on the severity, surgical intervention may be necessary.

Flexion Teardrop Fracture

A flexion teardrop fracture is a severe and unstable cervical spine injury resulting from high-energy flexion trauma, frequently occurring at the C5/C6 level. This type of fracture is significant due to its association with spinal instability and neurological damage.

Radiographic Findings (Lateral View):

  • The three longitudinal lines (anterior, posterior, and spinolaminar lines) are disrupted, indicating misalignment and instability.
  • A teardrop-shaped fragment is seen at the antero-inferior corner of the vertebral body, representing the avulsed piece of bone.
[7] Flexion Teardrop Fracture - El-Feky, Mostafa & Munir, Muhammad. (2020). Flexion teardrop fracture. 10.53347/rID-78890.

Mechanism of Injury

  • Caused by hyperflexion of the neck, which exerts excessive force on the cervical spine.
  • This leads to a disruption of the posterior longitudinal ligament, further contributing to instability.

Neurological Association

  • The injury often results in anterior cervical cord syndrome, characterized by loss of motor function and pain/temperature sensation below the level of injury, with preserved proprioception and vibration senses.

Management

  • Immediate Stabilization
    • Apply a cervical collar (C-collar) to immobilize the cervical spine and prevent further injury.
  • Advanced Imaging
    • A CT scan is the preferred imaging modality to confirm the diagnosis, evaluate the extent of the fracture, and identify associated injuries such as spinal canal compromise or ligamentous disruption.
    • MRI may be indicated to assess soft tissue and spinal cord involvement.
  • Consultation
    • Urgent consultation with a neurosurgeon is essential due to the unstable nature of this fracture. Surgical stabilization is often required to restore spinal alignment and prevent further neurological deterioration.

Clinical Importance

The flexion teardrop fracture is considered one of the most unstable cervical spine injuries. Prompt recognition, immobilization, and appropriate surgical management are critical to improving patient outcomes and minimizing long-term neurological deficits.

Clay Shoveler's Fracture

A Clay Shoveler’s fracture is a stable fracture that involves an avulsion of the spinous process, typically occurring in the lower cervical or upper thoracic spine (most commonly at C6, C7, or T1).

Clinical Presentation

  • Patients present with localized pain and tenderness over the affected area.
  • The pain is often exacerbated by movement or palpation of the spine.

Stability

  • This is considered a stable fracture as it does not involve the vertebral body, spinal canal, or neurological structures. However, the injury can still cause significant discomfort and impair mobility.
Clay Shoveler's Fracture The spinous process of C6 is displaced from the vertebra.- Radswiki T, Botz B, Baba Y, et al. Clay-shoveler fracture. Reference article, Radiopaedia.org (Accessed on 07 Dec 2024) https://doi.org/10.53347/rID-13207 - https://radiopaedia.org/articles/13207
Clay Shoveler's Fracture (Courtesy of Dejvid Ahmetovic)

Examination and Management

  • Neurological Assessment
    • A neurological examination should always be performed to rule out any associated injuries or deficits, even though this fracture typically does not affect the spinal cord or nerves.
  • Immobilization
    • The patient should be placed in a cervical collar (c-collar) to immobilize the spine and alleviate pain during the acute phase of the injury.
  • Imaging
    • A lateral cervical x-ray is often sufficient to diagnose the fracture, but a CT scan can provide additional details if needed.
  • Treatment
    • Since this is a stable fracture, management is typically conservative, including pain control, immobilization, and physical therapy as needed.

Clay Shoveler’s fractures are generally associated with good outcomes, and patients can recover fully with appropriate care and immobilization.

Retropharyngeal abscess

Patients with a retropharyngeal abscess often present with:

  • Sore throat and fever.
  • Torticollis: The head is tilted to one side due to neck stiffness and discomfort.
  • Dysphagia: Difficulty swallowing.
  • Respiratory Distress: Severe cases may manifest with stridor, drooling, or increased breathing effort with retractions, indicating a compromised airway.

Management

  • Immediate Interventions
    • Patients in respiratory distress should be closely monitored as the airway may become obstructed, necessitating emergency airway management, including the potential need for a surgical airway (e.g., tracheostomy).
  • Specialist Consultation
    • A prompt otolaryngology consult is warranted for evaluation, incision and drainage (I&D) of the abscess, and initiation of intravenous antibiotics.
  1.  

Radiographic Assessment

  • Measuring the Retropharyngeal Space
    • The retropharyngeal space is evaluated using lateral cervical spine x-rays.
    • Between C2 and C4, the vertebral bodies can be divided into thirds. The retropharyngeal space should not exceed one-third the width of the corresponding vertebral body.
    • At C4 and below, the vertebral bodies should be divided in half, with the prevertebral space width being approximately equal to the anterior half of the vertebral body [8].
  • Signs of Retropharyngeal Abscess
    • Widening of the retropharyngeal space beyond normal parameters is highly suggestive of an abscess.
    • Additional findings may include air-fluid levels, soft tissue swelling, or displacement of adjacent structures.

Epiglottitis

Epiglottitis is a rapidly progressive and potentially life-threatening disease that primarily affects the upper airway. Patients often present with:

  • Fever and sore throat as initial symptoms.
  • Drooling and difficulty swallowing (dysphagia).
  • Inspiratory stridor, indicating partial airway obstruction.

These symptoms suggest an urgent need for airway evaluation and management.

  1.  

Lateral Neck X-ray

  • The hallmark finding is the “thumb sign”, which represents the swollen epiglottis.
  • Swelling of the epiglottis and aryepiglottic folds is characteristic of this condition.
  • The epiglottis appears enlarged and rounded, resembling the shape of a thumb.

Importance of Early Recognition

  • Epiglottitis can rapidly progress to complete airway obstruction, particularly in children.
  • It is critical to recognize these findings on a lateral neck x-ray and act promptly to secure the airway.

Management

Patients showing signs of airway obstruction require immediate attention, with priority given to securing the airway. In severe cases, this may involve intubation, preferably using fiberoptic intubation in a sitting position, or tracheostomy if necessary. This procedure should be performed collaboratively with ENT surgeons and anesthesia professionals in a controlled environment.

As a temporary measure, nebulized racemic epinephrine can be administered to reduce airway swelling, and broad-spectrum antibiotics should be started promptly to treat the underlying infection. Supportive care, such as humidified oxygen, may also be beneficial. Additionally, a nasopharyngoscopy should be performed to directly visualize the epiglottis and assess the extent of swelling.

Laryngotracheobronchitis (Croup)

Laryngotracheobronchitis, commonly referred to as croup, presents with characteristic symptoms including:

  • Barking cough, often likened to a seal’s bark.
  • Inspiratory stridor, indicating upper airway obstruction.
  • Drooling or dysphagia, in some cases.
  • Signs of increased work of breathing, such as retractions and nasal flaring.

These symptoms are typically caused by inflammation and narrowing of the subglottic airway, often following a viral infection.

Radiographic Findings

  • An anteroposterior (AP) neck x-ray may reveal the steeple sign, which represents narrowing of the subglottic trachea [10].
  • The steeple sign is considered pathognomonic for croup, though it is also occasionally observed in bacterial tracheitis.
  • A neck x-ray is not required for diagnosing croup but may be helpful to confirm the diagnosis when the patient is stable and cooperative [11].
[10] - Gaillard F, Kearns C, Murphy A, et al. Croup. Reference article, Radiopaedia.org (Accessed on 07 Dec 2024) https://doi.org/10.53347/rID-1185 - https://radiopaedia.org/articles/1185

While croup is usually a clinical diagnosis, imaging may be considered in atypical presentations or to rule out other conditions like epiglottitis or retropharyngeal abscess. Prompt recognition of croup and appropriate management can prevent complications associated with airway obstruction.

Clinical Decision Rule

There are two widely used scoring systems for neck injuries, primarily for diagnostic purposes: the National Emergency X-Radiography Utilization Study (NEXUS) criteria and the Canadian C-spine rules (CCR). Both have high sensitivity (89% and 98%, respectively) but low specificity (39% and 16%, respectively) [12]. Neither tool is used for patients over 65 years of age.

The NEXUS criteria can be easily remembered using the mnemonic NSAID:

  • N: Neurological deficit
  • S: Spine tenderness, midline
  • A: Altered mental state
  • I: Intoxicated
  • D: Distracting injury

A positive finding in any of these categories requires imaging.

The Canadian C-spine rule, on the other hand, categorizes patients into two groups based on severity: high risk and low risk. It uses a stepwise, question-based approach. Patients who are 65 years or older, those with a high-risk mechanism of injury, or those presenting with neurological symptoms always require imaging.

Refer to the diagram for a simplified explanation.

Specific Patient Groups

Pediatrics

Younger patients have anatomical differences compared to adults, including a larger head, incomplete ossification of the vertebrae, and firm attachment of the ligaments to the spine, which predispose them to injuries. Poor balance and a flexible spine further increase the risk of injury. As children reach the age of 8, their balance improves, and the injury rates decrease.

Nevertheless, pediatric patients can sustain spinal cord syndromes similar to those in adults, which may cause lifelong disabilities. Examples include central cord syndrome, anterior cord syndrome, posterior cord syndrome, Brown-Séquard syndrome, and spinal shock. The decision to perform imaging and the modality chosen are based on criteria similar to those used for adults.

In pediatric trauma patients, the ABCDE trauma evaluation must be followed, as with adults. An important entity to consider is SCIWoRA (Spinal Cord Injury Without Radiographic Abnormality), which is defined specifically for children under 8 years of age. This condition occurs when hyperextension forces injure the neck, leading to neurological deficits without abnormalities detected on x-rays or CT scans. MRI is required to assess the severity and prognosis. Favorable MRI findings include small hematomas and edema, whereas large hematomas or spinal cord transections are considered unfavorable [13].

Geriatrics

Motor vehicle accidents and falls from standing or sitting positions remain the two most common causes of cervical spine injuries in geriatric patients [14]. Due to anatomical degenerative changes and low bone density, even low-energy mechanisms can result in high-impact injuries. CT scanning is recommended for evaluating suspected cervical spine injuries in geriatric patients, who should always be considered trauma patients.

Pregnant Patients

Pregnant individuals involved in trauma require standard trauma protocols for evaluation and treatment, including CT imaging. Although CT imaging exposes both the mother and fetus to radiation, this exposure is not associated with an increased risk of fetal anomalies. However, the use of CT imaging should be carefully considered, with discussions involving the patient or their family, the radiologist, and a senior physician [15].

Authors

Picture of Maitha Mohammed Alneyadi

Maitha Mohammed Alneyadi

Emergency Medicine Department, Tawam Hospital, Al Ain, United Arab Emirates

Picture of Mansoor Masarrat Husain

Mansoor Masarrat Husain

Emergency Medicine Department, Tawam Hospital, Al Ain, United Arab Emirates

Listen to the chapter

References

  1. Raby N, Berman L, Morley S, Gerald De Lacey. Accident & Emergency Radiology: A Survival Guide. Saunders; 2015, P. 171-198
  2. Hurley CM, Baig MN, Callaghan S, Byrne F. Cervical spine hangman fracture secondary to a
    gelastic seizure. BMJ Case Reports. 2019;12(8):e230733. doi: https://doi.org/10.1136/bcr-2019-230733
  3. Gaillard F. Cervical spine fractures. Radiology Reference Article. Radiopaedia.org. Radiopaedia. https://radiopaedia.org/articles/cervical-spine-fractures
  4. Czarniecki M, Niknejad M. Mach effect – mimicking odontoid fracture. Radiopaediaorg. Published online November 24, 2012. doi: https://doi.org/10.53347/rid-20528
  5. Murphy A. Cervical spine (swimmer’s lateral view). Radiopaediaorg. Published online October 7, 2016. doi: https://doi.org/10.53347/rid-48437
  6. Erskine J Holmes, Misra RR. A-Z of Emergency Radiology. Cambridge University Press; 2006, P. 23-31
  7. Harvey H. Flexion teardrop fracture. Radiology Reference Article. Radiopaedia.org. Radiopaedia. https://radiopaedia.org/articles/flexion-teardrop-fracture-1?lang=us
  8. Sheikh Y, Bickle I. Retropharyngeal abscess. Published online July 13, 2014. doi:https://doi.org/10.53347/rid-30018
  9. Sutton AE, Guerra AM, Waseem M. Epiglottitis. In: StatPearls. Treasure Island (FL): StatPearls Publishing; October 5, 2024.
  10. Murphy A, Gaillard F. Croup. Radiopaediaorg. Published online May 2, 2008. doi: https://doi.org/10.53347/rid-1185
  11. Gaillard F. Steeple sign (trachea). Radiology Reference Article. Radiopaedia.org. Radiopaedia. https://radiopaedia.org/articles/steeple-sign-trachea?lang=us
  12. Vazirizadeh-Mahabadi M, Yarahmadi M. Canadian C-spine Rule versus NEXUS in Screening of Clinically Important Traumatic Cervical Spine Injuries; a systematic review and meta-analysis. Arch Acad Emerg Med. 2023;11(1):e5. Published 2023 Jan 1. doi:10.22037/aaem.v11i1.1833
  13. Szwedowski D, Walecki J. Spinal Cord Injury without Radiographic Abnormality (SCIWORA) – Clinical and Radiological Aspects. Pol J Radiol. 2014;79:461-464. Published 2014 Dec 8. doi:10.12659/PJR.890944
  14. Lomoschitz FM, Blackmore CC, Mirza SK, Mann FA. Cervical spine injuries in patients 65 years old and older: epidemiologic analysis regarding the effects of age and injury mechanism on distribution, type, and stability of injuries. AJR Am J Roentgenol. 2002;178(3):573-577. doi:10.2214/ajr.178.3.1780573
  15. Irving T, Menon R, Ciantar E. Trauma during pregnancy. BJA Educ. 2021;21(1):10-19. doi:10.1016/j.bjae.2020.08.005

FOAM and Further Reading

Reviewed and Edited By

Picture of Arif Alper Cevik, MD, FEMAT, FIFEM

Arif Alper Cevik, MD, FEMAT, FIFEM

Prof Cevik is an Emergency Medicine academician at United Arab Emirates University, interested in international emergency medicine, emergency medicine education, medical education, point of care ultrasound and trauma. He is the founder and director of the International Emergency Medicine Education Project – iem-student.org, chair of the International Federation for Emergency Medicine (IFEM) core curriculum and education committee and board member of the Asian Society for Emergency Medicine and Emirati Board of Emergency Medicine.

Basics of Bleeding Control (2024)

by Tasnim Ahmed & Abdulla Alhmoudi

Introduction

The primary objective in the resuscitation of traumatic hemorrhage is to achieve effective hemostasis and maintain hemodynamic stability. The severity of bleeding depends on the depth of the wound and the type of injured vessel. The approach to bleeding control should be tailored to the type and size of the bleeding vessel and the specific anatomical regions involved. Delayed or ineffective haemorrhage management can complicate the healing process and, in severe cases, lead to fatality. Extremity haemorrhage has historically contributed significantly to high mortality rates from casualties during wars [1]. Therefore, the prompt implementation of appropriate haemostatic techniques is a crucial aspect of efficient trauma management. This critical task is typically initiated by the prehospital team and followed by more advanced, invasive techniques provided by the trauma team in a controlled hospital setting

Types of Wounds

Wound is an impairment to the structural integrity of biological tissues, including the skin, mucous membranes, and organ tissues. This disruption in tissue integrity may arise from a diverse range of causes, including traumatic injuries, pathological processes, or surgical interventions. Metric parameters such as size (length), depth, shape, and whether they are open or closed are used to describe wounds.

The subsequent descriptors represent the terminology utilized for the classification of wounds:

Contusions

Contusions result from perpendicular blunt force to the skin, usually through a layer of clothes. Rupture of subcutaneous capillaries can occur, resulting in the formation of a hematoma (Figure 1). The recommended management for this type of wound consists of analgesics and following the “RICE” protocol (Rest, Ice, Compression, and Elevation) [2].

Figure 1 - Contusion

Abrasion

Abrasion is the scraping or scratching of the surface layers of skin (epidermis) when subjected to oblique forces (Figure 2). Proper wound care involves cleansing the wound, applying a sterile bandage, administering analgesics, ensuring tetanus protection, and implementing the RICE protocol [2].

Figure 2 - Abrasion

Incision

Incision is defined as a cut that features straight edges along the margins of the wound. It can be caused by sharp objects like scalpels, knives, sharp metal pieces, or glass (Figure 3). Tissue loss is uncommon, and the wound margins can be easily aligned for closure with medical glue or sutures [1,2,3].

Figure 3 - Incision

Lacerations

Characterized irregular or jagged edges, appearing torn rather than neat incisions [1,3]. They can have an irregular or linear direction and may branch out (Figure 4). Objects with broken or serrated edges or blunt impact on tissue overlying bone typically cause lacerations. Treatment approaches for lacerations are similar to those for incision wounds. However, the appropriate subspecialty should manage deep, complex lacerations or those involving sensitive areas like the face, joints, or tendons.

Figure 4 - Laceration

Avulsion

Avulsion involves a full-thickness laceration-type wound, which usually creates a flap of tissue (Figure 5) [1,3]. Mechanical accidents involving fingers (degloving injuries) can cause avulsions. More severe cases may include exposure of internal organs. Avulsions are challenging to repair and should never be considered minor injuries.

Figure 5 - Avulsion

Amputation

Amputations differ from avulsions in that they involve the complete loss of a limb, whereas avulsions result in the loss of just a flap of skin (Figure 6). It can occur at any point along an extremity and is usually accompanied by significant arterial bleeding. Despite the seriousness of this injury, a properly cooled and transported amputated limb may sometimes be surgically reattached in a hospital setting.

Figure 6 - Amputation

Puncture and Penetrating Wounds

Puncture and penetrating wounds result from the penetration of a sharp object into the tissue without lateral movement from the point of entry (Figure 7). Puncture wounds can be deceptive, as they may appear small on the surface but extend deeply, potentially damaging the neurovascular structure or internal organs and causing significant internal bleeding or secondary injuries. 

Figure 7 - Puncture wound with soft tissue infection

Stab wounds from knives or sharp objects, as well as bullet wounds, are examples of penetrating injuries [1,2,3]. Occasionally, the penetrating object may remain logged to the injury and should never be removed without careful assessment by the trauma team, as it might act as mechanical hemostatic and result in further bleeding once removed. 

Site of Injury

Injuries can also be classified into three types, depending on the injured site of the body; each entails a different approach to management. Extremity injuries refer to damage inflicted on the blood vessels of the arms or legs. Junctional injuries, on the other hand, involve vascular damage occurring at the junction where the extremities meet the torso, such as the hip, axilla, or base of the neck. Torso injuries often involve non-compressible truncal hemorrhage that occurs anywhere on the torso and involves large blood vessels.

Vascular Injury

Injury to any blood vessel type can result in external bleeding. The specific type of vascular injury can be identified based on the characteristics of bleeding observed [1,2,4].

The following are the distinct types of vascular injuries and their corresponding patterns of bleeding:

Arterial Bleeding

Arterial bleeding typically occurs as a consequence of deep penetrating injuries or amputations. It is distinguished by the forceful ejection of bright red blood from the wound synchronized with each heartbeat [2]. Complete laceration of the artery may trigger spontaneous constriction, which helps to control bleeding. However, if only the artery wall is damaged without complete dissection, it can lead to persistent bleeding.

Indicators of arterial injury are classified into hard signs and soft signs [2]. Identifying hard signs indicates an immediate need for arterial exploration and surgical intervention. To aid in the recollection of these hard signs, the mnemonic “The Broken PIPE” can be employed (Box 1). Conversely, soft signs indicate the necessity for additional investigations such as ankle-brachial index measurement, Duplex Doppler ultrasound, or CT angiography, as determined by clinical assessment. The soft signs can be represented by the mnemonic “NON-Deadly HemorrHage” (Box 2).

Venous Bleeding

Venous Bleeding is characterized by a slower flow of dark red blood out of the wound [2]. However, caution is still recommended in venous bleeding, as it can contribute to significant and rapid bleeding if left untreated [4].

Capillary Bleeding

Capillary Bleeding usually results from damage to subcutaneous capillaries. It is characterized by slow, intermittent bleeding in the form of dots or small oozing [2,4].

Indications of Bleeding Control Techniques

Achieving hemodynamic stability necessitates the effective control of all life- or limb-threatening bleeding. While in most cases of traumatic and non-traumatic resuscitation, emphasis is placed on managing the airway and ensuring proper breathing, in situations of exsanguinating bleeding, prioritizing massive hemorrhage control surpasses the immediate focus on airway and breathing management [1]. The choice of hemostatic technique should be based on the depth and specific location of the injury, as outlined in detail in the “Bleeding Control Techniques” section below.

Contraindications of Bleeding Control Techniques

There are no absolute contraindications to any specific hemostatic method [1]. However, bleeding injuries should not distract the physician from managing concurrent immediate life-threatening conditions. Additionally, immediate wound closure is not recommended in wounds older than 8 hours. Instead, these types of wounds should be cleaned thoroughly, covered with sterile dressing, and closed after 3-5 days if there are no signs of infection. This is referred to as “delayed primary closure” [2,3].

Preparation

Similar to all medical procedures, thorough preparation is essential to ensure efficient hemostasis. This preparation encompasses the healthcare team, equipment, medications, the patient, and the wound.

Team Preparation

The healthcare providers involved in the procedure should possess comprehensive knowledge of indications, contraindications, techniques, and potential complications. The team should wear appropriate personal protective equipment, including face masks, face shields, surgical gowns, gloves, and shoe covers as necessary [3]. This protective gear is crucial to safeguard against blood splashes and potential contact with body fluids, particularly in trauma settings where the patient’s health status may be unknown.

Equipment Preparation

The equipment and medications used for hemostasis must be meticulously prepared and checked for the expiry date and functionality. The required equipment is listed under the corresponding techniques in the “Bleeding Control Techniques” section below.

Patient Preparation

A detailed explanation of the procedure should be provided to the patient, and informed consent should be obtained if applicable. Additionally, securing intravenous access and collecting a blood sample for type and cross-matching and coagulation profile are imperative. Administering analgesics and local anesthetics before procedural maneuvers helps to effectively minimize patient discomfort and disruptive movements.

Wound Preparation

A thorough assessment of the wound should be conducted. Distal movement and neurovascular function should be assessed prior to any manipulation. Contaminated wounds require proper irrigation to remove foreign bodies, followed by sterilization of the surrounding skin using antiseptic solution such as povidone iodine or chlorhexidine. However, wound preparation should not delay definitive hemostatic measures [1,3]. 

Bleeding Control Techniques

Direct Pressure

The initial step in controlling bleeding involves applying direct pressure to the bleeding wound. This facilitates the formation of a platelet plug and the initiation of the physiologic coagulation cascade, which is typically achievable within 10 to 15 minutes of proper pressure application [1]. 

Equipment

  • Sterile gauze pad size 4×4
  • Compression bandage
  • Splint\brace

Technique

Ensuring the proper replacement of skin flaps is essential, followed by placing multiple 4×4 sterile gauzes, ideally low adherent type, with equal pressure applied. The wound can be wrapped with a compression bandage if it is in the head or extremities. Following the application of a compression bandage to the extremities, distal mobility, sensation, and perfusion should be checked. Limbs should be placed in a brace to minimize movement and keep it elevated. In body junctions, the wound can alternatively be packed with gauze or hemostatic agents along with topical pressure application [1,2,4].

Precautions

It is important to avoid removing soaked gauze, as this can function as a foreign clot; instead, a new gauze should be applied on top of the existing ones [4]. Compression bandages should be avoided in thoracic wounds, as they can constrict breathing.

Pressure on Arteries

When the source of bleeding cannot be identified, applying proximal pressure can help control the bleeding by reducing blood flow to the injured artery [1].  This is only feasible with extremity wounds and should not be applied to the carotid artery, as this can precipitate ischemic brain insult or vagal stimulation, resulting in bradycardia [4].

Precautions

The time of application is limited to 10 minutes due to the risk of tissue necrosis distal to the pressure point.

Tourniquet

The indication to use tourniquets is severe extremity bleeding that is not controlled by direct pressure application. The concept is constricting arterial flow to the injured area. It is an extremely painful procedure, and proper analgesia should be ensured before applying a tourniquet if time allows.

Equipment

  • Proper size tourniquet
  • Alternative: Blood pressure cuff

Technique

Remove any clothing obstructing the tourniquet application site, ensuring it is directly applied to the skin and remains visible. Position the tourniquet approximately 2-3 inches above the wound, avoiding joints (Figure 8). Tighten the tourniquet until the bleeding stops and the pulse distal to the tourniquet is no longer palpable. Note the time of placement on the tourniquet tag or consider using an indelible marker to write it on patient’s skin. [4,5].

Figure 8 - Tourniquet application

If bleeding is not controlled and the distal pulse is still present after applying the first tourniquet, apply a second one just above its location [4]. Increasing the width of the second tourniquet is more effective in controlling bleeding and reducing complications than excessively tightening the initial one. Administer analgesia as needed after the tourniquet is applied.

An alternative to the tourniquet is applying a blood pressure cuff proximal to the wound. The cuff is then inflated 20-30 mm Hg above systolic blood pressure or over 250 mm Hg, and the tubing is clamped with a hemostat [2]. There are many ways to improvise a tourniquet using non-stretchable clothing and a windlass rod like a pen; however, a commercially designed tourniquet is preferable and not likely to loosen easily with patient movement. 

To safely remove the tourniquet, apply a pressure dressing directly onto the wound. Then, gradually release the tourniquet while carefully monitoring for any signs of bleeding. If bleeding is successfully controlled, keep the tourniquet loosely secured in case of potential re-bleeding. If bleeding recurs, reapply firm pressure by tightening the tourniquet [5].

Precautions

The maximum duration for tourniquet application is 120 minutes [2]. Prolonged tourniquet application can lead to complications such as nerve injury, tissue necrosis, compartment syndrome, and rhabdomyolysis. However, if the extremity is amputated or if the tourniquet has been applied for more than 6 hours, it should not be loosened as permanent muscle damage occurs after 6 hours and might require amputation.1 Moreover, potential reperfusion injury may occur after 60 minutes of tourniquet use, leading to inflammation-induced damage in local areas and systemic effects on vital organs caused by inflammatory mediators [5].

Topical Hemostatic Agents

Another alternative or adjunct to tourniquet use is topical hemostatic agents. These agents create a platform for platelet deposition and facilitate hemostasis [6]. Examples include [1] oxidized cellulose (e.g., Surgicel), dry gelatin (e.g., Gelfoam, Surgifoam), or cyanoacrylate.

Equipment

  • Hemostatic agent (e.g., Combat Gauze, Celox Gauze, or ChitoGauze)
  • Pressure dressing

Technique

The hemostatic gauze is applied with direct pressure for at least 3 minutes. After the field dries, the wound can be sutured, or pressure dressing can be applied. It is important to note that a dry field is required to apply the cyanoacrylate type. Pressure or tourniquet should be used before its application. An alternative to hemostatic gauze is topical thrombin. It can be used directly or diluted with saline and sprayed onto the wound. A concentration of 100 units/mL is effective. In severe bleeding, a concentration of 1000 to 2000 units/mL can be used [1].

Precautions

Potential complications associated with hemostatic agents include excessive granulation tissue and fibrosis with absorbable gelatin agents or foreign body reaction with cellulose [1,7].

Balloon Catheter

Balloon catheters can be used as an improvised tamponade technique to temporarily control severe bleeding from deep injuries, when other conventional methods fail [1,8].

Equipment

  • Fogarty catheters, Foley catheters, or Sengstaken-Blakemore tubes.
  • 10 cc syringe

Technique

The tube is blindly inserted into the wound, then the ballon is inflated to halt bleeding from deep vascular injuries [1].

Suture Ligation

Suture ligation is used for controlling large bleeding vessels. An effective ligation technique requires careful examination and knowledge of the vascular anatomy to trace and identify the sources of bleeding. A retracted artery can be a potential source of delayed bleeding. Therefore, once an injured vessel is identified, the opposite end should also be traced and ligated [1]. 

Equipment

  • Blood pressure cuff
  • Absorbable suture (e.g., Vicryl, Monocryl, and PDS).
  • Haemostat
  • Needle holder
  • Scissors

Technique

A blood pressure cuff is placed proximally and inflated until the bleeding stops to create a clear field. With gradual deflation of the cuff, large bleeding vessels will start to be visible. Ligation is then completed with suturing in the following steps: [1]

  1. Using a haemostat pinch the free end of the bleeding vessel.
  2. Wrap a proper-sized suture around the vessel.
  3. Tie the suture at the base of the vessel.
  4. Release the haemostat carefully (Figure 9).

if the vessel can not be seen, a figure 8 suture can be applied (Figure 10) [1,3]. 

Figure 9 - Vessel ligation technique. (1) Grasp the cut end of the bleeding vessel with a haemostat. (2) Pass an appropriately sized suture around the vessel. (3) Tie and secure the suture around the base of the bleeding vessel. (4) Gently release the haemostat from the blood vessel. (Freeman C, Reichman EF. Hemorrhage Control. In: Emergency Medicine Procedures. 6th ed. Elsevier; 2020:112-1. "Control of the Bleeding Vessel that is Visualized." Adapted and redrawn by Tasnim Ahmed, MD).
Figure 10. Figure 8 stich. A. Needle directions, B.Tie. (Freeman C, Reichman EF. Hemorrhage Control. In: Emergency Medicine Procedures. 6th ed. Elsevier; 2020:112-2. "Control of a Bleeding Vessel Deep or Embedded in Tissue." Adapted and redrawn by Tasnim Ahmed, MD).

Cauterization

Cauterization is cost effective and simple haemostatic technique for small vessels measuring less than 2 mm in diameter. Electrical cauterization  involves using electrical current to heat an electrode, which then is used to thermally burn the vessel wall and seal it with charred tissue [1,10]. 

Chemical cauterization can be achieved using silver nitrate (AgNO3). This involves applying the agent to the vessel wall using an applicator, typically a long and small wooden stick tipped with the silver nitrate. Silver nitrate reacts with proteins in the tissue, forming an insoluble deposit that blocks the blood flow. It is only effective when applied to a dry tissue or minimal oozing [1]. 

Equipment

  • Blood pressure cuff
  • Silver nitrate or electric cautery

Technique

Position a blood pressure cuff proximally and gradually inflate it until bleeding stops, to achieve a clear field. Then gently release the pressure, until the smaller bleeding vessels become visible. Use the electrocautery to burn the end of the bleeding vessel or rub the silver nitrate against it to achieve an artificial clot [1].

Vasoconstrictors

In normal conditions, small vessels spontaneously stop bleeding. However, if bleeding persists, local vasoconstrictors mixed with local anaesthetics can be applied. Local anesthetic solutions containing epinephrine, such as lidocaine and bupivacaine, are readily available in the Emergency Department.

Equipment

  • 10 cc syringe
  • Epinephrine 1:1000
  • Saline-soaked gauze

Technique

Prepare the diluted epinephrine in a 10 cc syringe. Aspirate prior to injection to ensure that the solution is not injected into a blood vessel. Inject 1 to 2 mL of the solution around the bleeding vessel. Apply direct pressure with saline soaked gauze over the wound. Alternatively, spray the wound with the diluted solution. [1,3]

Precautions

It’s important to avoid using epinephrine or other vasoconstrictors in end-arterial areas like fingers, toes, ears, nose, or penis, to avoid organ ischemia.

Complications

Complications arise when the above-listed techniques are either overused or applied inappropriately. For detailed information regarding the particular complications associated with each technique, please refer to the corresponding technique’s “Precautions” section.  

Special Patient Groups

Obtaining hemostasis might be challenging in patients with coagulopathy. Therefore, it is important to remain vigilant and promptly assess the platelet count and plasma coagulation profile (PT/PTT/INR) in patients experiencing external bleeding. The early administration of tranexamic acid, blood products, and cryoprecipitate can aid in achieving hemostasis.

Authors

Picture of Tasnim Ahmed

Tasnim Ahmed

Emergency Medicine Residency graduate from Zayed Military Hospital, Abu Dhabi, UAE. Deputy Editor-in-Chief of the Emirates Society of Emergency Medicine (ESEM) newsletter. Senior Board Member and Website Manager of the Emirates Collaboration of Residents in Emergency Medicine (ECREM). Awarded Resident of the Year twice, at ESEM23 and Menatox23. Passionate about medical education, with a focus on blending art and technology into innovative teaching strategies.

Picture of Abdulla Alhmoudi

Abdulla Alhmoudi

Dr Abdulla Alhmoudi is a Consultant Emergency Medicine, serving at Zayed Military Hospital and Sheikh Shakhbout Medical City - Abu Dhabi. He pursued his residency training in Emergency Medicine at George Washington University in Washington DC and further enhanced his expertise with a Fellowship in Extreme Environmental Medicine. Dr Alhmoudi's passion for medical education is evident in his professional pursuits. He currently holds the position of Associate Program Director at ZMH EM program and is a lecturer at Khalifa University College of Medicine and Health Sciences. Beyond medical education, he maintains a keen interest in military medicine and wilderness medicine.

Listen to the chapter

References

  1. Chapter 112. Hemorrhage Control. In: Reichman EF. eds. Emergency Medicine Procedures, 2e. McGraw Hill; 2013. Accessed May 22, 2023. https://accessemergencymedicine.mhmedical.com/content.aspx?bookid=683&sectionid=45343754
  2. Spehonja A, Prosen G. Basics of Bleeding Control. In: Cevik AA, ed. International Emergency Medicine Education Project. iEM Education Project; 2018:598-601.
  3. Lammers RL, Smith ZE. Principles of wound management. In: Roberts JR, Hedges JR, eds. Roberts & Hedges’ Clinical Procedures in Emergency Medicine. 6th ed. Philadelphia, PA: Elsevier; 2014:611-634.
  4. Department of the Navy. Bleeding. Brooksidepress.org. 2001. Accessed May 22, 2023. https://www.brooksidepress.org/Products/OperationalMedicine/DATA/operationalmed/Manuals/Standard1stAid/chapter3.html.
  5. Lee C, Porter KM, Hodgetts TJ. Tourniquet use in the civilian prehospital setting. Emergency Medicine Journal. 2007;24(8):584-587. doi:10.1136/emj.2007.046359
  6. Sileshi B, Achneck HE, Lawson JH. Management of surgical hemostasis: topical agents [published correction appears in Vascular. 2009 May-Jun;17(3):181]. Vascular. 2008;16 Suppl 1:S22-S28.
  7. Levy JH. Hemostatic agents and their safety. J Cardiothorac Vasc Anesth. 1999;13(4 Suppl 1):6-37.
  8. Feliciano DV, Burch JM, Mattox KL, Bitondo CG, Fields G. Balloon catheter tamponade in cardiovascular wounds. Am J Surg. 1990;160(6):583-587. doi:10.1016/s0002-9610(05)80750-0
  9. Rudge WB, Rudge BC, Rudge CJ. A useful technique for the control of bleeding following peripheral vascular injury. Ann R Coll Surg Engl. 2010;92(1):77-78. doi:10.1308/rcsann.2010.92.1.77
  10. Kamat AA, Kramer P, Soisson AP. Superiority of electrocautery over the suture method for achieving cervical cone bed hemostasis. Obstet Gynecol. 2003;102(4):726-730. doi:10.1016/s0029-7844(03)00622-7

Reviewed and Edited By

Picture of Arif Alper Cevik, MD, FEMAT, FIFEM

Arif Alper Cevik, MD, FEMAT, FIFEM

Prof Cevik is an Emergency Medicine academician at United Arab Emirates University, interested in international emergency medicine, emergency medicine education, medical education, point of care ultrasound and trauma. He is the founder and director of the International Emergency Medicine Education Project – iem-student.org, chair of the International Federation for Emergency Medicine (IFEM) core curriculum and education committee and board member of the Asian Society for Emergency Medicine and Emirati Board of Emergency Medicine.

Blood Transfusion And Its Complications (2024)

by Yaman Hukan, Thiagarajan Jaiganesh

You have a new patient!

A 68-year-old male with a history of controlled HTN, DM, and Ischemic heart disease presents to the Emergency Department with complaints of easy fatiguability that started 2 months ago. He reports a gradual onset of symptoms and inability to tolerate his usual morning walk. He denies chest pain or palpitations. Upon further questioning, he mentioned that he noticed his clothes getting loose, and his family noticed he had lost weight. On review of systems, he states he has bouts of diarrhea with dark stools. Upon arrival, his vitals are Temp 36.9 C, HR 105 BPM, BP 122/68 mmHg, RR 17 BPM, and SpO2 of 98% on RA.  Blood investigations reveal an Hgb level of 5.0 g/dL. Therefore, you decide to initiate a Packed RBC transfusion in the ER. One hour after starting the transfusion, you are called by the nurse as the patient is becoming distressed. You attend to the patient and notice him to be in severe respiratory distress.

What do you need to know?

Often, patients presenting to Emergency Departments require a blood transfusion. According to the National Blood Collection and Utilization survey administered by the US Department of Health and Human Services, 2019 around 1 million RBC transfusions took place in EDs across the United States [1]. The clinical conditions necessitating a blood transfusion include upper and lower gastrointestinal bleeding, traumatic shock, symptomatic anemia, etc., to name a few. Therefore, medical trainees and emergency physicians must be aware of complications that may arise from blood transfusions and manage them appropriately.

Commonly administered blood products in the emergency department (ED) include packed red blood cells (PRBCs), fresh frozen plasma (FFP), platelet concentrates, and cryoprecipitate. PRBCs are frequently used to increase oxygen-carrying capacity in patients with significant anemia or hemorrhage. FFP provides essential clotting factors, making it valuable in cases of coagulopathy or massive transfusion protocols. Platelet concentrates are utilized to manage thrombocytopenia or platelet dysfunction, while cryoprecipitate supplies fibrinogen, von Willebrand factor, and other clotting factors, supporting hemostasis in patients with severe bleeding or fibrinogen deficiency.

The choice of components should be directed by the patient’s clinical condition, rate of bleeding, cardiopulmonary status, and operative intervention, with the goal of restoring volume and oxygen-carrying capacity [2].

Administering blood and blood products to patients has resulted in numerous adverse reactions. These reactions are broadly classified as either Acute (onset within 24 hrs), such as febrile nonhemolytic reactions, or Delayed (onset beyond 24 hrs), such as delayed hemolytic reactions [3].

Data from the National Healthcare Safety Network Hemovigilance Module in the United States demonstrate that 1 in 455 blood components transfused was associated with an adverse reaction. However, the incidence of serious reactions was much lower, at 1 in 6224. Despite the relatively lower rates of serious reactions, 23 fatalities were recorded between 2013 and 2018 [4].

Severe adverse reactions result from transfusing incompatible (ABO or Rh) blood. The ABO blood group system remains of extreme importance in blood transfusions, as it is the most immunogenic of all blood group antigens [5]. The four blood groups are A, B, O, and AB.

The table shows the summary of ABO Antigens and Antibodies contained within each blood type.

 

A

B

O

AB

Antigens

A

B

None

A and B

Antibodies

Anti-B

Anti-A

Anti-A & Anti-B

None

There are several ABO blood group antigens expressed on every RBC cell. Each blood group early on during life forms antibodies against ABO antigens not found on the surface of RBCs. When an individual is transfused ABO-incompatible blood, preformed antibodies in its own serum react against the donor’s red blood cells, causing rapid acute intravascular hemolysis, a life-threatening transfusion reaction.

The second significant blood grouping system is the Rh system. The presence of Rh Antigen implies that the patient is Rh(+) (e.g., Blood group O+). Patients who are Rh(-) lack the RhD antigen. Therefore, their blood develops antibodies against Rh(+) blood groups if they are ever exposed to it. This incompatibility can lead to a hemolytic reaction, but it is much less likely than a hemolytic reaction due to ABO incompatibility. The clinical significance of the Rh system lies in the pregnancy setting when a Rh(-) mother is pregnant with an Rh(+) fetus. Upon first exposure to the positive blood from the fetus, the mother’s blood would form antibodies against Rh-blood. In case of a repeated pregnancy with Rh+ fetus, the mother’s antibodies cross the placenta and attack the RBCs of the fetus, which can lead to a condition called hemolytic disease of the newborn [6]. This is the reason why women of childbearing age should always receive O(-) blood in the setting of acute hemorrhage needing a blood transfusion, as opposed to men who may receive O(+) blood safely.

Medical History

Should a patient receiving or recently received a blood or a blood product transfusion develop new signs and symptoms, consider a transfusion reaction. Commonly encountered signs and symptoms of mild transfusion reactions include:

  • Increase in body temperature/fever,
  • Chills/Rigors,
  • Pruritis, New rash, or swelling of the mucous membranes.

Severe reactions include:

  • Difficulty in breathing,
  • Respiratory distress,
  • Altered level of consciousness,
  • Decreased urinary output.

Reaction Types

Acute Transfusion Reactions

Febrile nonhemolytic transfusion reaction

This is one of the most common transfusion reactions, occurring at a rate of around 1:900 [7]. It has been attributed to cytokines released from white blood cells and their accumulation in blood products [8].

Diagnostic criteria

  • A reaction which occurs during or within 4 hours of cessation of transfusion,

AND

  • Either Fever (> 38 C° and a change of at least 1 C° from pretransfusion value) OR Chills/Rigors is present [9].

Caution must be exercised when distinguishing between febrile nonhemolytic transfusion reactions and hemolytic reactions, which could also present with fever. Febrile nonhemolytic transfusion reaction is considered a diagnosis of exclusion [8]. In the case of first onset of a febrile reaction, a hemolytic reaction must be suspected until proven otherwise.

Allergic and anaphylactic transfusion reactions

Another very common non-infectious transfusion reaction is allergy. Allergic reactions vary in severity from mild to severe. Mild reactions are primarily characterized by itching and hives. They occur at a rate of 1:1200 transfusions. However, rates may be much higher due to underreporting [7].

On the other hand, anaphylactic reactions are typically more severe and occur at a rate of around 1:30000 blood transfusions [7]. Anaphylactic reactions are acute systemic allergic reactions characterized most significantly by hypotension and/or respiratory compromise. They typically arise abruptly within 0-4 hours of initiating the transfusion.

Allergic reactions are thought to be multifactorial in etiology, mainly caused by an antibody-mediated response to donor proteins. These reactions fall under Type 1 hypersensitivity reactions and involve pre-existing IgE antibodies [10].

The criteria for a definitive diagnosis of an allergic reaction encompasses two or more of the following during or within 4 hours of cessation of transfusion: conjunctival edema, edema of lips, tongue, and uvula; Erythema and edema of the periorbital area, generalized flushing, hypotension, localized angioedema, maculopapular rash, pruritis (itching), respiratory distress/bronchospasm, and urticaria (hives) [9].

Acute hemolytic transfusion reaction

The hemolytic transfusion reaction is perhaps the most severe and life-threatening transfusion reaction. They account for 5% of all severe adverse reactions of blood transfusions.  Reactions due to ABO incompatibility occur at a rate of 1:200000 [7]. The rate significantly increases in the setting of uncross-matched blood transfusions in bleeding patients (e.g., major trauma), where the rate reaches as high as 1:2000 [11].

Despite their relative rarity, mainly due to growing hemovigilance procedures and schemes, acute hemolytic transfusion reactions can lead to significant morbidity and mortality. Mortality rates increase with the increase in the volume of the incompatible transfused blood. However, even a volume of as low as 30 mL could lead to a severe fatal reaction [12].

Reactions due to ABO system incompatibility most often occur due to a clerical or laboratory error, including misidentification of patient or mislabelling blood samples collected from the recipient for crossmatching. The recipient’s blood contains pre-existing antibodies against ABO antigens that are not present in their blood. When incompatible blood is administered, those pre-existing antibodies attack the donor’s RBCs. Through complement activation and membrane attack complex, the donor’s RBCs are destroyed, leading to intravascular hemolysis, which subsequently gives rise to the clinical features of hemolysis, including acute tubular necrosis, renal failure, hypotension, disseminated intravascular coagulopathy (DIC), and shock [13].

The criteria for the definitive diagnosis of acute hemolytic transfusion reactions is complex and includes components that can be obtained from clinical presentation combined with laboratory studies, detailed below [9]:

Decision-Making Algorithm for Suspected Hemolytic Transfusion Reaction
1. Identify New-Onset Symptoms

Check if the patient has developed any new symptoms during the transfusion or within 24 hours of transfusion cessation. The presence of any of the following symptoms warrants further investigation:

  • Back or flank pain
  • Chills or rigors
  • Disseminated intravascular coagulation (DIC)
  • Epistaxis (nosebleed)
  • Fever
  • Hematuria (indicative of gross hemolysis)
  • Hypotension
  • Oliguria or anuria (reduced or absent urine output)
  • Pain and/or oozing at the IV site
  • Renal failure

AND

Check for Laboratory Evidence of Hemolysis
Confirm the presence of at least two of the following laboratory findings:

  • Decreased fibrinogen
  • Decreased haptoglobin
  • Elevated bilirubin
  • Elevated lactate dehydrogenase (LDH)
  • Hemoglobinemia
  • Hemoglobinuria
  • Plasma discoloration consistent with hemolysis
  • Spherocytes visible on blood film

AND EITHER

Determine the Mechanism of Hemolysis. Differentiate between immune-mediated and non-immune-mediated hemolysis.

IMMUNE-MEDIATED HEMOLYSIS

  • Perform a Direct Antiglobulin Test (DAT) to detect anti-IgG or anti-C3.
  • Conduct an elution test to detect any alloantibodies on the transfused red blood cells. If the DAT or elution test is positive, this suggests an immune-mediated HTR.

NON-IMMUNE-MEDIATED HEMOLYSIS

  • If serologic testing is negative and there is evidence of a physical cause (e.g., thermal, osmotic, mechanical, or chemical), consider a non-immune etiology. A confirmed physical cause indicates a non-immune-mediated HTR.
Transfusion related acute lung injury (TRALI)

Transfusion-related acute lung injury (TRALI) is an infrequent but incredibly serious blood transfusion reaction. Despite only occurring at the rate of 1:60000 [7], TRALI is reported to be one of the most life-threatening complications according to data from the US Food and Drug Administration, coming in 2nd place among the most fatal blood transfusion reactions in the United States between 2016 and 2020, causing 21% of reported fatalities [14].

TRALI results in a constellation of symptoms that manifest as acute respiratory distress along with hemodynamic instability and can occur with virtually all blood components. The proposed mechanism is complex and involves activation of pulmonary endothelium and polymorphonuclear leucocytes and transfusion of plasma-containing antibodies directed against antigens on the surface of those leucocytes, leading to their activation [15].

The TRALI diagnosis remains clinical and significantly overlaps with other respiratory conditions (e.g., ARDS and Transfusion-associated circulatory overload). A set of clinical features have been adopted to define TRALI, including [9]:

  • No evidence of acute lung injury prior to transfusion, AND,
  • Acute lung injury onset during or within 6 hours of cessation of transfusion, AND,
  • Hypoxemia defined by any of the following methods:
      • PaO2/FiO2 less than or equal to 300 mmHg
      • Oxygen saturation less than 90% on room air
      • Other clinical evidence

AND,

  • Radiographic evidence of bilateral infiltrates
  • No evidence of left atrial hypertension (i.e., circulatory overload)
Transfusion associated circulatory overload (TACO)

The last of the acute transfusion reactions is transfusion-associated circulatory overload (TACO), which carries the highest mortality risk among all reactions. Between 2016 and 2020, 34% of recorded fatalities due to reactions to blood transfusions were caused by TACO [14]. It is relatively more common than TRALI, occurring at an estimated rate of 1:9000 transfusions [7]. TACO can present on a spectrum of mild symptoms to life-threatening ones. Significant overlap exists between TRALI and TACO as both may cause respiratory distress and potentially lead to hemodynamic instability.

TACO is a form of volume overload leading to pulmonary edema. Patients who are older than 70 years of age, suffer from pre-existing cardiac disease, or have a history of renal dysfunction are at increased risk of developing this complication [16].

The criteria for diagnosing TACO have evolved several times over the years. Currently, establishing a definitive diagnosis would require the following [9]:

New onset or exacerbation of 3 or more of the following within 12 hours of cessation of transfusion:

At least 1 of the following two items:-

  1. Evidence of acute or worsening respiratory distress (dyspnea, tachypnoea, cyanosis, and decreased oxygen saturation values in the absence of other specific causes) and/or 
  2. Radiographic or clinical evidence of acute or worsening pulmonary edema (crackles on lung auscultation, orthopnea, cough, a third heart sound, and pinkish frothy sputum in severe cases) or both

             AND;

  • Elevated brain natriuretic peptide (BNP) or NT-pro BNP relevant biomarker
  • Evidence of cardiovascular system changes not explained by underlying medical condition (Elevated central venous pressure, evidence of left heart failure including development of tachycardia, hypertension, widened pulse pressure, jugular venous distension, enlarged cardiac silhouette, and/or peripheral edema)
  • Evidence of fluid overload

Delayed Transfusion Reactions

In addition to acute blood transfusion reactions, there are certain reactions which could appear days or weeks following blood transfusions.

Delayed hemolytic transfusion reaction

Delayed hemolytic transfusion reactions are less severe forms of hemolytic reactions in patients receiving blood transfusions. They appear to be caused by secondary (anamnestic) responses in patients who have already received transfusions. They rarely cause life-threatening or serious manifestations [17]. Those reactions may occur up to 4 weeks following the completion of the transfusion. They are less common than acute hemolytic transfusions, occurring at a rate of 1:22000 transfusions [7].

The criteria for definitive diagnosis of delayed hemolytic transfusion reactions include [9]:

Positive direct antiglobulin test (DAT) for antibodies developed between 24 hours and 28 days after cessation of transfusion

AND EITHER

  • Positive elution test with alloantibody present on the transfused red blood cells OR
  • Newly identified red blood cell alloantibody in recipient serum

AND EITHER

  • Inadequate rise of post-transfusion hemoglobin level or rapid fall in hemoglobin back to pre-transfusion levels OR
  • Otherwise, unexplained appearance of spherocytes
Transfusion associated graft vs. host disease

Transfusion-associated graft vs. host disease is an extremely rare and exceptionally dangerous complication of transfusions, occurring at a rate of 1 in every 13 million [7]. It can present any time up to 6 weeks following the transfusion. It is thought to be caused by viable lymphocytes in the donor’s blood recognizing their new host’s cells as foreign and attacking them, often leading to fatal outcomes [17].

Diagnosis is made when the following characteristics appear between 2 days to 6 weeks from cessation of transfusion [9]:

  • Characteristic rash: erythematous, maculopapular eruption centrally that spreads to extremities and may, in severe cases, progress to generalized erythroderma and hemorrhagic bullous formation.
  • Diarrhea
  • Fever
  • Hepatomegaly
  • Liver dysfunction (i.e., elevated ALT, AST, Alkaline phosphatase, and bilirubin)
  • Marrow aplasia
  • Pancytopenia

AND

  • Characteristic histological appearance of skin or liver biopsy
Post transfusion purpura

This reaction may appear up to 2 weeks post-transfusion and involves platelets [17]. Its prevalence is thought to be around 1 in 57,000 transfusions [7]. A definitive diagnosis may be reached by the following two findings [9]:

  • Alloantibodies in the patient directed against human platelet antigens (HPAs) or other platelet-specific antigens detected at or after the development of thrombocytopenia AND
  • Thrombocytopenia (i.e., decrease in platelets to less than 20% of pre-transfusion count)

Physical Examination

Transfusion reactions could manifest in several organ systems. It is important to exercise vigilance when approaching a patient with a suspected transfusion reaction, as clinical features significantly overlap between several reactions.

One unified step in the physical examination of patients with suspected transfusion reactions is to obtain a complete set of vital signs. This can provide important clues to the diagnosis. For instance, a rise in baseline temperature could indicate a Febrile nonhemolytic reaction, Acute hemolytic reaction, or even TRALI.

Hypotension is a feature of anaphylaxis or acute hemolysis. In addition, while keeping in mind that TRALI can present with either Hypotension or Hypertension, hypotension is more common in TRALI [18] and can help distinguish it from TACO, which can present with respiratory distress coupled with hypertension. Tachypnea and desaturation can be signs of respiratory distress, which would point to either TRALI or TACO as possible diagnoses. Following vitals, emphasis should be on signs relating to the suspected reactions.

Chills and rigors might be observed in acute hemolytic transfusion reaction, along with fever and hypotension. Respiratory status examination is essential and could yield signs of acute distress, including tachypnea, oxygen desaturation, use of accessory muscles, and wheezing. Patients would be anxious, with some reporting a sense of impending doom. Additionally, urine frequency and color should be observed for oliguria or dark-colored urine, pointing to acute hemolysis.

Observe any signs of maculopapular urticarial rash in suspected allergic reactions. Also, look for any signs of dyspnea, wheezing, anxiety, and angioedema. Anaphylaxis could further present with hypotension which could pose a diagnostic dilemma.

There are significant similarities between TRALI and TACO. Examination should look for dyspnea, tachypnoea, cyanosis, and decreased oxygen saturation. Furthermore, auscultation for crackles might be evidence of pulmonary edema. Orthopnea, cough, a third heart sound, and pinkish frothy sputum could all be clues leading to the diagnosis of these reactions.

Alternative Diagnoses

When new symptoms arise after blood transfusions, the diagnosis of transfusion reactions should be established. However, an extensive differential diagnosis list must be carefully formulated depending on the presentation.

In the context of transfusions, certain signs and symptoms may indicate potential complications or adverse reactions. A new rash or swelling of mucous membranes could suggest an allergic reaction, anaphylaxis, urticaria, food allergies, or angioedema. Dyspnea, or respiratory distress, may be indicative of transfusion-related acute lung injury (TRALI), transfusion-associated circulatory overload (TACO), anaphylaxis, cardiogenic pulmonary edema, acute respiratory distress syndrome, or acute chest syndrome. Hypotension could point to anaphylaxis, TRALI, septic shock, hemorrhagic shock, or neurogenic shock. Lastly, the presence of fever may indicate a febrile non-hemolytic reaction, an acute hemolytic reaction, an infection from any source, or sepsis. Identifying these symptoms promptly is essential to manage and mitigate potential adverse events during transfusions.

The table summarizes signs&symptoms and potential differential diagnoses. 

Signs and Symptoms

Differential Diagnoses

New rash, or swelling of mucous membranes

Allergic reaction, Anaphylaxis, Acute, Urticaria, Food Allergies, Angioedema

Dyspnea (Respiratory distress)

TRALI, TACO, Anaphylaxis, Cardiogenic pulmonary edema, Acute respiratory distress syndrome, Acute chest syndrome

Hypotension

Anaphylaxis, TRALI, Septic shock, Hemorrhagic shock, Neurogenic shock

Fever

Febrile nonhemolytic reaction, Acute hemolytic reaction, infection of any source, sepsis

Acing Diagnostic Testing

While most transfusion reaction diagnoses are primarily clinical, few diagnostic tests may assist clinicians in establishing a diagnosis.

  1. Visual inspection of the pre-transfusion sample for its color and any unusual clumps [19].
  2. Allergic reactions: IgA levels could also be obtained in patients with suspected IgA deficiency, although the diagnosis for moderate or severe allergic reactions is usually clinical. Eosinophilia could indicate allergic reactions but may not always be present [10].
  3. Hemolytic reactions: Elevated Lactate dehydrogenase levels (LDH) as well as indirect bilirubin levels with decreased haptoglobin levels would suggest a hemolytic reaction arising out of an ABO incompatibility. Elevated PTT and PT/INR, as well as D-Dimer coupled with decreased fibrinogen, would suggest the presence of DIC. Blood film can be examined for schistocytes or spherocytes [12]. Dark urine could suggest hemoglobinuria. Direct antiglobulin test (DAT) for anti-IgG or anti-C3 and elution test with alloantibody present on the transfused red blood cells would help.
  4. TRALI & TACO: arterial blood gas (ABG) is used to calculate the PaO2/FiO2 ratio, and Chest XR is used to evaluate the presence of bilateral infiltrates or features of pulmonary edema. Bedside ultrasound can confirm the absence of circulatory overload in TRALI, which is a distinguishing feature from TACO. Additionally, a BNP level should be obtained when evaluating for TACO.

Risk Stratification

Unfortunately, no objective risk stratification tool exists that would lead to recognizing patients with worse outcomes due to transfusion reactions.

Characteristics which place patients at increased risk of developing transfusion reactions are:

  • Previous transfusion history,
  • Abortions or termination of pregnancy history,
  • Longer blood storage time,
  • Receiving three or more units of blood [3].
  • Critically ill and surgical patients (Risk of mortality due to TRALI appears to be higher) [20].

Management

In case of transfusion reactions, the ABCDE algorithm for managing conditions in the emergency department should be followed. The airway must be assessed for patency and secured if needed, followed by addressing breathing and circulation.

The cornerstone of managing most transfusion reactions is stopping the transfusion and maintaining Intravenous access. In all reactions, the next step is to confirm the details of the transfused unit, make sure no clerical error occurred, and then report the reaction to the concerned blood bank [17].

Febrile nonhemolytic reaction:  Management of this reaction encompasses frequent monitoring of vital signs and administering antipyretics. Transfusion can be continued in stable patients with no other symptoms [12]. However, this remains a diagnosis of exclusion, and other reactions must be considered.

Mild allergic reaction: An H1 antihistamine (e.g., Diphenhydramine 25-50 mg IV) should be administered for symptom management in case of a mild allergic reaction. Restart the transfusion under direct supervision at a slower rate upon resolution of symptoms. In case of recurrence, transfusion must be suspended [17].

Anaphylaxis reaction: Manage as per standard institutional protocol or as delineated in an earlier chapter within this textbook (e.g., IM 1:1000 Epinephrine, H1 antihistamine, e.g., IV Diphenhydramine, Beta-adrenergic drugs, e.g., Salbutamol nebs in case of wheezing and/or bronchospasm, Steroids, e.g., Hydrocortisone and IV Fluids as required) [17].

Acute hemolytic transfusion reaction: The onset of hemodynamic instability will indicate an acute hemolytic transfusion reaction, and it is imperative to immediately halt the transfusions. Treatment is largely supportive. Focus on supporting the respiratory, cardiovascular, and renal systems and treating possible complications such as DIC to halt the patient’s condition [21].

Transfusion-related acute lung injury (TRALI): Similar to acute hemolytic reaction, treatment of TRALI is supportive. Most importantly, support of ventilatory status should be established with noninvasive or invasive means. Most patients who develop TRALI require ventilatory support [22]. As most patients with TRALI develop hypotension, supporting hemodynamics with IV fluids and possible vasopressors may be needed to ensure adequate organ perfusion.

Transfusion-associated circulatory overload (TACO): Since TACO reflects a volume overload status, this condition can be treated similarly to other conditions that result in volume overload. In deteriorating patients, ventilatory support may be needed through noninvasive or mechanical ventilation. Furosemide 0.5/1 mg/kg may be used. In addition, IV Nitroglycerin 50 – 100 mcg as an initial dose may theoretically have a role in clinical status improvement [16,17].

Special Patient Groups

Pregnant Patients

This patient population should always receive O(-) blood when prompt uncross-matched blood is needed for transfusion to minimize the risk of Rh(-) mothers developing antibodies against the Rh(+) fetus, leading to subsequent hemolytic disease of the newborn [5].

Geriatrics

About half of RBC units are administered to patients aged 70 and above [23]. They are frail, have various comorbid conditions, and age-related altered physiology. Clinicians must base their transfusion decisions on the risk-benefit ratio for elderly patients [24]. TACO is the most common transfusion reaction in elderly patients. It occurs at a substantially higher rate in this population compared to younger patients, and those with more comorbidities are at higher risk. Slower transfusion rates are recommended to mitigate the risk [25]. In addition, several studies have mentioned that blood transfusions in the elderly are linked to the risk of developing delirium, although the causation is unknown [26].

Pediatrics

According to a recent meta-analysis, the incidence of transfusion reactions is higher in children than in adults, including rare transfusion reactions [27], due to their size difference (volume-related) and immature liver [28].

When To Admit This Patient

It is advisable to observe patients with hemodynamic instability or severe reactions following a blood transfusion (e.g., ICU for Acute hemolytic reaction). No clear guidelines exist on the criteria for admission for patients with transfusion reactions, and the decision might need to be made on a case-by-case basis, depending on the clinician’s experience and clinical evaluation.

Revisiting Your Patient

Recall that your patient was started on a blood transfusion for a Hgb of 5.0 g/dl and then developed respiratory distress. You arrive at the room and connect to the patient on a monitor. His vitals now show a temperature of 38 C, HR of 132 BPM, RR of 35, BP of 205/120, and SpO2 of 75% on Room Air. You immediately assess the airway and note that the patient is talking clearly but cannot complete full sentences. No secretions in the oral cavity. You judge the airway to be patent and move to assess breathing. He is tachypneic and desaturating, and you immediately place him on 15L O2 via a nonrebreather mask. The patient’s SpO2 picks up to 90%. Upon chest inspection, you hear diffuse crackles. The patient is also unable to lie supine. Hypertension and tachycardia are noted, as well as elevated Jugular venous pressure.

By now, you judge the patient has developed a transfusion reaction, and you immediately order the nurse to suspend the transfusion and notify the blood bank.

An X-ray was ordered, and it showed features of pulmonary edema as well as blunting of the costophrenic angles. Arterial blood gas shows a PaO2/FiO2 ratio 190 and a lactate 4. A BNP is sent and returns at 25,000 pg/mL

Upon review of the patient, he is in significant distress despite the nonrebreather mask, so the respiratory therapist is contacted to initiate BiPAP treatment. You diagnose TACO and, in addition, start the patient on 100 mcg/min of IV Nitroglycerin and a 40 mg dose of IV Furosemide.

The patient started improving shortly after and stated that his breathing was improving. The patient was admitted to the ICU for further stabilization and management of his condition.

Author

Picture of Yaman Hukan

Yaman Hukan

Yaman Hukan is an Emergency Medicine resident at Tawam Hospital in the United Arab Emirates. He completed his bachelor's of medicine (MBBS) degree in 2018 from the University of Sharjah. He is interested in humanitarian medicine. As a medical student, he joined the Syrian American medical society (SAMS) on several of their missions to provide healthcare for Syrian refugees in Jordan. His interests also include resuscitation and toxicology, a field in which he hopes to pursue further training.

Picture of Thiagarajan Jaiganesh

Thiagarajan Jaiganesh

Dr. Jaiganesh is a Chairman and Consultant in Adult and Pediatric Emergency Medicine and serves as an Adjunct Assistant Professor at UAE University. As the former Director of the Emergency Medicine Residency Program at Tawam Hospital in Al Ain, UAE, Dr. Jaiganesh is dedicated to training the next generation of emergency medicine professionals. With a strong academic and professional background, Dr. Jaiganesh has published numerous peer-reviewed articles on emergency medicine and contributes as a Section Editor and Chapter Author for notable medical texts, including the Oxford Handbook for Medical School. A sought-after speaker, Dr. Jaiganesh has been invited to present at numerous national and international conferences and serves as an instructor in various life support courses. Additionally, Dr. Jaiganesh is an expert in medico-legal and clinical negligence matters, providing valuable insights into complex legal and ethical cases in healthcare.

Listen to the chapter

References

  1. Mowla SJ, Sapiano MRP, Jones JM, Berger JJ, Basavaraju SV. Supplemental findings of the 2019 National Blood Collection and Utilization Survey. Transfusion. 2021;61 Suppl 2(Suppl 2):S11-S35. doi:10.1111/trf.16606
  2. Kumar TA, Geet A. Blood transfusion therapy and related complication. In: Richhariya D, ed. Textbook of Emergency Medicine including intensive care and trauma. New Delhi: Jaypee brothers medical publishers; 2022: 990
  3. Gelaw Y, Woldu B, Melku M. Proportion of Acute Transfusion Reaction and Associated Factors Among Adult Transfused Patients at Felege Hiwot Compressive Referral Hospital, Bahir Dar, Northwest Ethiopia: A Cross-Sectional Study. J Blood Med. 2020;11:227-236. Published 2020 Jun 30. doi:10.2147/JBM.S250653
  4. Kracalik I, Mowla S, Basavaraju SV, Sapiano MRP. Transfusion-related adverse reactions: Data from the National Healthcare Safety Network Hemovigilance Module – United States, 2013-2018. Transfusion. 2021;61(5):1424-1434. doi:10.1111/trf.16362
  5. Dean L. The ABO Blood Group. In: internet, ed. Blood Groups and Cell Antigens. Bethesda MD: National Center for Biotechnology Information (US); 2005: 25-31
  6. Dean L. The ABO Blood Group. In: internet, ed. Blood Groups and Cell Antigens. Bethesda MD: National Center for Biotechnology Information (US); 2005: 39-44
  7. Goel R, Tobian AAR, Shaz BH. Noninfectious transfusion-associated adverse events and their mitigation strategies. Blood. 2019;133(17):1831-1839. doi:10.1182/blood-2018-10-833988
  8. Shmookler AD, Flanagan MB. Educational Case: Febrile Nonhemolytic Transfusion Reaction. Acad Pathol. 2020;7:2374289520934097. Published 2020 Jul 14. doi:10.1177/2374289520934097
  9. Division of healthcare quality promotion. National healthcare safety network biovigilance component hemovigilance module surveillance protocol. Atlanta, GA: Center of disease control and prevention; February 2023: 9-22
  10. Tobian A. Immunologic transfusion reactions. UpToDate. https://www-uptodate-com.eu1.proxy.openathens.net/contents/immunologic-transfusion-reactions?search=anaphylaxis%20transfusion&source=search_result&selectedTitle=1~150&usage_type=default&display_rank=1 . Updated on Sep 06, 2022. Accessed on March 15, 2023.
  11. Fiorellino J, Elahie AL, Warkentin TE. Acute haemolysis, DIC and renal failure after transfusion of uncross-matched blood during trauma resuscitation: illustrative case and literature review. Transfus Med. 2018;28(4):319-325. doi:10.1111/tme.12513
  12. San Miguel C. & Kaide C. Blood and Blood Components. In: Walls R., ed. Rosen’s Emergency Medicine: Concepts and Clinical Practice. Philadelphia, PA: Elsevier; 2023: 1452-1561
  13. Panch SR, Montemayor-Garcia C, Klein HG. Hemolytic Transfusion Reactions. N Engl J Med. 2019;381(2):150-162. doi:10.1056/NEJMra1802338
  14. Center for Biologics Evaluation and Research. Fatalities reported to FDA following blood collection and transfusion. United States: Food and drug administration; 2020: 4-5
  15. Otrock ZK, Liu C, Grossman BJ. Transfusion-related acute lung injury risk mitigation: an update. Vox Sang. 2017;112(8):694-703. doi:10.1111/vox.12573
  16. van den Akker TA, Grimes ZM, Friedman MT. Transfusion-Associated Circulatory Overload and Transfusion-Related Acute Lung Injury [published correction appears in Am J Clin Pathol. 2022 Nov 3;158(5):665]. Am J Clin Pathol. 2021;156(4):529-539. doi:10.1093/ajcp/aqaa279
  17. Delaney M, Wendel S, Bercovitz RS, et al. Transfusion reactions: prevention, diagnosis, and treatment. Lancet. 2016;388(10061):2825-2836. doi:10.1016/S0140-6736(15)01313-6
  18. Vlaar AP, Juffermans NP. Transfusion-related acute lung injury: a clinical review. Lancet. 2013;382(9896):984-994. doi:10.1016/S0140-6736(12)62197-7
  19. Al-Riyami AZ, Al-Hashmi S, Al-Arimi Z, et al. Recognition, Investigation and Management of Acute Transfusion Reactions: Consensus guidelines for Oman. Sultan Qaboos Univ Med J. 2014;14(3):e306-e318.
  20. Gajic O, Rana R, Winters JL, et al. Transfusion-related acute lung injury in the critically ill: prospective nested case-control study. Am J Respir Crit Care Med. 2007;176(9):886-891. doi:10.1164/rccm.200702-271OC
  21. Tobian A. Hemolytic transfusion reactions. UpToDate. https://www-uptodate-com.eu1.proxy.openathens.net/contents/hemolytic-transfusion-reactions?sectionName=DELAYED%20HEMOLYTIC%20TRANSFUSION%20REACTIONS%20AND%20DELAYED%20SEROLOGIC%20TRANSFUSION%20REACTIONS&search=blood%20transfusion%20reaction&topicRef=95132&anchor=H354791&source=see_link#H354791. Updated on Jan 05, 2022. Accessed on March 15, 2023
  22. Vlaar AP, Binnekade JM, Prins D, et al. Risk factors and outcome of transfusion-related acute lung injury in the critically ill: a nested case-control study. Crit Care Med. 2010;38(3):771-778. doi:10.1097/CCM.0b013e3181cc4d4b
  23. Bosch MA, Contreras E, Madoz P, et al. The epidemiology of blood component transfusion in Catalonia, Northeastern Spain. Transfusion. 2011;51(1):105-116. doi:10.1111/j.1537-2995.2010.02785.x
  24. Boureau AS, de Decker L. Blood transfusion in older patients. Transfus Clin Biol. 2019;26(3):160-163. doi:10.1016/j.tracli.2019.06.190
  25. Menis M, Anderson SA, Forshee RA, et al. Transfusion-associated circulatory overload (TACO) and potential risk factors among the inpatient US elderly as recorded in Medicare administrative databases during 2011. Vox Sang. 2014;106(2):144-152. doi:10.1111/vox.12070
  26. van der Zanden V, Beishuizen SJ, Swart LM, de Rooij SE, van Munster BC. The Effect of Treatment of Anemia with Blood Transfusion on Delirium: A Systematic Review. J Am Geriatr Soc. 2017;65(4):728-737. doi:10.1111/jgs.14564
  27. Wang Y, Sun W, Wang X, et al. Comparison of transfusion reactions in children and adults: A systematic review and meta-analysis. Pediatr Blood Cancer. 2022;69(9):e29842. doi:10.1002/pbc.29842
  28. Sostin N, Hendrickson JE. Pediatric Hemovigilance and Adverse Transfusion Reactions. Clin Lab Med. 2021;41(1):51-67. doi:10.1016/j.cll.2020.10.004

Reviewed and Edited By

Picture of Arif Alper Cevik, MD, FEMAT, FIFEM

Arif Alper Cevik, MD, FEMAT, FIFEM

Prof Cevik is an Emergency Medicine academician at United Arab Emirates University, interested in international emergency medicine, emergency medicine education, medical education, point of care ultrasound and trauma. He is the founder and director of the International Emergency Medicine Education Project – iem-student.org, chair of the International Federation for Emergency Medicine (IFEM) core curriculum and education committee and board member of the Asian Society for Emergency Medicine and Emirati Board of Emergency Medicine.

The ABCDE Approach to Undifferentiated Critically Ill and Injured Patient (2024)

by Roxanne R. Maria, Hamid A. Chatha

You have a new patient!

A 40-year-old male, a truck driver, is involved in a head-on collision with another vehicle. He has been brought in by ambulance. According to the paramedics, the vehicles were traveling at approximately 85 km/hr, and the patient was restrained by a seatbelt. On arrival at the Emergency Department (ED), the patient is agitated and mildly disoriented. He is tachypneic with a respiratory rate of 30/min, maintaining an O2 saturation of 95% on 12 L/min oxygen via a non-rebreather mask, heart rate of 128 beats/min, blood pressure of 90/52 mmHg, and temperature of 36.1°C. The patient also received 1 L of 0.9% normal saline and 1 unit of O-negative packed red cells in the ambulance. Despite this, his respiratory rate, heart rate, and level of disorientation have worsened.

Emergency Department

In the ED, patients present with a variety of clinical presentations, including both life-threatening and non-life-threatening. Some may have been seen and referred by a clinician before arrival or brought to the department after pre-hospital assessment and care by the emergency medical services (EMS) [1]. Health emergencies affect all age groups and include conditions like acute coronary syndrome, strokes, acute complications of pregnancy, or any chronic illness. Emergency health care providers should respond to these clinically ‘undifferentiated’ patients with symptoms for which the diagnosis may not be known [2].  The root cause of most life-threatening conditions in the ED may be medical or surgical, infection or trauma [2].

In the Emergency Department (ED), there are several potentially life-threatening presentations that demand immediate stabilization. These include trauma, which can result from various forms of accidents or injuries, and shortness of breath, which might indicate critical respiratory distress. An altered mental state also requires prompt attention, as it may signal underlying neurological or systemic issues. Shock, often evidenced by dangerously low blood pressure. Chest pain or discomfort, which could be indicative of a cardiac event, are other urgent concerns. Additionally, cases of poisoning, ingestion of harmful substances, or exposure to toxic materials also necessitate rapid intervention to prevent further harm. Each of these presentations is a medical priority, highlighting the importance of timely and effective response in the ED to ensure patient safety and stability.

These symptoms maybe the only picture that the patients present with, and may constitute the early stage of a critical illness requiring rapid, appropriate intervention and resuscitation, even when the patient seems to appear relatively well [2].

Emergency conditions often require immediate intervention long before a definitive diagnosis is made to stabilize the critically ill patient [3]. Thus, this chapter intends to briefly introduce a basic systematic approach to identifying and managing acute, potentially life-threatening conditions in these patients. This approach will enable all frontline providers, including students, nurses, pre-hospital technicians, and physicians, to manage these patients even in the setting of limited resources [2].

A complete assessment and management of each of the presentations mentioned above is beyond the scope of this chapter. However, the initial approach remains the same, regardless of the patient population or setting [4].

History of the ABCDE approach

The ABC mnemonic’s origins may be traced back to the 1950s. The first two letters of the mnemonic, A and B, resulted from Dr Safar’s description of airway protection techniques and administration of rescue breaths. Kouwenhoven and colleagues later added the letter C to their description of closed-chest cardiac massage [3].

Styner is credited with further developing the Airway, Breathing, Circulation, Disability, and Exposure (ABCDE) approach. After a local aircraft disaster in 1976, Styner and his family were taken to a local healthcare facility, where he saw an insufficiency in the emergency treatment offered. He then founded the Advanced Trauma Life Support course, emphasizing a methodical approach to treating severely injured patients.

The ABCDE approach is universally accepted and utilized by emergency medicine clinicians, technicians, critical care specialists, and traumatologists [3]. Thus, this approach is recommended by international guidelines for suspected serious illness or underlying injury, irrespective of the diagnosis [5]. It is also the first step in post-resuscitation care after the patient achieves return of spontaneous circulation (ROSC) from a cardiac arrest [3]. This systematic approach also aims to improve coordination among the team members and saves time to make critical decisions [3].

The ABCDE approach

Since time is of the essence, the ABCDE method is a systematic approach that can be easily and quickly practiced in the ED. This is incorporated into what is known as ‘Initial patient assessment,’ one of the most crucial steps in evaluation [6]. At each step of this approach, life-threatening problems must be addressed before proceeding to the next assessment step. After the initial assessment, patients must be reassessed regularly to evaluate the treatment response. Anticipate and call for extra help early [7]. Appropriate role allocation and good communication are important for effective team working [7]. Once the patient is stabilized, a secondary survey should be conducted, which includes a thorough history, physical examination, and diagnostic testing [8]. Finally, the tertiary survey is done within 24 hours of presentation to identify any other missed injuries in trauma. Once it is recognized that the patient’s needs exceed the facility’s capabilities, the transfer process must be initiated to an appropriately specialized care center accordingly [8].

Ensure Safe Environment

Before initiating the ABCDE approach, it is essential to ensure both personal safety and a secure environment. This preparation includes addressing any potential risks, such as unexpected or violent behavior, environmental hazards, and the risk of exposure to communicable diseases. Health professionals should consider using appropriate personal protective equipment (PPE) suited to the situation, which may include gloves, gowns, masks, goggles, and thorough hand washing. These precautions are vital to protect both the healthcare provider and the patient, ensuring a safe environment for medical intervention [4].

Initiate First Response

The Resuscitation Council UK (RCUK) (2015) recommends performing a range of initial activities before proceeding with the ABCDE approach [4].

Examine the patient in general (skin color, posture, sensorium, etc.) to determine whether they seem critically ill [4].

After introducing yourself, an initial assessment can be completed in the first 10-15 seconds by asking patients their names and about their active complaints. If they respond normally, it means the airway is patent and brain perfusion is expected [9]. Check for breathing and pulse if the patient appears unconscious or has collapsed. If there is no pulse, call for help and immediately start cardiopulmonary resuscitation (CPR), adhering to local guidelines [9].

Detailed ABCDE Evaluation

Primary Survey

Patients are assessed and prioritized according to their presentations and vital signs. In primary survey, critically ill patients are managed efficiently along with resuscitation. The approach represents the sequence of steps as described below [10]:

A – Airway (with C spine control in Trauma patients)

B – Breathing and Ventilation

C – Circulation (With Hemorrhage control in active bleeding)

D – Disability

E – Exposure / Environment control

A – Airway

Airway obstruction is critical! Gain expert help immediately. If not treated, it can lead to hypoxia, causing damage to the brain, kidneys, and heart, resulting in cardiac arrest and death [4].

Airway management remains the cornerstone of resuscitation and is a specialized skill for the emergency clinician [9].

Assessment of airway patency is the first step. Can the patient talk? If yes, then the airway is patent and not in immediate danger. If not, look for the signs of airway compromise: Noisy breathing, inability to speak, presence of added sounds, stridor or wheezing, choking or gagging, cyanosis, and use of accessory muscles.

The next step is to open the mouth and look for anything obstructing the airway, such as secretions, blood, a foreign body, or mandibular/tracheal/laryngeal fractures [10].

While examining and managing the airway, great care must be taken to restrict excessive movement of the cervical spine and assume the existence of a spinal injury in cases of trauma [11].

Several critical factors can compromise a patient’s airway and must be addressed promptly in emergency settings. A depressed level of consciousness, which may result from conditions such as opioid overdose, head injury, or stroke, can impair airway protection and lead to significant risk [10]. Additionally, an inhaled foreign body, or the presence of blood, vomit, or other secretions, can obstruct the airway and necessitate immediate intervention. Fractures of the facial bones or mandible further complicate airway management due to potential structural damage. Soft tissue swelling, whether caused by anaphylaxis (angioedema) or severe infections like quinsy or necrotizing fasciitis, also seriously threatens the airway. These conditions highlight the importance of vigilant monitoring and rapid response to maintain airway patency and prevent complications.

angioedema - DermNet New Zeeland, CC BY NC ND 3.0
uvula edema - WikiMedia Commons - CC-BY-SA-3.0

Intervention: Several basic maneuvers can help maintain a clear airway. Suctioning should be performed if there are any secretions or blood present. Additionally, using the head-tilt, chin-lift, and jaw-thrust maneuvers can aid in keeping the airway open. For patients with a low Glasgow Coma Scale (GCS) score, placing an oropharyngeal or nasopharyngeal airway can be beneficial in maintaining airway patency. It’s also important to inspect the airway for any obvious obstructions; if a visible object is within reach, it may be removed carefully using a finger sweep or suction. It is crucial to remember that assistance from an anesthetist may be required in some cases. 

Head-Tilt, Chin-Lift maneuver

In trauma patients, to protect the C-spine, perform a jaw-thrust rather than a head-tilt chin-lift maneuver and immobilize the C-spine with a cervical collar [9].

A definitive airway, such as endotracheal intubation, may be necessary in patients with airway obstruction, GCS ≤ 8, severe shock or cardiac arrest, and at risk of inhalation injuries [8].

If intubation has failed or is contraindicated, a definitive airway must be established surgically [11].

B – Breathing and Ventilation

Effective ventilation relies on the proper functioning of the lungs, chest wall, and diaphragm, along with a patent airway and sufficient gas exchange to optimize oxygenation [10]. To assess breathing and ventilation, clinicians should evaluate oxygen saturation, monitor the respiratory rate for any signs of abnormality—such as rapid breathing (tachypnea), slow breathing (bradypnea), or shallow breathing (Kussmaul breathing)—and observe for increased work of breathing, such as accessory muscle use, chest retractions, or nasal flaring. Other critical assessments include checking for neck vein distention, examining the position of the trachea, chest expansion, and any injuries or tenderness, as well as auscultating for bilateral air entry and any additional sounds. Chest percussion should be performed to identify dullness, which may indicate hemothorax or effusion, or hyperresonance, suggestive of pneumothorax. Certain pathologies, like tension pneumothorax, massive hemothorax, open pneumothorax, and tracheal or bronchial injuries, can rapidly disrupt ventilation. Other conditions, including simple pneumothorax, pleural effusion, simple hemothorax, rib fractures, flail chest, and pulmonary contusion, may compromise ventilation to a lesser degree [10].

Interventions:

  • Oxygen – Ensure all patients are adequately oxygenated, with supplemental oxygen delivered to all severely injured trauma patients [11]. Place them on well-fitted oxygen reservoir masks with a flow rate > 10 L/min, which can then be titrated as needed to maintain adequate saturations. Other means of oxygen delivery (nasal catheter, nasal cannula, non-rebreather) can also be used.
  • Bag mask valve ventilation with oxygen – should be given to unconscious patients with abnormal breathing patterns (slow or shallow respiration).
  • Other interventions include salbutamol nebulizers, epinephrine, steroids, needle decompression, chest tube insertion, and the use of noninvasive ventilation and pressure support in different clinical scenarios.

C – Circulation (With Hemorrhage control in active bleeding)

Major circulatory compromise in critically ill patients can result from either blood volume loss or reduced cardiac output. In trauma cases, hypotension is assumed to be due to blood loss until proven otherwise. To assess the hemodynamic status, several key evaluations should be performed. These include checking the level of consciousness, as an altered state may indicate impaired cerebral perfusion, and assessing skin perfusion for signs like pallor, cyanosis, mottling, or flushing. Vital signs such as heart rate and blood pressure should be monitored for abnormalities like tachycardia, bradycardia, hypotension, or hypertension. Auscultation can reveal muffled heart sounds, which may suggest cardiac tamponade or pneumothorax, as well as murmurs or a pericardial friction rub that could indicate pericarditis. Checking the extremities for capillary refill and skin temperature is also essential. Additionally, palpation of the abdomen for tenderness or a pulsatile mass may reveal an abdominal aortic aneurysm, while peripheral edema, such as pedal edema, might indicate heart failure.

Interventions:

  • Two large-bore IV cannulations must be placed. If this attempt fails, intraosseous access is necessary. Hemorrhagic shock—A definitive control of bleeding along with replacement of intravascular volume is essential. Initial resuscitation should start with warm crystalloids, and blood products should be used. Massive Transfusion Protocol (MTP) should be activated according to local guidelines. In hemorrhagic shock, vasopressors and reversal of anticoagulation (if required) can be considered.
  • Hemorrhage control: External hemorrhage can be controlled by direct manual pressure over the site of the wound or tourniquet application.
  • In the case of pelvic or femur fractures, placement of pelvic binders or extremity splints may help to stabilize, although definitive management may be surgical or interventional radiological procedures.
  • Obstructive shock – Immediate pericardiocentesis for cardiac tamponade, chest tube insertion for tension pneumothorax, and thrombolysis for massive pulmonary embolism.
  • Distributive shock – intramuscular epinephrine for anaphylactic shock, empiric antibiotics for sepsis, and hydrocortisone for adrenal crisis.
  • Appropriate antihypertensives in hypertensive emergency.

D – Disability

Evaluate neurological status either with AVPU (Alert, Verbal, Pain and Unresponsive) [5] or GCS (Glasgow Coma Scale).

Evaluate for agitation, head and neck trauma, focal neurological signs (seizure, hemiplegia, etc), lateralizing signs, meningeal signs, signs of raised intracranial pressure, and pupillary examination (size and symmetry). Identify any classic toxidromes (sympathomimetic, cholinergic, anticholinergic, opioid, serotonergic, and sedative-hypnotic toxidromes). 

Choose the best response of patient
EYE OPENING
4: Spontaneously
3: To verbal command
2: To pain
1: No response
BEST VERBAL RESPONSE
5: Oriented and converses
4: Disoriented and converses
3: Inappropriate words; cries
2: Incomprehensible sounds
1: No response
BEST MOTOR RESPONSE
6: Obeys command
5: Localizes pain
4: Flexion withdrawal
3: Flexion abnormal (decorticate)
2: Extension (decerebrate)
1: No response
Glasgow Coma Score (GCS) (Modified from Teasdale, G., & Jennett, B. (1974). Assessment of coma and impaired consciousness: a practical scale. The Lancet, 304(7872), 81-84.) - Please read this article to get more insight regarding GCS.

The Glasgow Coma Scale (GCS) is a critical tool for assessing the level of consciousness in critically ill patients, providing a score based on eye, verbal, and motor responses. A GCS score ranges from 3 to 15, with lower scores indicating more severe impairment. Scores of 13-15 generally indicate mild impairment, 9-12 suggest moderate impairment, and scores of 8 or below (comatose patient) represent severe impairment and a high risk of poor outcomes. In critically ill patients, a declining GCS score can signal worsening neurological status, potentially due to factors like traumatic brain injury, hypoxia, or systemic deterioration, and often warrants immediate intervention to address underlying causes.

E – Exposure and Environmental control

It is necessary to expose the patient appropriately whilst maintaining dignity and body temperature.

Look at the skin for any signs of trauma (burns, stab wounds, gunshot wounds, etc.), rashes, infected wounds, ulcers, needle track marks, medication patches, implantable devices, tubes, catheters, and stomas; measure core body temperature, and perform logroll (trauma).

Do not forget to check frequently concealed and overlooked areas such as the genital, inguinal, perineal, axilla, back and under dressings [8].

Interventions:

  • Use specialized personal protective equipment (PPE), remove all possible triggers such as wet or contaminated clothing, and maintain core body temperature.
  • Minimize hypothermia (external rewarming, warm IV fluids) and hyperthermia (surface cooling, cold IV fluids, antipyretics for fever).

Adjuncts to primary survey

1. Electrocardiography (ECG)
2. Pulse oximetry
3. Carbon dioxide (CO2) monitoring
4. Arterial blood gas (ABG) analysis
5. Urinary catheterization (to assess for hematuria and urine output)
6. Gastric catheterization (for decompression)
7. Blood lactate level measurement
8. Chest and pelvis X-rays
9. Extended focused assessment with sonography for trauma (eFAST)

These adjuncts help provide a comprehensive evaluation of the patient’s condition [10].

Secondary Survey

After the initial primary survey and stabilization, proceed to the secondary survey. This includes a detailed history (SAMPLE)and a head-to-toe examination, including reassessment of vital signs, as there is a potential for missing an injury or other findings in an unresponsive patient [10].

The SAMPLE mnemonic is a structured approach for gathering essential patient history in emergency settings. It stands for Signs and Symptoms, Allergies, Medications, Past Medical History, Last Oral Intake, and Events leading to the illness or injury [5].

  • “Signs and Symptoms” involves asking the patient, family, or other witnesses about any observable signs or reported symptoms.
    “Allergies” are crucial to identify to prevent harm and may help recognize conditions like anaphylaxis.
  • Medications” requires a comprehensive list of all current and recent medications, including any changes in dosage.
  • Past Medical History” provides insights into underlying health conditions that may influence the current illness.
  • Last Oral Intake” is important for assessing risks of aspiration or complications if the patient requires sedation or surgery.
  • Finally, understanding the “Events” surrounding the illness or injury aids in determining its cause and severity.

Together, these components guide healthcare providers in developing a more accurate and effective treatment plan.

In the secondary survey, a thorough approach is taken to ensure comprehensive care for the patient. This includes performing relevant and appropriate diagnostic tests based on the clinical assessment to confirm diagnoses and guide further treatment. Critical, targeted treatments should be initiated promptly, along with adequate supportive care to stabilize the patient’s condition. If necessary, specialized consults are obtained to address specific medical needs. Additionally, the healthcare team must assess the need for escalation of care or consider an interfacility transfer if the patient requires more specialized resources or advanced care options [8]. This structured approach ensures that all aspects of the patient’s condition are managed effectively. 

Adjuncts to secondary survey

Additional x-rays for the spine and extremities, CT scans of the head, chest, abdomen, and spine, urography and angiography with contrast, transesophageal ultrasound, bronchoscopy, and other diagnostics [10].

If the patient starts to deteriorate, immediately go back to the ABCDE approach and reassess!

Special Patient Groups

In recent ATLS updates, the ABCDE approach has been modified to the xABCDEF approach, where “x” stands for eXsanguinating eXternal hemorrhage control and “F” stands for further factors such as special groups (pediatric, Geriatric, and Pregnancy).  While the xABCDEF approach is universal and applies to all patient groups, specific anatomic and physiological differences in different populations should be considered while evaluating and treating life-threatening conditions. Some special population groups are discussed here:

Pediatrics [10]

Children have smaller body mass but higher body surface area than their body mass and proportionately larger heads than adults. These characteristics cause children to have increased energy transfer, hypothermia, and blunt brain trauma.

A useful adjunct is the Broselow® Pediatric Emergency Tape, which helps to rapidly identify weight-based medication doses, fluid volumes, and equipment sizes.

The ABCDE approach in children should proceed in the same manner as in adults, bearing in mind the anatomical differences.

Airway – Various anatomical features in children, such as large tissues of the oropharynx (tongue, tonsils), funnel-shaped larynx, more cephalad and anteriorly placed larynx and vocal cords, and shorter length of the trachea, make assessment and management of the airway difficult. Additionally, in smaller children, there is disproportionality in size between the cranium and the midface, making the large occiput in passive flexion of the cervical spine, resulting in the posterior pharynx being displaced anteriorly. The neutral alignment of the spine can be achieved by placing a 1-inch pad below the entire torso of the infant or toddler.

The most preferred technique for orotracheal intubation is under direct vision, along with restriction of the cervical spine, to achieve a definitive airway.

Infants are more prone to bradycardia due to laryngeal stimulation during intubation than older children and adults. Hence, when drug-assisted intubation is required, the administration of atropine sulfate pretreatment must be considered. Atropine also helps to dry out oral secretions, further enhancing the view of landmarks for intubation.

When the airway cannot be maintained by bag-mask ventilation or orotracheal intubation, a rescue airway with either a laryngeal mask airway (LMA), an intubating LMA, or a needle cricothyroidotomy is required.

Red flag signs in children include stridor, excessive drooling, airway swelling, and the child’s unwillingness to move the neck. Examine the airway carefully for any foreign bodies, burns, or obstruction.

Breathing and ventilation – Children’s respiratory rates decrease with age. The normal tidal volumes in infants and children vary from 4-6 ml/kg to 6-8 ml/kg while assisting in ventilation. Care must be taken to limit pressure-related barotrauma during ventilation. It is recommended that children weighing less than 30 kg use a pediatric bag valve mask.

Injuries such as pneumothorax, hemothorax, and hemopneumothorax should be treated by pleural decompression, for tension pneumothorax, and needle decompression in the 2nd intercostal space (over the top of the third rib) at the midclavicular line. The site for chest tube insertion remains the same as in adults.

The most common cause of pediatric cardiac arrest is hypoxia, and the most common acid-base abnormality encountered is respiratory acidosis due to hypoventilation.

Circulation – Important factors in assessing and managing circulation and shock are looking for signs of circulatory compromise, ascertaining the patient’s weight and circulatory volume, gaining timely peripheral venous access, delivering an appropriate volume of fluids with or without blood replacement, evaluating the adequacy of resuscitation, and aiming for thermoregulation.

Children have increased physiological reserves. A 30% decrease in the circulating blood volume may be required for a fall in the systolic blood pressure. Hence, it is important to look for other subtle signs of blood loss, such as progressive weakening of peripheral pulses, narrow pulse pressure to less than 20 mm Hg, skin mottling (in infants and young children), cool extremities, and decreased level of consciousness.

The preferred route is peripheral venous access, but if this is unsuccessful after two attempts, intraosseous access should be obtained.

Fluid resuscitation must be commenced at 20 ml/kg boluses of isotonic crystalloids. If the patient has ongoing bleeding, packed red blood cells may be initiated at 10 ml/kg as soon as possible. Given that children have increased metabolic rates, thinner skin, and lack of substantial subcutaneous tissue, they are prone to develop hypothermia quickly, which may impede a child’s response to treatment, increase coagulation times, and affect the central nervous system (CNS) function. Therefore, overhead lamps, thermal blankets, as well as administration of warm IV fluids, blood products, and inhaled gases may be required during the initial phase of evaluation and resuscitation.

Disability – Hypoglycemia is a very common cause of altered mental state in children, and children can present with altered mental state or seizures. Check for blood glucose in children; if low, administer glucose (IV D10 or D25).

Geriatric [10]

In cases of trauma in geriatric patients, physiological events that may have led to it (e.g., cardiac dysrhythmias) must be considered. A detailed review of long-term medical conditions and medications, along with their effect on vital signs, is necessary. Risk factors for falls include physical impairments, long-term medication use, dementia, and visual, cognitive, or neurological impairments.

Elderly patients are more prone to sustaining burn injuries due to decreased reaction times, hearing and visual impairment, and inability to escape the burning structure. Burn injury remains the cause of significant mortality.

AirwayDue to loss of protective airway reflexes, airway management in the elderly can be challenging and requires a timely decision to establish a life-saving definitive airway. Opening of the mouth and cervical spine maneuvering may be challenging with arthritic changes. Loose dentures should be removed, while well-fitted dentures should be better left inside. Some patients may be edentulous, making intubation easier, but bag-mask ventilation is difficult.

While performing rapid sequence intubation, it is recommended to lower the doses of barbiturates, benzodiazepine, and other sedatives to 20% to 40% to avoid the risk of cardiovascular depression.

Breathing – Elderly patients have decreased compliance of the lungs and the chest wall, which leads to increased breathing work, placing them at a higher risk for respiratory failure. Aging also results in suppressed heart rate during hypoxia, and respiratory failure may present alongside.

Circulation – These patients may have increasing systemic vascular resistance in response to hypovolemia, given that they may have a fixed heart rate and cardiac output. Also, an acceptable blood pressure reading may truly indicate a hypotensive state, as most elderly patients have preexisting hypertension.

A systolic blood pressure of 110 mm Hg is used as a threshold for identifying hypotension in adults over 65.

Several variables, namely base deficit, serum lactate, shock index, and tissue-specific lab markers, can be used to assess for hypoperfusion. Consider early use of advanced monitoring of fluid status, such as central venous pressure (CVP), echocardiography, and bedside ultrasonography, to guide resuscitation.

Disability – Traumatic brain injury is one of the significant complications among the elderly. The dura becomes more adherent to the skull with age, which increases the risk of epidural hematoma. Moreover, these patients are commonly prescribed anticoagulant and antiplatelet medications, which puts these individuals at a higher risk of developing intracranial hemorrhage. Therefore, a very low threshold is indicated for further CT scan imaging in ruling out acute intracranial and spinal pathologies.

Exposure – Increased risk of hypothermia due to loss of subcutaneous fat, nutritional deficiencies, chronic medical illnesses, and therapies. Complications of immobility, such as pressure injuries and delirium, may develop.

Rapid evaluation and relieving from spine boards and cervical collars will help to reduce these injuries.

Pregnant [10]

Evaluation and management of pregnant individuals can be challenging due to the physiological and anatomical changes that affect nearly every organ system in the body. Therefore, knowledge of the physiological and anatomical changes during pregnancy regarding the mother and the fetus is important to provide the best and most appropriate resuscitation and care for both.  

The best initial treatment for the fetus is by providing optimal resuscitation of the mother.

Female patients in the reproductive age who present to the ED must be considered pregnant until proven by a definitive pregnancy test or ultrasound exam.

A specialized obstetrician and surgeon should be consulted early in the assessment of pregnant trauma patients; if not available, early transfer to an appropriate facility should be sought.

The uterus is an intrapelvic organ until the 12th week of gestation, around 34 to 36 weeks when it rises to the level of the costal margin. This makes the uterus and its contents more susceptible to blunt abdominal trauma, whereas the bowel remains somewhat preserved. Nevertheless, penetrating upper abdominal trauma in the late gestational period can cause complex intestinal injury due to displacement.

Amniotic fluid embolism and disseminated intravascular coagulation are significant complications of trauma in pregnancy. In the vertex presentation, the fetal head lies in the pelvis, and any fracture of the pelvis can result in fetal skull fracture or intracranial injury.

A sudden decrease in maternal intravascular volume can lead to a profound increase in uterine vascular resistance, thus reducing fetal oxygenation regardless of normal maternal vital signs.

The volume of plasma increases throughout pregnancy and peaks by 34 weeks of gestation. Physiological anemia of pregnancy occurs when there is an increase in red blood cell (RBC) volume, leading to decreased hematocrit levels. In normal, healthy pregnant individuals, blood loss of 1200 to 1500 ml can occur without showing any signs or symptoms of hypovolemia. Nonetheless, this compromise may be seen as fetal distress, indicated by an abnormal fetal heart rate on monitoring.

Leukocytosis is expected during pregnancy, peaking up to 25,000/mm3 during labor. Serum fibrinogen and other clotting factors may be mildly increased, with shorter prothrombin and partial thromboplastin times. However, bleeding and clotting times remain the same.

During late pregnancy, in a supine position, vena cava compression can cause a decrease in cardiac output by 30 % due to lesser venous return from the lower extremities.

In the third trimester of pregnancy, heart rate increases up to 10-15 beats/min than the baseline while assessing for tachycardia in response to hypovolemia. Hypertension, along with proteinuria, indicates the need to manage preeclampsia. Be mindful of eclampsia as a complication during late pregnancy, as its presentation can be similar to a head injury (seizures with hypertension, hyperreflexia, proteinuria, and peripheral edema)

An increase in the tidal volume causes increases in the minute ventilation and hypocapnia (PaCO2 of 30 mm Hg), which is common in the later gestational period. Therefore,

Maintaining adequate arterial oxygenation during resuscitation as oxygen consumption increases during pregnancy is also important.

By the seventh month of gestation, the symphysis pubis widens to about 4 to 8 mm, and sacroiliac joint spaces increase. These alterations must be kept in mind while evaluating pelvic X-ray films during trauma. Additionally, the pelvic vessels that surround the gravid uterus can become engorged, leading to large retroperitoneal hemorrhage after blunt trauma with pelvic fractures.

Every pregnant patient who has sustained major trauma must be admitted with appropriate obstetric and trauma facilities.

Pregnant individuals may present to the ED with non-obstetric causes such as intentional (intimate partner violence, suicide attempt) and unintentional trauma (MVC, fall), and obstetric causes such as ectopic pregnancy, vaginal bleed, contractions, abdominal pain, decreased fetal movement, etc.

“To optimize outcomes for the mother and fetus, assessment and resuscitation of the mother is performed first and then the fetus, before proceeding for secondary survey of the mother.”

Primary Survey - Mother

Airway – Ensure the patient has a patent and maintainable airway with adequate ventilation. In cases where intubation is necessary, maintain appropriate PaCO2 levels according to the patient’s gestational age.  Due to the superior displacement of abdominal organs and delayed gastric emptying, there is an increased risk of aspiration during intubation.

BreathingThese patients may have an increased rate of respiration due to pressure effects or hormonal changes. Pulse oximetry and arterial gas must be monitored as adjuncts. It must be remembered that normal maternal bicarbonate levels will be low to compensate for the respiratory alkalosis.

Circulation – Attempt to manually reposition the uterus towards the left side to relieve the pressure on the inferior vena cava and improve the venous return.

Since pregnant individuals have increased intravascular volumes, they can lose a large amount of blood before the onset of tachycardia, hypotension, or other signs of hypovolemia. Therefore, it is essential to remember that the fetus and the placenta are deprived of perfusion, leading to fetal distress while the maternal conditions appear stable.

Administer crystalloid IV fluids and type-specific blood. Vasopressors must be used only as a last resort to raise maternal blood pressure, as these agents can further cause a reduction of the uterine blood flow, leading to fetal hypoxia.

Primary Survey - Fetus

Leading causes of fetal demise include maternal shock and death, followed by placental abruption.

Assess for signs of abruptio placentae (vaginal bleeding, uterine tenderness, frequent uterine contractions, uterine tetany, and irritability). Another rare injury is the uterine rupture (abdominal tenderness, rigidity, guarding or rebound tenderness, abnormal fetal lie, etc.) accompanying hypovolemia and shock.

By 10 weeks of gestation, fetal heart tones can be assessed by Doppler ultrasound, and beyond 20-24 weeks of gestation, continuous fetal monitoring with a tocodynamometer must be performed. At least 6 hours of continuous monitoring in patients with no risk factors for fetal death is recommended, and 24 hours of monitoring in patients with a high risk of fetal death.

Secondary Survey

Perform the secondary survey for non-pregnant individuals, as mentioned.

An obstetrician should ideally examine the perineum, including the pelvis. The presence of amniotic fluid in the vagina, PH greater than 4.5, indicates chorioamniotic membrane rupture.

All pregnant patients with vaginal bleeding, uterine irritability, abdominal tenderness and pain, signs and symptoms of shock, fetal distress, and leakage of amniotic fluid should be admitted for further care.

All pregnant trauma patients with Rh-negative blood group must receive Rh immunoglobulin therapy unless the injury is remote from the uterus within 72 hours of injury.

Obese Patients [10]

In the setting of trauma, procedures such as intubation can be challenging and dangerous due to their anatomy. Diagnostic investigations such as E-FAST, DPL, and CT scans may also be challenging. Moreover, most of these patients have underlying cardiopulmonary diseases, which hinders their ability to compensate for the stress and injury.

Athletes [10]

Owing to their prime conditioning, they may not exhibit early signs such as tachycardia or tachypnea in shock cases. Additionally, they usually have low systolic and diastolic blood pressure.

Revisiting Your Patient

Let’s get back to the patient we discussed earlier and start assessing him:

Airway – The patient maintains his airway but finds breathing hard. Intervention: Apply 15L Oxygen via a nonrebreather mask.

Breathing—A strap mark contusion is seen with multiple bruises. His chest expansion is asymmetrical, with reduced breath sounds on the right side of his chest. There is a dull percussion note on the right lower half of his chest. He maintains oxygen saturation. Intervention: Prepare for chest tube insertion on the right side.

Circulation – Heart sounds are muffled with marked engorgement of the external jugular veins in the neck, a good pulse still palpable in his left radial, but cold clammy extremities. His pulse is 128/min, and his blood pressure is 92/50 mm Hg. Bedside ultrasound FAST (Focused Assessment Sonography in Trauma) shows a pericardial tamponade. Intervention: IV access was gained with two large-bore IV cannulas, blood was drawn for labs, the massive transfusion protocol for blood products was activated, a Foley catheter was inserted to monitor urinary output, and the surgery team was on board to plan for emergent pericardiocentesis.

Disability – Patient’s GCS remains 15, unremarkable pupillary examination and POC glucose is 7 mmol/dl.

Exposure – you notice the strap mark on his chest secondary to his seatbelt restraint, and the multiple bruises. The remaining evaluation is unremarkable, with no head, spine, abdomen, or limb injury.

Adjunct investigations – A portable chest x-ray shows increased cardiac shadow and multiple bilateral rib fractures. There is opacification in the right lung [12]. 

Discussion

This patient sustained a blunt trauma leading to pericardial tamponade and right-sided hemothorax, leading to hypovolemic shock. The most common cause of shock in a trauma patient is hypovolemic shock due to hemorrhage. However, other types of shock like cardiogenic shock (due to myocardial dysfunction), neurogenic shock (due to sympathetic dysfunction), or obstructive shock (due to tension pneumothorax, obstruction of great vessels) can occur.

Early signs of shock include tachycardia, which is the body’s attempt to preserve cardiac output and cool peripheries, and reduced capillary refill time caused by peripheral vasoconstriction. This is caused by the release of catecholamine and vasoactive hormone, which leads to increased diastolic blood pressure and reduced pulse pressure. For this reason, measuring pulse pressure rather than systolic blood pressure allows earlier detection of hypovolaemic shock, as the body can lose up to 30% of its blood volume before a drop in systolic blood pressure is appreciated.

Initiate fluid resuscitation in these patients and do not wait for them to develop hypotension.
The main aim is to maintain organ perfusion and tissue oxygenation. In children, start with crystalloid fluid boluses of 20 ml/kg, and in adults, an initial 1 L can be given. In patients who have sustained a major blood loss, consider initiating the Massive Transfusion Protocol (MTP) for blood products as soon as possible.

A few current trauma guidelines have recommended ‘permissive hypotension’ or ‘balanced resuscitation,’ where the principle is to stabilize any blood clots that may have been formed, and aggressive blood pressure resuscitation may disrupt this ‘first formed clot’ and may contribute to further hemorrhage.

To evaluate response to fluid resuscitation, assess the level of consciousness, improvement in tachycardia, skin temperature, capillary refill, and urine output (>0.5 ml/kg/hour in adults).
Besides administering packed red blood cells, do not forget to transfuse platelets, fresh frozen plasma, or cryoprecipitate, as large blood loss can develop coagulopathy in 30% of these injured patients. Tranexamic acid (TXA), an antifibrinolytic, can be utilized in addition as a 1 g bolus over 10 minutes followed by 1 g over 8 hours within 3 hours of trauma without an increased risk of thromboembolic events [11].

This systematic approach focuses on identifying and treating this hemorrhagic shock case. Bedside adjuncts such as FAST examination and portable chest X-ray can provide valuable clues to the cause of shock. A trauma CT scan is only performed once the patient is stable enough to go to the scan room.

This patient’s vital signs improve slightly but remain unstable, and blood is kept draining into the chest drain. The patient is taken to the operation theatre for an emergency thoracotomy [12].

Authors

Picture of Roxanne R. Maria

Roxanne R. Maria

Picture of Hamid A. Chatha

Hamid A. Chatha

Listen to the chapter

References

  1. Initial Assessment of Emergency Department patients, The Royal College of Emergency Medicine, Feb 2017
  2. World Health Organization. BASIC EMERGENCY CARE : Approach to the Acutely Ill and Injured.World Health Organization; 2018.
  3. Thim T. Initial assessment and treatment with the airway, breathing, circulation, disability, exposure (ABCDE) approach. International Journal of General Medicine. 2012;5(5):117-121. doi:https://doi.org/10.2147/IJGM.S28478
  4. Peate I, Brent D. Using the ABCDE Approach for All Critically Unwell Patients. British Journal of Healthcare Assistants. 2021;15(2):84-89. doi:https://doi.org/10.12968/bjha.2021.15.2.84
  5. Schoeber NHC, Linders M, Binkhorst M, et al. Healthcare professionals’ knowledge of the systematic ABCDE approach: a cross-sectional study. BMC Emergency Medicine. 2022;22(1). doi:https://doi.org/10.1186/s12873-022-00753-y
  6. Learning Objectives. https://www.moh.gov.bt/wp-content/uploads/moh-files/2017/10/Chapter-2-Emergency-Patient-Assessment.pdf
  7. Resuscitation Council UK. The ABCDE Approach. Resuscitation Council UK. Published 2021. https://www.resus.org.uk/library/abcde-approach#:~:text=Use%20the%20Airway%2C%20Breathing%2C%20Circulation
  8. Management of trauma patients – Knowledge @ AMBOSS. http://www.amboss.com. https://www.amboss.com/us/knowledge/Management_of_trauma_patients/
  9. Oxford Medical Education. ABCDE assessment. Oxford Medical Education. Published 2016. https://oxfordmedicaleducation.com/emergency-medicine/abcde-assessment/
  10. HENRY SM. ATLS Advanced Trauma Life Support 10th Edition Student Course Manual, 10e. 10th ed. AMERICAN COLLEGE OFSURGEO; 2018.
  11. Walls RM, Hockberger RS, Gausche-Hill M, Erickson TB, Wilcox SR. Rosen’s Emergency Medicine : Concepts and Clinical Practice. Elsevier; 2.
  12. Eamon Shamil, Ravi P, Mistry D. 100 Cases in Emergency Medicine and Critical Care. CRC Press; 2018.

Reviewed By

Picture of Arif Alper Cevik, MD, FEMAT, FIFEM

Arif Alper Cevik, MD, FEMAT, FIFEM

Prof Cevik is an Emergency Medicine academician at United Arab Emirates University, interested in international emergency medicine, emergency medicine education, medical education, point of care ultrasound and trauma. He is the founder and director of the International Emergency Medicine Education Project – iem-student.org, chair of the International Federation for Emergency Medicine (IFEM) core curriculum and education committee and board member of the Asian Society for Emergency Medicine and Emirati Board of Emergency Medicine.

Maxillofacial Trauma (2024)

by Maitha Ahmad Kazim & David O. Alao

You have a new patient!

A 48-year-old man was brought to the ED by ambulance shortly after sustaining blunt trauma to the face. The patient was off-loading his quad bike from a truck when it accidentally flipped over and fell directly on his face. He could not recall the incident.

Upon arrival, his vitals were BP: 144/85 mmHg, HR: 104 bpm, T: 36.8°C, RR: 23 bpm, and SPO2: 99% on room air. He was awake on the AVPU score. On examination, the patient was bleeding profusely from his nostrils, breathing from his mouth, and having diffuse facial swelling. You are concerned about the extent of the injuries sustained, and you assemble a team to manage the patient.

Importance

The significance of proficiently managing maxillofacial trauma in the fast-paced emergency medicine setting cannot be overstated. Not only do these traumas cause direct physical harm, but they also impact the patient’s appearance and their ability to perform vital functions like breathing, speaking, and chewing. Given the complex and sensitive nature of the maxillofacial region, emergency physicians must comprehensively understand how to manage such injuries effectively. Proficiency in diagnosing and managing maxillofacial trauma ensures timely and appropriate treatment and prevents potential complications and long-term sequelae. 

Epidemiology

Maxillofacial injuries are a prevalent global health concern. There were an estimated 7.5 million new facial fractures globally in 2017, with 1.8 million individuals living with a disability from a facial fracture [1]. Undoubtedly, the incidence and prevalence vary significantly from one country to another. Singaram et al. reported that the prevalence varied between countries from 17% to 69% [2]. In many regions, inadequate infrastructure, limited access to healthcare, and poor safety regulations contribute to a higher incidence of maxillofacial injuries.

Pathophysiology

Road traffic accidents, interpersonal violence, industrial accidents, and sports-related incidents are the most common etiologies of maxillofacial injuries globally. However, the predominant causes differ in developed and developing countries. Assault is the most common mechanism of injury in developed countries, while motor vehicle accident (MVA) is the most common mechanism in developing countries [3].

Low or high-impact forces can cause maxillofacial injuries. The force needed to cause damage differs from one bone to another. For instance, the zygoma and nasal bones can be damaged by low-impact forces. In contrast, the frontal bone, supraorbital rim, maxilla, and mandible are damaged by high-impact forces [4].

Furthermore, the etiology of maxillofacial trauma can predict the type of facial injuries and fractures sustained. For example, MVAs have been associated with higher instances of mandibular fractures. That is mainly due to its position compared to the rest of the facial bones and its relatively thin structure [5].

Medical History

Maxillofacial injuries often occur in association with other injuries and, thus, can be missed initially. Obtaining a systemic and thorough history can aid the diagnosis. At the initial presentation, the mnemonic “AMPLE” (Allergies, Medications currently used, Past illness/Pregnancy, Last meal, Events/Environment related to the injury) can be used to assess the patient’s pre-injury health status. Then, the following should be probed:

  • What was the mechanism of injury?
    Understanding the cause of the injury (e.g., fall, vehicle collision, assault) provides insights into potential injuries and the extent of trauma. Different mechanisms (blunt vs. penetrating, low vs. high-impact) influence the pattern and severity of injuries and aid in anticipating associated injuries.

    • Environment related to the injury
      Environmental context (e.g., construction site, sports field) can highlight additional risk factors or clues about the nature and potential complications of the injury. It may also help assess the likelihood of secondary injuries or infections.

    • Blunt vs. penetrative
      The type of trauma affects the damage pattern. Blunt trauma may result in fractures or soft tissue injuries, while penetrating trauma may involve more focal injury with a higher risk of infection and internal damage.

    • Low vs. high-impact force
      High-impact injuries are more likely to cause fractures and significant soft tissue damage. Knowing the force helps anticipate the severity and depth of injuries.

    • Direction of force
      The direction can indicate which structures might be compromised (e.g., anterior force could affect the nose, mandible, and dental structures, while lateral force may impact the zygomatic arch or TMJ).

  • Was there a loss of consciousness or an altered level of consciousness?
    Altered consciousness or loss of consciousness may indicate a head injury or neurological involvement, which necessitates further investigation and monitoring for brain injury.

  • Are there any visual disturbances?
    Vision changes can signal orbital fractures or injuries to the optic nerve, potentially affecting ocular function or indicating damage to the orbit and nearby structures.

  • Is there any change in hearing? Is the patient experiencing tinnitus or vertigo? Did they notice any discharge from the ears (clear or bloody)?
    Hearing changes, tinnitus, vertigo, or ear discharge suggest possible basilar skull fractures or damage to the auditory system, which are essential to identify to avoid long-term complications.

  • Any trouble breathing through the nose? Did they notice any discharge (clear or bloody)?
    Difficulty breathing through the nose or nasal discharge may indicate nasal fractures, airway obstruction, or cerebrospinal fluid (CSF) leakage if clear, which is critical to address in traumatic injuries.

  • Any pain while talking? Do the teeth come together normally?
    Pain when speaking or abnormal occlusion may signal fractures in the mandible, maxilla, or TMJ dislocation, impacting facial symmetry, function, and long-term outcomes.

  • Is there difficulty opening or closing the mouth? Is there any pain when biting down the teeth?
    Difficulty or pain in mouth movement often suggests mandibular fractures or TMJ injury. Restricted movement can help identify specific injury locations and aid in planning management.

  • Numbness or tingling sensation in any area of the face?
    Sensory changes suggest possible nerve damage, often related to fractures affecting the infraorbital, mental, or other facial nerves. This information helps predict potential complications and guides treatment planning.

Consider the following symptoms when obtaining a history from maxillofacial trauma patients:

  • Orbital floor fractures commonly present with symptoms such as tingling or numbness around the nose, upper lip, and maxillary gums due to infraorbital nerve damage, along with difficulty looking upward or laterally, double vision (diplopia), and pain during eye movement.
  • Nasal fractures are characterized by swelling, pain, and nosebleeds (epistaxis).
  • Nasoethmoidal fractures can cause cerebrospinal fluid (CSF) rhinorrhea, epistaxis, and tearing (epiphora) due to nasolacrimal duct obstruction.
  • Zygomaticomaxillary complex (ZMC) fractures may lead to numbness around the nose and upper lip, issues with eye movement, double vision, and difficulty opening the mouth (trismus).
  • Maxillary fractures often result in CSF rhinorrhea or epistaxis and may cause mobility in the upper teeth and gingiva.
  • Alveolar fractures are typically associated with gingival bleeding.
  • Mandibular fractures can present as painful jaw movements and tingling or numbness affecting half of the lower lip, chin, teeth, and gingiva.

Red Flags in History

Due to the complex nature of the maxillofacial region, one should be vigilant for red flags when taking history from the patients. Its proximity to the brain and central nervous system makes injuries to these very likely. Thus, identifying them early on can prevent irreversible sequelae and medicolegal implications. Red flags include memory loss, fluctuations in the level of consciousness, nausea/vomiting, and headache that does not improve with analgesia [6].

Neurological involvement can further be assessed by asking about the presence of diplopia or a change in visual acuity. Vision loss usually occurs immediately, but in 10%, symptoms are delayed [7]. Another red flag that is associated with high morbidity and mortality is cervical cord syndrome. Maxillofacial injuries associated with falls are often associated with cervical spinal injury. The patient may complain initially about neck pain or a loss of motor/sensory function in the arms [8].

Physical Examination

Maxillofacial trauma is commonly associated with polytrauma [9]. Thus, it often gets missed in examinations. Physical examination should be done systematically to ensure that all injuries are noted. Like all trauma cases, life-threatening injuries should be addressed first, and the ATLS protocol should be applied accordingly. After that, a physical examination of maxillofacial trauma would involve several key steps. Hard and soft tissue injuries (hematoma, laceration, foreign body, swelling, missing tissue, bleeding, or clear discharge) should be noted upon general inspection of the head and face. Symmetry and alignment of the face should also be noted, bearing in mind that asymmetry may be hidden by edema [10]. Facial elongation and flattening can be seen in midface fractures. Increased intercanthal distance, also known as telecanthus, indicates a nasoethmoidal injury.

Palpation of the whole face should follow, going from top to bottom to avoid missing any injury. Identify step-offs, crepitus, instability or excessive mobility, and malocclusion. Le Fort fractures, complex midface fractures, can be identified during physical examination. 

Next, a complete ocular examination should be done. Assess visual acuity, visual field, pupillary reflex, anterior chamber, and extraocular movements. An ophthalmologic consultation is recommended if any abnormalities are present [10]. The nose and septum should be inspected for any hematoma, bulging mass, or CSF leakage and palpated for any signs of fracture. The oral cavity should be inspected for palatal ecchymoses, lacerations, malocclusion, or missing teeth. Manipulate each tooth individually for movement or pain. Palpate the entire mandible for step-offs or injury. Motor and sensory functions of the face should be evaluated. A thorough cranial nerve examination will help identify sensorimotor injuries. 

Le Fort Classification

Le Fort I Fracture: A Le Fort I fracture, often referred to as a “floating palate,” is a horizontal maxillary fracture that separates the teeth from the upper face. The fracture line passes through the alveolar ridge, lateral nose, and the inferior wall of the maxillary sinus. Patients with this fracture often present with a swollen upper lip, open bite malocclusion, and ecchymosis of the hard palate. When the forehead is stabilized, the maxilla may also have noticeable mobility (including the hard palate and teeth).

Le Fort II Fracture: Known as a “floating maxilla,” the Le Fort II fracture builds upon the characteristics of Le Fort I but extends to involve the bony nasal skeleton, giving it a pyramidal shape. This fracture often leads to a widening of the intercanthal space, bilateral raccoon eyes, epistaxis, and open bite malocclusion. Physical examination may reveal mobility of the maxilla and nose, ecchymosis of the hard palate, and cerebrospinal fluid (CSF) rhinorrhea. Patients may also experience sensory deficits in the infraorbital region extending to the upper lip.

Le Fort III Fracture: Referred to as a “floating face” or “craniofacial disjunction,” the Le Fort III fracture involves a separation of the midfacial skeleton from the base of the skull. The fracture line extends from the frontozygomatic suture across the orbit and through the base of the nose and ethmoid region, running parallel with the skull base. Physical signs include bilateral raccoon eyes, ecchymosis of the hard palate, and a dish-face deformity characterized by elongation and flattening of the face. Additional signs may include enophthalmos (sunken eyes), Battle’s sign (ecchymosis over the mastoid bone), CSF rhinorrhea or otorrhea, and hemotympanum.

Red Flags in Examination

Look for “red flags” during physical examination. These red flags include cervical spine injuries, loss of teeth, Battle’s sign/Raccoon eyes with CSF rhinorrhea, and Le Fort fractures. Facial bones should not be manipulated until cervical spine injuries, which are present in 2.2% of cases, have been ruled out [11]. The oral cavity should be carefully examined for loss of teeth, as it may be aspirated during the injury. For missing teeth, a chest X-ray should be done to rule out or confirm aspiration.

Moreover, facial fractures can extend to the cranium [4]. Depending on the mechanism of injury, the patient may suffer from a concomitant base of the skull fracture, which may present with Battle’s sign and Raccoon eyes as well as CSF rhinorrhea in some cases [11]. LeFort fractures are complex fractures of the midface and are further classified into LeFort I, II, and III. These fractures are considered a red flag as they may cause airway obstruction and life-threatening bleeding [12].

Alternative Diagnoses

Given that the cause is usually known, doctors must identify the injuries sustained and the extent of injuries sustained. While blunt trauma to the face is an apparent cause of maxillofacial injuries, concomitant and alternative diagnoses should not be missed. Patients with maxillofacial trauma can present with a wide range of symptoms that are similar to those from intracranial and cervical spinal injuries.

Acing Diagnostic Testing

The diagnosis of maxillofacial injuries is not based on a single diagnostic test. It is a correlation between history, physical examination, and imaging studies. Given that the etiologies of the injury vary, the differentials are vast, and the clinical presentation differs from one patient to another. Thus, bedside testing and laboratory studies should be tailored to each patient’s clinical presentation and existing symptoms.

Bedside Testing

ECG monitoring is essential for all trauma patients. Dysrhythmias, atrial fibrillation, and ST segment changes can be seen in blunt cardiac injury. Point-of-care (POCT) glucose testing quickly assesses the patient’s glucose level. Hypoglycemia can cause confusion and an altered mental status, which are common findings in patients with maxillofacial trauma. Point-of-care blood gas testing may be beneficial in case of excessive bleeding or airway compromise. In case of tissue hypoperfusion and shock, metabolic acidosis and elevated lactate levels may be noted. Oxygen saturation and carbon dioxide should be monitored in case of midface fractures and suspected airway compromise. A POCT pregnancy test should be done in women of childbearing age, as almost all maxillofacial trauma patients require imaging for diagnosis.

Laboratory Testing

A complete blood count (CBC), particularly hemoglobin and hematocrit, is indicated when the patient is bleeding profusely. LeFort II and III have been associated with an increased risk of life-threatening hemorrhage compared to other facial fractures [12]. Therefore, blood typing and crossmatching are crucial if the patient needs a blood transfusion. A coagulation panel is done to rule out trauma-induced coagulopathy, a preventable factor for progressive brain injury and massive bleeding [13].

A CSF analysis is warranted when there are secretions from the nose or ear. Beta-2 transferrin testing is the current preferred test to confirm the presence of a CSF leak [14]. Other less used methods include beta-trace protein, double-ring sign, and glucose oxidase test. A blood ethanol test and urine toxicology screen can be considered in agitated patients or those with altered levels of consciousness.

Imaging Studies

CT scans are the “gold standard” diagnostic modality for evaluating maxillofacial trauma [15]. Using narrow-cut CT scans without contrast provides detailed cross-sectional images of the facial structures, thus allowing for a comprehensive evaluation of complex fractures. In addition to identifying facial fractures, it can detect head and cervical spinal injuries, air and fluid in the intracranial space and sinuses, periorbital injury, soft tissue injury, and embedded foreign bodies. A non-contrast head CT helps identify intracranial bleeding and distinguish between the types of bleeds if present. This is recommended, especially when the patient experiences loss of consciousness for several minutes. Because maxillofacial trauma is highly associated with cervical spine injury, the physician must have a high index of suspicion for cervical spine fractures. The NEXUS criteria is used to guide imaging in these situations. 

Plain radiographs of the head are used when CT scans are not available. They may be used to screen for fractures and provide some insight into displaced fragments, but they have low sensitivity for detecting and establishing the extent of the injuries. A chest x-ray should be done when a missing tooth is noted on physical examination, as the patient may have aspirated it.

Ultrasound is a helpful bedside diagnostic tool in any trauma patient, and it has been shown to be an accurate diagnostic method when evaluating orbital trauma [16]. It is used when an isolated orbital injury is suspected or a CT scan is not readily available. It can pick up muscle entrapment, soft-tissue herniation, and orbital emphysema.

Risk Stratification

Several risk stratification tools have been developed for maxillofacial trauma. However, these are commonly used in clinical research to assess injury severity and determine the appropriate course of action. Although no specific tool was developed for use in an emergency department, other nonspecific tools like the Glasgow Coma Scale (GCS) and NEXUS criteria come in handy. The GCS score is used to rapidly assess the patient’s level of consciousness, guiding immediate interventions. The NEXUS criteria is used to clear patients from cervical injury clinically without imaging. 

The diagnosis of maxillofacial trauma is based on a combination of clinical assessment and diagnostic imaging. A thorough evaluation of both helps predict the risk. Some common clinical factors that may contribute to poorer outcomes include severe and complex fractures, extensive soft tissue injury, high-energy trauma, open fractures, ocular injuries, and pediatric and geriatric age groups [17,18].

Management

Initial Stabilization

Treating patients with maxillofacial trauma aims to restore function and optimize appearance. However, the primary focus upon presentation is to stabilize the patient. Initial management begins with a primary survey, which constitutes the “ABCDE” approach to identify life-threatening conditions and treat them promptly. 

Airway

Airway patency is a serious concern in maxillofacial trauma, and the nature of the injury often complicates airway management. Airway compromise may be complete, partial, or progressive [9]. Early signs of airway compromise include tachypnea, inability to speak in complete sentences, and abnormal noisy breathing. Agitation and abnormal behavior may indicate hypercapnia.

If the patient has obstruction from soft tissue, perform a jaw thrust maneuver. Cervical spine injury should be presumed in all maxillofacial injury patients until proven otherwise. Therefore, avoid mobilizing the neck until it is cleared. Inspect the oral cavity for any bleeding or secretions and suction accordingly. Consider manual removal with a finger sweep or forceps if a foreign body or debris is identified. Control patients with nasopharynx or oropharynx bleeding with nasal packing or compression with gauze [19].

The need for airway protection increases with severe maxillofacial fractures, expanding neck hematoma, stridor, profuse bleeding or continuous vomiting, and unconsciousness [9]. A nasopharyngeal airway is indicated in a conscious patient without a midface trauma. If the patient was unconscious or had a midface injury, an oropharyngeal airway may help temporarily. However, a definitive airway must be secured in patients who cannot maintain airway integrity. Definitive airway control is done by an endotracheal intubation (nasal or oral). Nasal endotracheal intubation is contraindicated in a base of skull fracture. Given the area’s delicacy and complexity of the injuries sustained, fiberoptic intubation by a skilled physician may provide immediate confirmation of tracheal placement and avoid further complications [10]. If the previous methods cannot be accomplished, a surgical airway (cricothyroidotomy or tracheostomy) should be considered. 

Breathing

The patient’s breathing, ventilation, and oxygenation should be assessed. Check the alignment of the trachea and listen to the patient’s chest bilateral for air entry and added sounds. Deviated trachea and decreased air entry upon auscultation increase the likelihood of tension pneumothorax, and a needle decompression should be performed. Look for soft tissue abnormalities and subcutaneous emphysema.

The patient should be connected to a pulse oximeter to monitor adequate hemoglobin oxygen saturation. If the patient is hypoxic, they should receive oxygen supplementation. Non-invasive ventilation should precede invasive ventilation methods. However, in severe injuries, mask ventilation may be difficult due to the disrupted anatomy of the face [20].

Like all trauma patients, a “full stomach” should be presumed in patients with maxillofacial trauma as digestion stops during trauma. In addition, blood is often swallowed and accumulates in the stomach. Regurgitation and aspiration are a big risk in such patients, and evacuation of stomach content is recommended [20]. A nasogastric tube is contraindicated in a skull base fracture. An orogastric tube is recommended instead to prevent intracranial passage [21].

Circulation

Maxillofacial trauma can cause profuse bleeding that can lead to shock. Monitor blood pressure and heart rate, auscultate, and check capillary refill and hand warmth. Tachycardia precedes low blood pressure in shock. Establish bilateral IV access with two large bore cannulas and draw blood for type and crossmatch. Fluid therapy with crystalloids should be initiated. Identify the source of hemorrhage. If external or intraoral bleeding occurs, apply direct pressure, pack, and suture. Carefully examine the tongue, as persistent bleeding can obscure the airway. In the case of epistaxis, anteroposterior packing will control the bleeding in most cases [10]. Additionally, topical tranexamic acid can be used in anterior epistaxis. In cases of LeFort fractures, intermaxillary fixation might be required when packing fails to stop the bleeding [10]. If the previously mentioned measures fail, consult IR, ENT, or surgery for more advanced interventions like arterial embolization and fracture reduction [22].

Disability

The patient’s mental status and neurologic function should be assessed initially. Glucose is measured at this point if not done upon arrival. The Glasgow Coma Scale helps assess the patient’s level of consciousness. Note any change in the mental status. A brief neurological exam is recommended. 

Exposure

Expose the patient fully while keeping them warm. Look for bruises, bite marks, lacerations, and other injuries, as the etiology of maxillofacial trauma is broad and often presents as polytrauma. Decontamination might be required depending on the nature of the trauma.

Medications

Isotonic crystalloid fluids and blood products are common treatments in trauma patients. Adequate pain management should be provided with NSAIDs, opioids, or local anesthesia. There are no guidelines on the use of prophylactic antibiotics in maxillofacial trauma. Nonetheless, there are specific scenarios where prophylactic antibiotics administration is recommended. Depending on the type of injury sustained, additional medications might be required. Refer to Table to explore the additional medications used in the setting of maxillofacial trauma:

Drug name (Generic)

Potential Use

Dose

Frequency

Cautions / Comments

Acetaminophen

mild-moderate pain (can be given with NSAIDs, with or without Opioids)

325-1,000 mg PO

 

Max Dose: 4 g daily

q4-6h

  • Ask for allergies
  • Ask for if/when they took Acetaminophen at home

Ibuprofen

mild-moderate pain (can be given with Acetaminophen)

600 mg PO

 

Max Dose: 3,200 mg daily

q6h

  • Can cause GI upset and increase risk of GI bleed
  • Renal insufficiency

Hydromorphone

Moderate – severe pain

0.5-4 mg IV/IM/SC

 

Max Dose: n/a

q4-6h

  • Risk of respiratory depression
  • Risk of addiction and abuse

Morphine sulfate

Moderate – severe pain

2.5-10 mg IV/IM/SC

 

Max Dose: n/a

q2-6h

  • Risk of respiratory depression
  • Risk of addiction and abuse
  • Hypotension

Metoclopramide

Nausea and vomiting (to prevent risk of aspiration)

1 to 2 mg/kg/dose IV

 

Max Dose: n/a

Every 2 hours for the first two doses, then every 3 hours for the subsequent doses.

  • Extrapyramidal side effects
  • If acute dystonic reactions occur, 50 mg of diphenhydramine may be injected IM.

Ondansetron

Nausea and vomiting (to prevent risk of aspiration)

0.15 mg/kg IV (not to exceed 16 mg)

 

Max Dose: n/a

q8hr PRN

  • Increased risk of QT prolongation, which increases the risk of cardiac arrhythmia and cardiac arrest.

Amoxicillin-clavulanic acid

Nasal packing (ppx for epistaxis – TSS)

 

Facial fractures communicating with open wounds of the skin

 

Mandibular fractures that extend into the oral cavity

2g PO (extended-release tablets)

 

Max Dose: n/a

q12h (7 days)

  • Ask for allergies
  • Ask if they have taken any antibiotic recently.
  • Hives and skin rash

Procedures

Epistaxis: Epistaxis is a common issue in maxillofacial trauma due to damage to the nasal structures and blood vessels. Managing epistaxis is crucial to prevent blood loss and ensure the airway remains clear. For anterior epistaxis, anterior nasal packing can effectively apply pressure to stop the bleeding. If the bleeding source is posterior, posterior nasal packing using a balloon catheter or Foley’s catheter may be necessary. These techniques help control bleeding and stabilize the patient, especially in cases where blood loss might obstruct the airway or lead to hemodynamic instability.

Inability to Protect Airway: In cases of severe maxillofacial trauma, there may be a risk of airway compromise due to swelling, bleeding, or physical obstruction from broken facial structures. If a patient cannot protect their airway, endotracheal intubation is required to secure it and maintain ventilation. Intubation provides a definitive airway, bypassing obstructions and ensuring adequate oxygenation, which is critical in trauma patients to prevent hypoxia and support life-sustaining measures.

Failed Intubation: Occasionally, intubation may be unsuccessful, particularly in patients with extensive facial injuries or anatomical challenges. In such cases, a cricothyroidotomy is performed. This emergency surgical procedure creates an opening in the cricothyroid membrane, providing an alternative airway route directly into the trachea. Cricothyroidotomy is a life-saving measure when intubation fails, ensuring oxygen can still be delivered to the lungs when other methods are ineffective.

Tension Pneumothorax: Maxillofacial trauma can sometimes be associated with thoracic injuries, leading to complications like tension pneumothorax, where air is trapped in the pleural cavity and compresses the lungs and heart, causing a life-threatening situation. Needle decompression is the first step in relieving the pressure by inserting a needle into the pleural space to allow trapped air to escape. This is followed by a tube thoracostomy (chest tube placement) to maintain the release of air and prevent the recurrence of tension pneumothorax. This procedure is essential to restore normal lung function and stabilize the patient’s respiratory status.

Special Patient Groups

Pediatrics

Pediatric patients’ anatomical and developmental differences should be considered when evaluating them for maxillofacial trauma. An infant’s frontal bone dents, while a child’s frontal bone experiences a depressed fracture under a force that causes facial fractures in adults [4]. Smaller force loads are needed to damage the facial bones than adults [4]. Given pediatric patients’ underdeveloped facial skeletons and sinuses, growth dysplasia is a common outcome of suboptimal treatment. Standard facial radiographs often miss fractures; a CT scan is more reliable in this age group [23]. Assess for orbital fracture thoroughly, as children’s orbital floor is pliable, increasing the risk of entrapment and rectus muscle ischemia [6].

Geriatrics

The impaired physiologic response and frailty of geriatric patients make their treatment more challenging. Although they are subject to the same mechanism of maxillofacial trauma as the other age groups, their response to the injuries differ. They are at a high risk of intracranial hemorrhage, but their basal vital signs often do not reflect signs of hemorrhage or hypoperfusion, making diagnosing shock difficult. Comorbidities and polypharmacy in this age group further mask the normal shock response. In addition, the likelihood of associated injuries in this group is high [24]. Elderly patients were reported to have more frequent cerebral concussions and internal organ injuries [25]. Nonetheless, a GCS of <15 has also been associated with higher mortality rates, especially in those older than 70 years [25]. Putting all of this into perspective when assessing elderly patients, a lower threshold for extensive investigations and referral is necessary.

When to admit this patient

Definitive repair of facial fractures is not a surgical emergency, and patients can be discharged home with a close follow-up in the clinic in most cases. An awake patient with good home care and isolated stable injuries (i.e., mandibular or nasal fracture) may be discharged home. However, admission should be considered in a number of situations. These include severe complex facial fractures, open fractures, the presence of comorbidities, and cases of associated injuries that need close monitoring. Admission is made to the intensive care unit or a surgical ward with a high level of monitoring.

Revisiting Your Patient

A 48-year-old male was brought to the ED by ambulance shortly after sustaining blunt trauma to the face. The patient was loading his quad bike off a truck when it accidentally flipped over and fell directly on his face and upper body. He could not recall what happened thereafter.
Upon arrival, his vitals were BP: 144/85 mmHg, HR: 104 bpm, T: 36.8°C, RR: 23 bpm, and SPO2: 99% on room air. He was awake on the AVPU score. On examination, the patient was bleeding profusely from his nose, breathing from his mouth, and having diffuse facial swelling. You are concerned about the extent of injuries sustained and have assembled your team to manage the patient adequately.

History was taken from his brother, who witnessed the incident. The brother confirmed that the patient had no LOC, dizziness, or vomiting but reported that the patient kept complaining of neck pain. He is known to have L5-S1 disc prolapse, does not take any medication, and has no known allergies.

You worry that the patient might suffer from airway compromise and quickly begin your primary survey. You hear gurgling noises and check the patient’s mouth to find it filled with blood. You suction and look for sources of bleeding in the mouth but find none. The airway becomes patent. You notice that EMS has placed a C-spine collar on the patient already. His lungs are clear bilaterally, and you insert an orogastric tube to suction his stomach contents. He is bleeding profusely from his nostrils, so you pack his nose anteriorly. This does not stop the bleeding, and the patient is spitting out blood. You then apply topical tranexamic acid and more packs, and the bleeding stops. His pulses are present, extremities are warm, and capillary refill time is less than 2 seconds. His GCS is 15/15, and his pupils are reactive to light. Upon exposing him, you notice lacerations on his lips and ears but no other injuries on the rest of his body.

Two large bore IV lines are inserted peripherally, blood is drawn for laboratory investigations, and intravenous normal saline is administered immediately. A 12-lead ECG demonstrated sinus tachycardia. You perform a bedside E-FAST to rule out pneumothorax/hemothorax, pericardial fluid, and peritoneal fluid. You ask for urgent CT scans, including a CT Head and Neck without contrast and a Maxillofacial CT. The CT scan report confirms no C-spine fractures, skull fractures, or brain injury. However, it identifies a Le Fort 1 fracture and fracture involving the right orbital wall. You safely remove the c-spine collar. You consult the Oral and Maxillofacial surgeon and the Ophthalmologist, and both agree to see the patient. You give the patient morphine to alleviate his pain.

You performed a secondary survey to ensure the patient was not deteriorating and to identify any additional injuries. The patient remained stable, and he was admitted to the surgical floor.

Figure: Fracture of the lateral wall left maxilla (long arrow) and a tripod fracture of the right zygoma (short arrows).

Author

Picture of Maitha Ahmad Kazim

Maitha Ahmad Kazim

Dr. Maitha Ahmad Kazim is an Emergency Medicine Resident at Dubai Health, recognized for her dedication in patient care and medical research. She earned her Doctor of Medicine degree from the United Arab Emirates University, where she graduated with distinction. Dr. Kazim is known for her commitment to advancing emergency care, demonstrated by her active engagement in research, mentorship, and medical education.

Picture of David O. Alao

David O. Alao

David is a senior consultant in emergency medicine and associate professor of medicine College of Medicine and Health Sciences, United Arab Emirates University, Al Ain, UAE
He graduated from the University of Ibadan, Nigeria. After initial training in general surgery in Leeds and Newcastle Upon-Tyne, United Kingdom, he had higher specialist training in emergency medicine in the South West of England.
He was a consultant in emergency medicine for 15 years at the University Hospitals Plymouth, United Kingdom where he was a Clinical Tutor, Academic Tutor and, Assistant professor at Plymouth University Peninsular School of Medicine and Dentistry (PUPSMD) UK.
David is a fellow of the Royal College of Surgeons of Edinburgh and the Royal College of Emergency Medicine UK.
His interests are undergraduate and postgraduate medical education, skills training and transfer, trauma systems development and resuscitation science. He has published over 30 papers in peer-reviewed journal.

Listen to the chapter

References

  1. Lalloo R, Lucchesi LR, Bisignano C, et al. Epidemiology of facial fractures: incidence, prevalence and years lived with disability estimates from the Global Burden of Disease 2017 study. Inj Prev. 2020;26(Supp 1):i27-i35. doi:10.1136/injuryprev-2019-043297
  2. Singaram M, G SV, Udhayakumar RK. Prevalence, pattern, etiology, and management of maxillofacial trauma in a developing country: a retrospective study. J Korean Assoc Oral Maxillofac Surg. 2016;42(4):174. doi:10.5125/jkaoms.2016.42.4.174
  3. Nalliah RP, Allareddy V, Kim MK, Venugopalan SR, Gajendrareddy P, Allareddy V. Economics of facial fracture reductions in the United States over 12 months. Dent Traumatol Off Publ Int Assoc Dent Traumatol. 2013;29(2):115-120. doi:10.1111/j.1600-9657.2012.01137.x
  4. Pappachan B, Alexander M. Biomechanics of Cranio-Maxillofacial Trauma. J Maxillofac Oral Surg. 2012;11(2):224-230. doi:10.1007/s12663-011-0289-7
  5. Sharifi F, Department of Oral & Maxillofacial Surgery, Mashhad University of Medical Sciences, Mashhad, Iran., Samieirad S, et al. The Causes and Prevalence of Maxillofacial Fractures in Iran: A Systematic Review. WORLD J Plast Surg. 2023;12(1):3-11. doi:10.52547/wjps.12.1.3
  6. Van Gijn D. Tips for GP trainees working in oral and maxillofacial surgery. Br J Gen Pract. 2012;62(594):50-51. doi:10.3399/bjgp12X616490
  7. Lynham A, Tuckett J, Warnke P. Maxillofacial trauma. Aust Fam Physician. 2012;41(4):172-180.
  8. Philip MR, Soumithran CS. Prevalence of Neurologic Deficits in Combined Facial and Cervical Spine Injuries: A Retrospective Analysis. Craniomaxillofacial Trauma Reconstr. 2021;14(1):49-55. doi:10.1177/1943387520940182
  9. Saigal S, Khan MM. Primary Assessment and Care in Maxillofacial Trauma. Oral and Maxillofacial Surgery for the Clinician. 2021:983-995. doi:10.1007/978-981-15-1346-6_48
  10. Truong T. Initial Assessment and Evaluation of Traumatic Facial Injuries. Semin Plast Surg. 2017;31(02):069-072. doi:10.1055/s-0037-1601370
  11. Mukherjee S, Abhinav K, Revington P. A review of cervical spine injury associated with maxillofacial trauma at a UK tertiary referral centre. Ann R Coll Surg Engl. 2015;97(1):66-72. doi:10.1308/003588414X14055925059633
  12. Patel BC, Wright T, Waseem M. Le Fort Fractures. In: StatPearls. StatPearls Publishing; 2023. Accessed August 12, 2023. http://www.ncbi.nlm.nih.gov/books/NBK526060/
  13. Peng N, Su L. Progresses in understanding trauma-induced coagulopathy and the underlying mechanism. Chin J Traumatol. 2017;20(3):133-136. doi:10.1016/j.cjtee.2017.03.002
  14. Das D, Salazar L. Maxillofacial Trauma: Managing Potentially Dangerous And Disfiguring Complex Injuries. Emerg Med Pract. 2017;19(4):1-24.
  15. Meara DJ. Diagnostic Imaging of the Maxillofacial Trauma Patient. Atlas Oral Maxillofac Surg Clin North Am. 2019;27(2):119-126. doi:10.1016/j.cxom.2019.05.004
  16. Forrest CR, Lata AC, Marcuzzi DW, Bailey MH. The role of orbital ultrasound in the diagnosis of orbital fractures. Plast Reconstr Surg. 1993;92(1):28-34. doi:10.1097/00006534-199307000-00004
  17. Sharma R, Parashar A. Unfavourable outcomes in maxillofacial injuries: How to avoid and manage. Indian J Plast Surg. 2013;46(2):221. doi:10.4103/0970-0358.118597
  18. Krausz AA, Krausz MM, Picetti E. Maxillofacial and neck trauma: a damage control approach. World J Emerg Surg. 2015;10(1):31. doi:10.1186/s13017-015-0022-9
  19. Hutchison I, Lawlor M, Skinner D. ABC of major trauma. Major maxillofacial injuries. BMJ. 1990;301(6752):595-599. doi:10.1136/bmj.301.6752.595
  20. Barak M, Bahouth H, Leiser Y, Abu El-Naaj I. Airway Management of the Patient with Maxillofacial Trauma: Review of the Literature and Suggested Clinical Approach. BioMed Res Int. 2015;2015:724032. doi:10.1155/2015/724032
  21. Spurrier E, Johnston A. Use of Nasogastric Tubes in Trauma Patients – A Review. J R Army Med Corps. 2008;154(1):10-13. doi:10.1136/jramc-154-01-04
  22. Jose A, Nagori S, Agarwal B, Bhutia O, Roychoudhury A. Management of maxillofacial trauma in emergency: An update of challenges and controversies. J Emerg Trauma Shock. 2016;9(2):73. doi:10.4103/0974-2700.179456
  23. Stewart C, Fiechtl JF, Wolf SJ. Maxillofacial trauma: Challenges in ED diagnosis and management. Emerg Med Pract. 2008;10(2):1-18.
  24. Shumate R, Portnof J, Amundson M, Dierks E, Batdorf R, Hardigan P. Recommendations for Care of Geriatric Maxillofacial Trauma Patients Following a Retrospective 10-Year Multicenter Review. J Oral Maxillofac Surg. 2018;76(9):1931-1936. doi:10.1016/j.joms.2017.10.019
  25. Kokko LL, Puolakkainen T, Suominen A, Snäll J, Thorén H. Are the Elderly With Maxillofacial Injuries at Increased Risk of Associated Injuries?. J Oral Maxillofac Surg. 2022;80(8):1354-1360. doi:10.1016/j.joms.2022.04.018

Reviewed By

Picture of Arif Alper Cevik, MD, FEMAT, FIFEM

Arif Alper Cevik, MD, FEMAT, FIFEM

Prof Cevik is an Emergency Medicine academician at United Arab Emirates University, interested in international emergency medicine, emergency medicine education, medical education, point of care ultrasound and trauma. He is the founder and director of the International Emergency Medicine Education Project – iem-student.org, chair of the International Federation for Emergency Medicine (IFEM) core curriculum and education committee and board member of the Asian Society for Emergency Medicine and Emirati Board of Emergency Medicine.

Disaster Medicine Triage (2024)

Disaster Medicine Triage

by Parker Maddox, Hassan Khuram & Scott Goldstein

A Brief Introduction to Disaster Medicine

Disaster Medicine is a medical discipline that centers around events significant enough that, without external aid, emergency departments (ED) would not be able to adequately provide care to those affected [1]. For example, a multi-vehicle car accident may result in multiple casualties, but would not be considered a disaster if the responding medical infrastructure was able to handle the influx of patients. Many entities including the United Nations International Strategy for Disaster Reduction (UNISDR) and the World Health Organization (WHO) have amended their definitions of the term “disaster” to specify occurrences that exceed a community’s ability to cope with the effects of said disaster [2,3]. Incidents that have satisfied this requirement include natural disasters and more modern occurrences such as water contamination, human conflict, chemical spills, radiation, power outages resulting in infrastructure collapse, etc. [1]. The field of disaster medicine operates with the goal of aiding with these disaster-level events and all of the associated medical, logistical, and ethical issues that come along with them [4].

This is accomplished through a joint effort by many disciplines within healthcare including emergency medicine (EM), Emergency Medical Services, pediatrics, public health, social work, and many more [5]. However, emergency medicine providers’ experience in high acuity, large patient volume settings set them up to take a leadership role in disaster medicine. The emergency department is also typically the first point of contact between patients and health care providers in a disaster setting providing emergency medicine physicians an advantage in these responses [6]. In addition to their traditional responsibilities of stabilization and disposition of disaster patients, EM physicians are typically delegated the responsibilities of coordinating disaster response as chief medical officer, interfacing with government and community relief efforts, and directing disaster triage efforts [6].  

Disaster Triage

When a disaster occurs, hospital systems experience what is known as a critical care surge. This is defined as “any increase in the number of critically ill or injured patients beyond the baseline rate a hospital or critical care unit usually experiences.” [7]. These surges are classified based on the size of the critical patient increase with disasters typically causing large surges or megasurges. Megasurges are caused by grand scale, unexpected disaster events (tsunamis, earthquakes, terrorist attacks, etc.) and can require greater than 200% of the resource capacity a hospital has to care for patients [7]. This dramatic scarcity of resources in the face of overwhelming casualties results in an inevitable need to ration resources.
 

According to the World Health Organization, “triage” is the rapid examination and sorting of patients into groups according to their medical needs and the availability of resources [8]. In this setting of mass casualties and resource scarcity, disaster triage protocols are governed by the utilitarian concept of doing the greatest good for the greatest number[9]. As opposed to routine triage seen in the ED, this requires a shift of focus from the outcomes of single patients to outcomes on a population level [7].

However, despite the common misconception, disaster triage is not simply making the decision of whether to treat a patient or not. Disaster triage is more complex with most decisions centering around what level of treatment a patient should receive [7]. Rarely is it decided or even proposed that a patient should receive no treatment at all [7]. The consensus within disaster medicine is that disaster triage should optimize patient care and resource allocation by considering the incremental improvement in survival that a treatment would provide a patient in their current condition[10]. In order to make these decisions, the foundation of disaster triage lies in the use of triage tools and protocols to systematically assess patient conditions and prognoses following these devastating events. With proper triage and resultant treatment, it has been shown that trauma patients can experience at least a 25% reduction in mortality [11]. In disaster settings where hundreds to thousands of lives hang in the balance, 25% is not an insignificant number.

Measures of Success: Undertriage and Overtriage

Prior to discussing the wide array of triage methods used in disasters, it is pertinent to describe the terms used describe and assess them. Triage efficacy is typically judged by its validity, or how accurate the acuity assigned during triage, by tool or clinical assessment, is to the actual acuity of the patient. In order to measure this validity, we rely on rates of undertriage and overtriage observed during real world scenarios (Table 1) [12].

Concept

Definition

Impact

Sensitivity & Specificity

Acceptable Rate

Undertriage

Patient condition is classified as a lower acuity than it actually is.

Patients are under prioritized, under treated, and providers may miss savable lives.

Low sensitivity

Low (<5-10% of patients)

Overtriage

Patient condition is classified as a higher acuity than it actually is.

Patients are overtreated leading to disorganization, misallocation of scarce resources, and wasting time that could be used to save other patients.

Low specificity

Variable depending on context (25-35% and others 50-60%)

Table 1:  Definitions, major impacts, relationships to sensitivity and specificity, and acceptable rates of overtriage and undertriage [11, 12, 13, 15, 17].

Undertriage

Undertriage is when a patient’s condition or injury is under classified in terms of acuity, and the patient is under prioritized or under treated as a result. This results in situations where critically ill patients could have benefited from a justifiable use of resources to further evaluate and treat their injuries [13, 14]. An example of this could be a severely injured patient that was transferred to a non-trauma center, or a patient that could have survived with prioritized evacuation and admission to the ICU or OR. However, it is worth noting that moderately injured patients are the most often undertriaged since severe acuity patients are more easily recognized [11]. Nonetheless, these are essentially missed opportunities to save patients and, statistically, this would represent a low sensitivity of the triage process. Therefore, there is a low tolerance for undertriage with most entities recommending an acceptable undertriage level of less than 5-10% of patients [15].

Overtriage

Overtriage is defined as the inaccurate classification of a patient as high priority or acuity when their injuries are actually non-urgent [11].  A classic example of overtriage would be when a stable, non-critical patient is unnecessarily expedited ahead of sicker patients for a surgery they do not acutely require. According to Foley and Reisner, another form of overtriage is when patients with little to no chance of surviving receive aggressive medical treatment, inappropriately allocating scarce resources [13]. Compared to undertriage, overtriage represents a poor specificity and complicates triage by creating disorganization that misallocates time and resources [14,16]. In the setting of a disaster, overtriage can be just as dangerous as undertriage as this misallocation can take away scarce resources and time that could be used to save other patients. The acceptable level of overtriage is more debated, with some sources recommending 25-35% [15] and others 50-60% of patients [13].

The reason for less stringency surrounding acceptable overtriage rates is due to the influence the two rates have on each other. Overtriage and undertriage do not exist independently of one another and demonstrate an inverse relationship [13]. As overtriage rates increase and patients are treated more liberally, undertriage rates decrease as less savable patients are missed. Therefore, when faced with the decision of overallocating resources or missing a potential savable life, most entities recommend maintaining a higher allowance for overtriage in order to reduce undertriage. Previously, it was reported that 50% of overtriage was required to reduce the rate of undertriage to 0% [17], but more recently that recommendation has been decreased to 25-35% [15] due to the detrimental effects higher overtriage can have on triage efforts with minimal decrease in undertriage rates [11, 18].

However, when applying these rules to real life disaster triage, it is important to also factor in the environment and working conditions of the disaster. The amount of acceptable overtriage and undertriage in a mass casualty event should change depending on resources and casualties present [13]. For example, in a scenario where casualties are few and resources are plenty, the risks of raising the overtriage rate are outweighed by the benefit of possibly reducing undertriage as there would be little chance of running out of resources or time. On the other hand, in a grand scale disaster where resources are extremely limited, a lower overtriage rate would be acceptable, despite the possible increase in undertriage, due to the higher likelihood of running out of resources and time. Therefore, it is imperative in disaster triage to always perform an initial survey of the amount of casualties and resources available in order to form an educated plan to maximize the greatest benefit for the greatest number of people [19].

Primary, Secondary, and Tertiary Triage

In comparison to traditional triage in an ED, disaster victims are triaged multiple times throughout their medical course beginning at the site of the incident and continuing through possible admission to the ICU or OR. These multiple points of reevaluation account for the evolving nature of disasters over time in terms of resource availability, treatment delays, and injury progression or resolution [20]. Triage in the setting of mass casualties can be broken down into three different types: primary, secondary, and tertiary triage (Table 2). These different classifications differ based on the triage timing, location, and what level of care is being addressed [19, 21, 22].

Primary Triage

Primary triage occurs at first contact with patients after a disaster [22]. This can occur at the scene of the disaster, or any other setting outside of the hospital including an area away from the incident [19]. The goal of primary triage is to establish the priority of injured patients for on-site treatment and evacuation to the nearest available hospital [21]. This can also include decisions such as routing patients to trauma versus non-trauma centers or performing life saving measures in the field [7, 19]. Primary triage is regarded as the most critical stage in the disaster triage process with the greatest potential to save lives and influence population outcomes [21, 23]. As a result, many different tools and methods for primary triage have been developed and primary triage will be the major focus of this chapter [14]. The tools and specifics of primary triage will be discussed in more detail in the Triage Tools section.

Secondary Triage

Secondary triage is the second evaluation of patients’ condition and overall acuity. This occurs upon patient arrival to the hospital and commonly takes place in the emergency department [21]. However, the level of care being addressed in secondary triage changes depending on the context of the disaster and the resulting bottlenecks in ED patient care. If it is a mass casualty event involving patients with highly acute complaints such as trauma, then secondary triage will focus on prioritizing patients for initial stabilizing measures in the ED [7]. Alternatively, if the disaster takes a less acute and more extended course, such as the Coronavirus Disease 2019 pandemic, then secondary triage will focus on determining disposition of patients from the ED after the initial stabilization has been performed. The disposition prioritization can include what patients are admitted to the hospital, transferred to more specialized areas within the ED, or discharged home [19, 21]. Secondary triage is especially vital when evacuation from the site of a disaster is prolonged resulting in a large influx of deteriorating patients arriving to the hospital [22].

Tertiary Triage

Tertiary triage is the third evaluation of patients involved in a disaster taking place after initial stabilization or hospital admission. This also occurs within the hospital, but does not need to be in the ED [19]. Tertiary triage is when questions of definitive care are addressed and prioritized such as ICU admission, surgery, and other procedures including those performed by interventional radiology [21]. This final form of triage is typically performed by a physician with critical care training or a surgeon [22]. This is also a chance to reassess continued medical management of severe patients in the setting of dwindling resources such as continuing life support measures or additional treatment after poor prognostic laparoscopic findings [19]. In disasters with fewer critical patients with life threatening injuries, tertiary triage is less utilized [7].

Triage

Timing

Location

Level of Care (LOC)

Primary Triage

First patient contact

At the scene of the disaster

Determines the priority order for treatment in the field and emergency transport. Can also prioritize patients to be transported to trauma vs. non-trauma centers.

Secondary Triage

Second evaluation upon entry into the hospital

In the emergency department

Determines priority order for resuscitation in the ED and disposition after stabilization

Tertiary Triage

After initial treatment and stabilization in the ED

Anywhere in the hospital, not restricted to the ED

Determines the priority order for definitive care including ICU admission, surgery, or transfer to a higher-level facility,

Table 2: Definitions, timing, location, and level of care being decided on for the three levels of triage: primary, secondary, and tertiary [19, 21, 22].

How to Triage: Primary Triage Classification & Tools

Classification

How to properly perform primary triage in the field of a disaster has been a highly contentious area of research since before the establishment of disaster medicine [14]. In non-disaster level traumas, most medical providers in North America minimize their pre-hospital evaluation and treatment in favor of more expedient transportation to the hospital[24]. This is known as “scoop and run” and restricts pre-hospital treatment to Basic Life Support (BLS) with minimal classification of patient acuity. “Scoop and run” has been proven to have significant benefits in terms of trauma outcomes[24, 25], but more rigid triage systems are typically utilized in disasters to expedite patient prioritization, minimize uncertainty, and maximize effective use of resources to do the “greatest good for the greatest number” [26].

Globally, differing disaster triage tools and systems are implemented without any clear consensus in the literature on their efficacy [21]. However, one commonality among the majority of these systems is the four-level classification schema they use to group and prioritize disaster victims [26, 27]. First proposed by the World Medical Association (WMA), this system categorizes disaster victims into four different groups based on their acuity and how urgently they require medical intervention (Table 3) [26]. The literature labels these groups in various manners including by triage tag color (red, yellow, green, black), urgency of required treatment (emergency/immediate, delayed, minimal, expectant), or their priority level (P1, P2, P3, P4) [13, 21, 26, 28].

The immediate/emergency group consists of patients who are in critical condition, but can still be saved with immediate treatment within, at most, the next few hours. A red triage tag is commonly used to label this group [28].

The delayed patients are those who are not experiencing an imminent threat to their life, but urgent, definitive medical care will be required at some point. These patients are often labeled with a yellow triage tag [26].

The minimal group are sometimes referred to as the “walking well” or “walking wounded” and have the least severe injuries [27, 29]. These nicknames stem from the fact that most patients in the minimal group can walk following the incident. Various triage tools will even use the ability to ambulate in their algorithms since multiple studies have shown that walking following a disaster is a strong indicator of a relatively low risk patient with a good prognosis [22, 30]. These patients are labeled with a green triage tag and often require only minimal treatment that can be delayed until the rest of the patient categories have been treated.

The final category, deemed expectant, is marked with a black triage tag, and is made up of patients that are either dead or critically ill to the point that efforts to save them are deemed futile. This classification is typically made in situations where the patient’s condition is beyond treatment or when the complex treatment required to save the patient would be putting other patients at risk by misallocating already limited time and resources [21, 26]. It is important to acknowledge that this can be an especially difficult classification for providers to make. The WMA has addressed this by releasing a statement expressing that “It is unethical for a physician to persist, at all costs, at maintaining the life of a patient beyond hope, thereby wasting to no avail scarce resources needed elsewhere.[31]” However, as disasters are fluid and ever-evolving, it is the responsibility of a triage to repeatedly reassess the situation including reviewing the priority designations previously assigned to patients [28].

Table 3 Priority number, color, name, and description for the four commonly used triage tags [21, 26, 28, 30].

Table 3: Priority number, color, name, and description for the four commonly used triage tags [21, 26, 28, 30].

Tools

Despite a consensus on what priority groupings are used to sort patients during a disaster, how to place patients in each grouping is still highly divisive. The literature has been unable to provide any significant evidence for or against specific triage strategies resulting in a wide array of disaster triage systems used internationally [14, 21, 22]. In the 1980’s, formal triage scoring systems were developed for primary triage that categorized patients based on objective measures. The most used scoring systems are the Revised Trauma Score (RTS) and Champion’s Trauma Score (CTS), both of which utilize a patient’s Glasgow Coma Scale (GCS), systolic blood pressure, and respiratory rate to calculate a total score that sorts patients into the appropriate priority groupings [32]. However, triage scoring systems have been shown to not be as efficacious in the pre-hospital setting since objective measures of vital signs do not always correlate with clinical condition. As a result, triage scores have demonstrated poor sensitivity in the field and there have been instances where normal vital signs masked critical illness in disaster patients resulting in undertriage [13, 32]. Additionally, vital signs taken at the scene of a disaster are not always reliable due to various confounding variables and can create provider uncertainty in the field [32]. Therefore, triage scoring systems have fallen out of favor in disaster triage and this chapter will focus on the use of multi-tier triage algorithms.

Formalized triage algorithms are a set of rigid, pre-determined decision trees that quickly guide providers through the initial assessment of disaster victims in the field [14]. Triage algorithms base their decision making more in components of clinical presentation such as ability to ambulate and breathe rather than objective measures. These algorithms tend to be more suitable for mass casualty disasters as they minimize the time spent making active decisions and are easy to learn in a restricted amount of time [13]. The disadvantage of these algorithms is their lack of flexibility. As discussed previously in the Measures of Success: Undertriage and Overtriage section, it is important to be able to tailor your protocol, and subsequently your over and undertriage rates, depending on the number of casualties and the availability of resources. However, the rigid procedure of these algorithms does not allow for modifications of treatment criteria when time and resources are more plentiful [13]. Many algorithms have been developed with slightly different applications based on patient demographics, mechanism of the disaster, geography, etc. [14]. Due to the sheer number of triage algorithms currently available, this chapter will focus on the most used primary triage tools in disaster medicine: the Simple Triage and Rapid Treatment (START) and Sort, Assess, Lifesaving interventions, Treatment/Transport (SALT) algorithms.

The Simple Triage and Rapid Treatment (START) triage algorithm was originally developed as a result of joint efforts between a California Fire department, Marine department, and medical providers in 1983 [33].  This was one of the first triage systems developed outside of the military and, following its conception, the Domestic Preparedness Program of the Department of Defense made it standard practice in disaster events [28]. It is now the most prolific mass casualty triage system used in the United States [27].

The START triage algorithm was designed as an expedient triage system that would be easily teachable to emergency providers with minimal training [26]. The objective of the system is the be able to evaluate patients older than eight years old within 30-60 seconds and triage them into one of the four priority groupings discussed previously: immediate/emergency (red), delayed (yellow), minimal (green), expectant (black) [14, 27]. This is accomplished through strict criteria looking at patient ambulation, respiratory rate, radial pulse, mental status, and capillary refill, though many versions of START no longer assess capillary refill due to variabilities from the environment [13, 22, 33].

As depicted in Figure 5, the initial step of START is to prompt patients to walk [27]. If a patient can walk following a disaster, this has been shown to be an indicator of low risk and good prognosis [22, 30]. Therefore, patients who can walk are immediately classified as minimal, green, or priority 1. Following this initial step, the remaining non-minimal patients are evaluated based on their respiration, perfusion, and finally mental status. Examples of methods used to assess mental status during START triage include asking patients to perform simple command such as opening and closing their eyes or squeezing a hand [34]. A Yellow tag or delayed status is assigned to all patients that were not originally deemed minimal, but meet the respiratory, perfusion, and mental status criteria set by START. An easy mnemonic to remember the parameters looked at by START is “RPM:30-2-can do”, with RPM standing for Respiration,Perfusion, Mental status. The second portion “30-2-can do” are the associated cut off values for each category: > 30 respirations per minute, presence of radial pulse or capillary refill <2 seconds, and can follow simple commands [26, 27].

Figure 1 START Algorithm to triage patients based on severity

Figure 1: START Algorithm to triage patients based on severity [22, 28].

Though there is little research analyzing the overall efficacy of START, the triage system has demonstrated higher overtriage rates in more critically ill patients during a disaster [28]. In a study by Kahn et. al, START was shown to have an overall accuracy of 44.6% in assigning the correct acuity level for patients. Upon further analysis, START was shown to perform well when identifying patients in the minimal category, with the walk test demonstrating accurate prognostic predictions [35].  However, the triage system experienced higher levels of overtriage (53.38%) in the two more critical patient categories with a significant number of patients inappropriately being placed in the immediate/emergency classification when they belonged in the delayed category [35].

The Sort, Assess, Lifesaving interventions, Treatment/Transport (SALT) algorithm was designed to unify the many existing triage algorithms. Due to a significant lack of research surrounding the efficacy of these triage tools, the Center for Disease Control and Prevention (CDC) assembled a committee in 2008 to combine the most effective features of the current mass triage algorithms into a national standard [22, 28]. This resulted in a new triage algorithm that is very similar to START, but with some key differences. SALT performs a more comprehensive triage of patients of all ages by performing multiple stages of sorting and prioritization with opportunities for reassessment [27]. Additionally, SALT prioritizes life-saving interventions by incorporating them into the triage algorithm leaving less room for hesitancy and uncertainty [13]. The protocol and unique features of SALT can be further elucidated by breaking down what SALT stands for.

            Sort refers to the first step, unique to the SALT protocol, which entails an initial sorting of patients into three groups, prior to formal evaluation, to determine what order patients should be clinically assessed. This pre-sorting is based on their ability to ambulate and perform simple commands (Figure 2) [36]. If patients are able to ambulate, then they are placed in the “Walk” or “Able to walk” group. These patients will be assessed last due to the low risk associated with the ability to ambulate [22, 30]. Patients who are unable to walk, but are able perform purposeful movements such as waving will be placed in the “Wave” or “Able to make purposeful movements category” and assessed second. Finally, patients who have obvious life-threatening injuries or remain still despite prompts to walk or wave are placed in the “Still” or “Severely injured” category. These patients will be clinically evaluated first [27].

Figure 2. Step 1 of SALT algorithm to prioritize who to clinically assess first

Figure 2: Step 1 of SALT algorithm to prioritize who to clinically assess first [22, 27, 28].

The Assess and Lifesaving interventions steps come next and are performed almost simultaneously. Assessrepresents the clinical evaluations that are performed after the three priority groupings have been established during the Sort step. This step is looking for any life-threatening injuries that require immediate stabilization [14]. As threats to life are found during this evaluation, the Lifesaving interventions step calls for immediate medical intervention during triage to stabilize these patients. These interventions typically include opening the airway, hemorrhage control, needle thoracostomy for pneumothoraxes, and antidote auto-injection for poisoning [14, 22].

The last phase of SALT, Treatment/Transport, requires an additional evaluation of patients following lifesaving interventions in order to place patients in the same four priority classifications for evacuation and definitive treatment utilized in START (Figure 3). This step affords the triage an opportunity to reassess patients following life-saving interventions and factor in their response [14]. This reassessment involves many of the same parameters looked at in START including breathing, mental status, and peripheral pulse. However, it also includes a step to consider the patient’s condition in the setting of the resources available [22]. Similar to START, once the patients have been sorted and tagged, they are transported and treated according to their priority group.

Figure 2 Step 2 of SALT algorithm [31, 36, 37].

Figure 3: Step 2 of SALT algorithm [31, 36, 37].

SALT has taken over as one of the major triage algorithms used in disasters and is endorsed by numerous entities including the American College of Surgeons Committee on Trauma, American Trauma Society, National Association of EMS Physicians, National Disaster Life Support Education Consortium, American College of Emergency Physicians, and more [36]. Studies have already begun to show that SALT provides more accurate triaging when compared to START and other triage systems [37, 38]. On the other hand, some studies have observed high levels of overtriage in SALT, similar to those seen in START, and even instances of high undertriage [28, 37, 38]. However, it is important to keep in mind that no clear conclusions can be drawn regarding SALT’s efficacy until more research is performed.

Summary

Disaster Medicine Triage focuses on managing medical care during events that overwhelm local emergency departments, necessitating additional aid. Definitions of “disaster” have evolved to describe situations beyond a community’s coping capability, encompassing natural and man-made incidents like water contamination and power outages. Emergency Medicine, among other healthcare disciplines, plays a crucial role, particularly in disaster response coordination and triage, aiming to do the best possible care for the greatest number. Triage, a core component, involves sorting patients based on their medical needs versus resource availability. With mass casualties, effective use of resources becomes inevitable, underpinning the need for efficient triage to optimize care and resource allocation, emphasizing the utilitarian principle of maximizing survival on a population level.

Authors

Picture of Parker MADDOX BA, MS

Parker MADDOX BA, MS

Parker Maddox is a fourth-year medical student at Sidney Kimmel Medical College at Thomas Jefferson University in Philadelphia. He graduated from the University of Virginia with a double major in Biology and Chemistry and went on to obtain a master’s degree in Biophysics and Physiology at Georgetown University. Since arriving to medical school, Parker has developed a passion for Emergency Medicine and has performed research on a wide range of topics including early sepsis recognition, pandemic viruses including Coronavirus 2019 and Monkeypox, ischemic stroke, Bell’s palsy, and international ECMO critical care protocol. This work has yielded multiple publications and a presentation at the Society for Academic Emergency Medicine (SAEM) 2022 Conference.

Picture of Hassan KHURAM BS, MS

Hassan KHURAM BS, MS

Hassan Khuram is a 4th year medical student at Drexel University College of Medicine, with a background in psychology, biotechnology, and business of healthcare. He graduated Magna Cum Laude with a Bachelor of Science in Psychology from Virginia Commonwealth University and a Master of Science in Biotechnology from Georgetown University. He is passionate about neurocritical care, medical education, and bioethics. He has an extensive background in research, having conducted studies on various subjects, including substance misuse, Parkinson's disease, mindfulness meditation and more. He has published articles on neurological emergencies and ethical issues in neurological care.

Picture of Scott GOLDSTEIN, DO, FACEP, FAEMS, FAAEM, EMT-PHP

Scott GOLDSTEIN, DO, FACEP, FAEMS, FAAEM, EMT-PHP

Dr. Scott Goldstein started his medical career at New York College of Osteopathic Medicine in New York where he received his Doctorate of Osteopathy and continued his training at Einstein Healthcare Network in the field of  Emergency Medicine, Philadelphia. Dr. Goldstein is dual-boarded through the American Board of Emergency Medicine in Emergency Medicine and Emergency Medicine Services (EMS). He currently works at a Level 1 academic trauma center, Temple University Hospital, in Philadelphia where he is the Chief of EMS and Disaster Medicine. He has continued to be an active member of the education community and EMS community where he holds the title of Fellow of American College of Emergency Medicine through ACEP, Fellow of the Academy of Emergency Medical Services through NAEMSP and Fellow of the American Academy of Emergency Medicine through AAEM.  His current academic title is one of Clinical Associate Professor of Emergency Medicine at Lewis Katz School of Medicine at Temple University. 

Listen to the chapter

Cite This Article

Please replace “iEM Education Project Team” below with the author(s) surname and initials.

[cite]

References

  1. Hogan DE, Burstein JL. Disaster Medicine. Lippincott Williams & Wilkins; 2007.
  2. DISASTERS & EMERGENCIES WHO Training Package. Accessed April 12, 2023. https://apps.who.int/disasters/repo/7656.pdf
  3. United Nations International Strategy for Disaster Reduction. Available at: 7817_UNISDRTerminologyEnglish.pdf. Accessed April 12, 2023. https://www.unisdr.org/files/7817_UNISDRTerminologyEnglish.pdf
  4. Ciottone G. Introduction to Disaster Medicine. In: Ciottone’s Disaster Medicine. 2nd ed. Elsevier; 2016:2-5. doi:10.1016/B978-0-323-28665-7.00004-2
  5. KOCAK H, KINIK K, CALISKAN C, ACIKSARI K. The Science of Disaster Medicine: From Response to Risk Reduction. Medeni Med J. 2021;36(4):333-342. doi:10.4274/MMJ.galenos.2021.50375
  6. Wolfe RE. Role of Emergency Medicine in Disaster Management. In: Ciottone’s Disaster Medicine. 2nd ed. Elsevier; 2016:20-26. doi:10.1016/B978-0-323-28665-7.00004-2
  7. Christian M, Farmer J, Young B. Disaster Triage And Allocation of Scarce Resources. In: ; 2008.
  8. Manual for the Health Care of Children in Humanitarian Emergencies. World Health Organization; 2008. Accessed April 15, 2023. http://www.ncbi.nlm.nih.gov/books/NBK143751/
  9. Rimstad R. Mass casualty triage – the greatest good for the greatest number? Tidsskr Nor Laegeforen. 2020;140(14). doi:10.4045/tidsskr.20.0694
  10. Christian MD, Sprung CL, King MA, et al. Triage: Care of the Critically Ill and Injured During Pandemics and Disasters: CHEST Consensus Statement. Chest. 2014;146(4):e61S. doi:10.1378/chest.14-0736
  11. Najafi Z, Abbaszadeh A, Zakeri H, Mirhaghi A. Determination of mis-triage in trauma patients: a systematic review. Eur J Trauma Emerg Surg. 2019;45(5):821-839. doi:10.1007/s00068-019-01097-2
  12. Lentz BA, Jenson A, Hinson JS, et al. Validity of ED: Addressing heterogeneous definitions of over-triage and under-triage. Am J Emerg Med. 2017;35(7):1023-1025. doi:10.1016/j.ajem.2017.02.012
  13. Foley E, Reisner A. Triage. In: Ciottone’s Disaster Medicine. 2nd ed. Elsevier; 2016:337-343. doi:10.1016/B978-0-323-28665-7.00004-2
  14. Bazyar J, Farrokhi M, Salari A, Safarpour H, Khankeh HR. Accuracy of Triage Systems in Disasters and Mass Casualty Incidents; a Systematic Review. Arch Acad Emerg Med. 2022;10(1):e32. doi:10.22037/aaem.v10i1.1526
  15. Committee on Trauma ACoS. Resources for Optimal Care of the Injured Patient. American College of Surgeons; 2014. Accessed April 13, 2023. https://www.facs.org/media/yu0laoqz/resources-for-optimal-care.pdf
  16. Cotte J, Courjon F, Beaume S, et al. Vittel criteria for severe trauma triage: Characteristics of over-triage. Anaesthesia Critical Care & Pain Medicine. 2016;35(2):87-92. doi:10.1016/j.accpm.2015.06.013
  17. Frykberg ER. Medical management of disasters and mass casualties from terrorist bombings: how can we cope? J Trauma. 2002;53(2):201-212. doi:10.1097/00005373-200208000-00001
  18. ACS Highlight: Trauma Team Activation: Optimizing Prehospital Triage of the Injured Patient. The American Association for the Surgery of Trauma. Published November 1, 2021. Accessed April 13, 2023. https://www.aast.org/disaster-detail/acs-highlight-trauma-team-activation-optimizing-pr
  19. Hick JL, Hanfling D, Cantrill SV. Allocating Scarce Resources in Disasters: Emergency Department Principles. Annals of Emergency Medicine. 2012;59(3):177-187. doi:10.1016/j.annemergmed.2011.06.012
  20. Jenkins JL, McCarthy ML, Sauer LM, et al. Mass-Casualty Triage: Time for an Evidence-Based Approach. Prehosp Disaster med. 2008;23(1):3-8. doi:10.1017/S1049023X00005471
  21. Christian MD. Triage. Critical Care Clinics. 2019;35(4):575-589. doi:10.1016/j.ccc.2019.06.009
  22. Bazyar J, Farrokhi M, Khankeh H. Triage Systems in Mass Casualty Incidents and Disasters: A Review Study with A Worldwide Approach. Open Access Maced J Med Sci. 2019;7(3):482-494. doi:10.3889/oamjms.2019.119
  23. Kennedy K, Aghababian RV, Gans L, Lewis CP. Triage: techniques and applications in decision making. Ann Emerg Med. 1996;28(2):136-144. doi:10.1016/s0196-0644(96)70053-7
  24. Taran S. The Scoop and Run Method of Pre-clinical Care for Trauma Victims. McGill Journal of Medicine : MJM. 2009;12(2). Accessed April 15, 2023. https://www.ncbi.nlm.nih.gov/pmc/articles/PMC2997263/
  25. Haas B, Nathens AB. Pro/con debate: Is the scoop and run approach the best approach to trauma services organization? Critical Care. 2008;12(5):224. doi:10.1186/cc6980
  26. Iserson KV, Moskop JC. Triage in Medicine, Part I: Concept, History, and Types. Annals of Emergency Medicine. 2007;49(3):275-281. doi:10.1016/j.annemergmed.2006.05.019
  27. Clarkson L, Williams M. EMS Mass Casualty Triage. In: StatPearls. StatPearls Publishing; 2023. Accessed April 15, 2023. http://www.ncbi.nlm.nih.gov/books/NBK459369/
  28. Lee CH. Disaster and Mass Casualty Triage. AMA Journal of Ethics. 2010;12(6):466-470. doi:10.1001/virtualmentor.2010.12.6.cprl1-1006
  29. Foley E, Reisner A. Triage. Ciottone’s Disaster Medicine. Published online January 1, 2016:337-343. doi:10.1016/B978-0-323-28665-7.00054-6
  30. Meredith W, Rutledge R, Hansen AR, et al. Field Triage of Trauma Patients Based upon the Ability to Follow Commands: A Study in 29,573 Injured Patients. Journal of Trauma and Acute Care Surgery. 1995;38(1):129.
  31. WMA – The World Medical Association-WMA Statement on Medical Ethics in the Event of Disasters. Accessed April 15, 2023. https://www.wma.net/policies-post/wma-statement-on-medical-ethics-in-the-event-of-disasters/
  32. Gabbe BJ, Cameron PA, Finch CF. Is the Revised Trauma Score Still Useful? ANZ Journal of Surgery. 2003;73(11):944-948. doi:10.1046/j.1445-1433.2003.02833.x
  33. START Adult Triage Algorithm – CHEMM. Accessed April 16, 2023. https://chemm.hhs.gov/startadult.htm
  34. Disaster Preparedness Section – Los Angeles Fire Department Manual. Accessed April 16, 2023. https://www.cert-la.com/downloads/education/english/start.pdf
  35. Kahn CA, Schultz CH, Miller KT, Anderson CL. Does START triage work? An outcomes assessment after a disaster. Ann Emerg Med. 2009;54(3):424-430, 430.e1. doi:10.1016/j.annemergmed.2008.12.035
  36. SALT Mass Casualty Triage: Concept Endorsed by the American College of Emergency Physicians, American College of Surgeons Committee on Trauma, American Trauma Society, National Association of EMS Physicians, National Disaster Life Support Education Consortium, and State and Territorial Injury Prevention Directors Association. Disaster med public health prep. 2008;2(4):245-246. doi:10.1097/DMP.0b013e31818d191e
  37. Hartman EN, Daines B, Seto C, et al. Sort, Assess, Life-Saving Intervention, Triage With Drone Assistance in Mass Casualty Simulation: Analysis of Educational Efficacy. Cureus. 2020;12(9). doi:10.7759/cureus.10572
  38. McKee CH, Heffernan RW, Willenbring BD, et al. Comparing the Accuracy of Mass Casualty Triage Systems When Used in an Adult Population. Prehospital Emergency Care. 2020;24(4):515-524. doi:10.1080/10903127.2019.1641579

Reviewed By

Picture of Arif Alper Cevik, MD, FEMAT, FIFEM

Arif Alper Cevik, MD, FEMAT, FIFEM

Prof Cevik is an Emergency Medicine academician at United Arab Emirates University, interested in international emergency medicine, emergency medicine education, medical education, point of care ultrasound and trauma. He is the founder and director of the International Emergency Medicine Education Project – iem-student.org, chair of the International Federation for Emergency Medicine (IFEM) core curriculum and education committee and board member of the Asian Society for Emergency Medicine and Emirati Board of Emergency Medicine.

Approach to the trauma patient – ABCDE of trauma care

Approach to the trauma patient – ABCDE of trauma care

Case

Jane Doe, 22-year-old female, was in a major car crash and is approaching the trauma bay via an ambulance. You are aware that the patient’s condition is critical, so you do a quick run-through in your head about the approach that you will have to care for them once they arrive to your emergency department. What should your approach to a trauma patient be?

The ABCDE of Trauma Care

The Airway, Breathing, Circulation, Disability, Exposure (ABCDE) approach is a clinically proven approach to any critically ill patient that needs emergent care and treatment. It has been proven to improve patient outcomes, optimize team performance and save time when patients are in life-threatening conditions [1]. This approach is applicable to all patients (both adults and children), regardless of their underlying condition. However, the ABCDE approach is not applicable to patients who are in cardiac arrest, in which case the cardiopulmonary resuscitation guidelines should be used [2].

With the ABCDE approach, initial assessment and treatment are performed simultaneously. Once the entire survey is completed, reassessment should be conducted until the patient is stable enough for the care team to be able to move on to the secondary survey and look for a definitive diagnosis.

A - Airway

First, the care team should assess if the patient’s airway is patent. If the patient responds to the team in a normal voice, then that is a good sign that the airway is intact. It is important to note that airway obstruction can be complete or partial, and can be caused by upper airway obstruction or reduced level of consciousness.

Signs of complete airway obstruction are lack of respiration despite great effort. Signs of partial airway obstruction include:
– Changes in the patient’s voice
– Snoring or gurgling
– Stridor (noisy breathing)
– Increased breathing effort

Assess the patient’s airway by looking for rocking chest wall motion and any signs of maxillofacial trauma or laryngeal injury. Perform the head-tilt and chin-lift maneuver to open the airway (note that caution should be conducted in patients with C-spine injury). If there is anything that is noticeably obstructing the airway, suction or remove it. If possible, remove foreign bodies that are causing airway obstruction. Provide high-flow oxygen to the critically ill patient and perform definitive airway if needed [1].

B – Breathing

Generally, airway and breathing are examined simultaneously. Determine if breathing is intact by assessing the respiratory rate, inspecting the chest wall movement for symmetry, depth, and respiratory pattern. Additionally, assess for tracheal deviation and use of respiratory muscles. Percuss the chest for dullness or resonance, auscultate for breath sounds and apply a pulse oximeter [1].

Injuries that impact breathing should be immediately recognized, and life-threatening injuries should be addressed and managed [3]. For example, tension pneumothorax must be promptly relieved by needle thoracocentesis, bronchospasms should be managed with inhalation and assisted ventilation should be considered if breathing continues to be insufficient [1].

C – Circulation

Conditions that threaten the patient’s circulation and can be fatal include shock, hypertensive crises, vascular emergencies such as aortic dissection and aortic aneurisms. These conditions should be immediately identified and managed [1].

Circulation can be assessed by looking at the general appearance of the patient, including signs of cyanosis, pallor, flushing and diaphoresis. Assess for any obvious signs of hemorrhage, blood loss and level of consciousness. Additionally, capillary refill time and pulse rate should be assessed. Auscultate the chest for heart sounds, and blood pressure measurement and electrocardiography should be performed as soon as possible [1].

Additionally, assess for signs of hypovolemia and shock. If these are identified, obtain an intravenous access and infuse saline to restore circulating volume [1]. If there are life-threatening conditions that are compromising the patient’s circulation, promptly identify and treat them as needed. For example, tension pneumothorax should be immediately treated with needle decompression and cardiac tamponade can be relived with pericardiocentesis.

D - Disability

The main disability in the primary survey to be assessed for is the brain. Abnormal neurological status can be caused by primary brain injury or systemic conditions that effect brain perfusion, such as shock, hypoxia, intoxication etc. Assess the level of consciousness by using the Glasgow Coma Scale [4], look for pupillary response and limb movement.

The best way to prevent injury to the brain is to maintain adequate airway, breathing and circulation. Glucose levels can be assessed at bedside for decreased level of consciousness due to low blood glucose levels, and corrected with oral or infused glucose [1].

E – Exposure

The exposure portion of the ABCDE approach involves assessment of the whole-body to avoid any signs of missing injuries. During this part of the management, undress the patient fully and examine the back for any signs of C-spine precautions. Additionally, check for clues for any signs of underlying conditions, such as:

  • Signs of trauma (i.e. burns, gunshot wounds, stab wounds)
  • Rashes
  • Causes of sepsis (i.e. infected wounds, gangrene)
  • Toxins and drugs (i.e. needle track marks, chemicals, patches)
  • Other wounds such as bite marks, insect bites, embedded ticks
  • Iatrogenic causes (i.e. catheters, tubes, implants, surgical sites and scars)

Concluding Remarks

The ABCDE approach to the critically ill patient is a strong and proven clinical tool for initial assessment and treatment of patients in medical emergencies. Widespread knowledge of this skill is critical for healthcare workers and any team providing emergent care to trauma patients. 

*Note that this is a general approach to the trauma patient. Always consult your care team for adequate management of trauma patients and resort to reliable resources for more information on the ABCDE approach. 

References and Further Reading

  1. Thim, T., Krarup, N. H. V., Grove, E. L., Rohde, C. V., & Løfgren, B. (2012). Initial assessment and treatment with the Airway, Breathing, Circulation, Disability, Exposure (ABCDE) approach. International journal of general medicine5, 117.
  2. Koster, R. W., Baubin, M. A., Bossaert, L. L., Caballero, A., Cassan, P., Castrén, M., … & Sandroni, C. (2010). European Resuscitation Council Guidelines for Resuscitation 2010 Section 2. Adult basic life support and use of automated external defibrillators. Resuscitation81(10), 1277-1292.
  3. Subcommittee, A. T. L. S., & International ATLS Working Group. (2013). Advanced trauma life support (ATLS®): the ninth edition. The journal of trauma and acute care surgery74(5), 1363-1366.
  4. Sternbach, G. L. (2000). The Glasgow coma scale. The Journal of emergency medicine19(1), 67-71.
[cite]

Question Of The Day #70

question of the day
712 - deep fore arm laceration
Which of the following is the most appropriate next step in management for this patient’s condition?  

This patient arrives to the hospital after a suicide attempt with multiple bleeding arm wounds, hypotension, tachycardia, and a depressed mental status. This patient is in hemorrhagic shock.

The first step in evaluating any trauma patient involves the primary survey.  The primary survey is also known as the “ABCDEFs” of trauma.  This stands for Airway, Breathing, Circulation, Disability, Exposure, and FAST exam (Focused Assessment with Sonography in Trauma).  Each letter should be assessed in alphabetical order to avoid missing a time sensitive life-threatening condition.  The primary survey should be conducted prior to taking a full history.  After the primary survey, a more detailed physical exam (secondary survey) is conducted, followed by interventions and a focused patient history. 

The airway and breathing status of this patient have been assessed with no acute issues as noted in the question stem.  On assessment of the patient’s circulation, he is tachycardic, hypotensive, and has an actively bleeding extremity wound.  The first step in managing a bleeding wound is to apply constant direct pressure to the site.  Direct pressure to the site for 15 minutes should control bleeding in most cases.  If the origin of the bleeding is difficult to identify for direct pressure application, or if direct pressure fails, the next step is to apply a tourniquet (Choice D).  If a tourniquet is not available, an easy alternative is to apply a blood pressure cuff proximal to the bleeding wound and inflate the cuff to 250mmHg or until the bleeding stops.  This will allow careful examination and repair of the bleeding wound.  Topical tranexamic acid (Choice A) and subcutaneous lidocaine with epinephrine injections (Choice B) can work as adjuncts to tourniquet application.  Suturing the area will also help tamponade the bleeding site and aid in clot formation after a tourniquet is applied.  Checking a serum toxicology screen (Choice C) may be helpful to evaluate for a concurrent overdose, but this is not as important as initial hemorrhage management.   

Other steps to hemorrhage control and treatment include establishing large bore IV access, administering IV fluids or blood products as needed, and reversing coagulopathy. Correct Answer: D

References

[cite]

Question Of The Day #69

question of the day
Neck injury with fish
Which of the following is the most appropriate next step in management for this patient’s condition?  

The neck is a compact anatomical area with many vital structures, including blood vessels that provide oxygen to the brain, the aerodigestive tracts (trachea and esophagus), nerves, and the apices of the lungs.  A penetrating injury to the neck can be catastrophic and requires prompt examination and appropriate management.  The neck is divided into 3 anatomical zones, and each zone houses different anatomical structures.  Zone 1 is from the clavicle to the cricoid cartilage, Zone 2 is from the cricoid cartilage to the mandible, and Zone 3 is from the angle of the mandible to the base of the skull.  See the reference below for pictures and further descriptions of each zone.

The presence of any “hard signs” of aerodigestive or neurovascular injury should prompt emergent operative management.  These “hard signs” include airway compromise, expanding or pulsatile hematoma, active and brisk bleeding, hemorrhagic shock, neurological deficit, massive subcutaneous emphysema, and air bubbling through the wound.  If the patient is hemodynamically stable and does not have any of these dangerous “hard signs”, it is reasonable to pursue CT angiography of the neck (Choice A) to evaluate for any vascular, aerodigestive, or neurologic injuries.  The fish should not be removed (Choice B) in the Emergency department as this may result in uncontrolled bleeding.  A more controlled environment, like an operating theater, is a more appropriate setting to remove a penetrating foreign body.  The patient in this case has 2 hard signs (bubbling through wound and airway compromise), so he will need operative management (Choice C).  However, the patient’s airway compromise is a more emergent and time-sensitive issue that needs to be addressed first with endotracheal intubation (Choice D).  Intubation is the next best step in management.  Correct Answer: D

References

[cite]